飛行機ってなんで空を飛べるんですか?2

このエントリーをはてなブックマークに追加
1 
2前スレの920:2001/06/19(火) 07:38 ID:KGDrr0js
水中モーターなんですけど、ごめんなさい。ごめんなさい。
まだ、水中モーターにこだわってます。

えっと、水中モーターが沈むように重さを調整しても、
羽とかボディに水の抵抗を受けながら進むのなら、
等速で下降すんだと思ったんですけど〜。

違うか...こんど実験してみよ。
3前スレの920:2001/06/19(火) 07:42 ID:KGDrr0js
あと、2ch的にはどうでもいいことですけど、霊とか言ってるのは
私じゃないです。

って、しょっちゅう、IDが変わるんじゃ説得力無いか。宇津だ...
4ご冗談でしょう?名無しさん:2001/06/19(火) 11:35 ID:lrO/cGVI
>>976
翼端に付けるんじゃなくて、発進時に翼端に置くんです。
発進渦を大きく取る為。
発進して速度が出れば必要無い。
滑走中に翼に付けておいて、離陸する直前に取り外しても良いw)
5ご冗談でしょう?名無しさん:2001/06/19(火) 12:31 ID:???
>>4
ケルビンの循環定理があるから翼周りに循環が生じると
思い込んでる可哀想なコドモだ。誰か救ってやったら?
知ったかブリブリの教科書コピペ厨房の嵐の悪影響だな。
6ご冗談でしょう?名無しさん:2001/06/19(火) 13:48 ID:???
>>2 (=前スレの920)
前スレの921で強烈に書いた者だけど、前スレ969の記
述から見るに背面飛行時の力の釣り合い理解してない
でしょ。まあいいけど...。

水中モーターだけど、水中にモーター推進で翼を持った
飛行機(水中艇?)があるという仮定かい。そして浮力
は重力に対して充分小さいという仮定も付く訳だよね。

そうすれば飛行機と(微小浮力を除いて)全く同じ。
水中モーターを止めた場合は紙飛行機・グライダーと同
様に、前進しながら一定の速度(加速度じゃないよ)で
深度を増していく(=飛行機の場合は高度を失っていく)

水中モーターを回した場合は、その推進力、水中艇の翼
の性能(詳しくは揚力/抗力比)に依るけれども、水平
前進が可能。空中の飛行機と同じ。

前スレ969の参考文献は(バカにしてるわけじゃなくて)
君にとって難しすぎ。pdfファイルのほうは飛行の原理
には直接関係ないし、もう一方のは流体力学をある程度
理解してないとわからない。
7ご冗談でしょう?名無しさん:2001/06/19(火) 15:38 ID:???
>>5
私はどちらかというと、その発進渦という奴は副次的に出来る
もので、それが原因で循環が起こるなんて信じて無いほうで、
そういうニュアンスの説があるんで、もしそうなら、それを
もっと積極的に利用して飛ぶんじゃないかな?と皮肉めいた
意味で前スレで質問したのだが
8どしろうと:2001/06/19(火) 15:55 ID:j8CReOns
思うのですが、ジェット旅客機などはジェットタービンが翼の下についていますよね?
ってことは翼の下部に高速の空気の流れがあるので、
ベルヌーイの定理で、流速が速いのなら圧力が下がるらしいですからこれって無駄な下向きの力が働いていませんか?
逆にジェットタービンを翼の上に設けたのなら翼の上の圧力が減って揚力が大きくなっていいのでは?
圧縮してるから意味ないのかなあ。
でもプロペラ機なら意味ありそうな気がするのですが・・。
プロペラは羽の下より上につけたほうがいいのではないのですか?
それともタービン(プロペラ)が翼の下にあるほうが安定しているということだけで
だいたいのひこうきは下に取りつけているんですか?
92:2001/06/19(火) 19:31 ID:KGDrr0js
>>6 さん

>背面飛行時の力の釣り合い理解してないでしょ。まあいいけど...。

はい、ごめんなさい。多分わかってないんだと思います。
で、水中艇の話なんですけど、モーターが回ってるときだけ考えて
ください。鉛直方向には、翼の揚力、重力、ちっちゃい浮力の
3つの力が水中艇に働いているってことでいいんでしょうか〜。
correctしてくださいねー。

んで、翼の揚力は循環の符号で上向きか下向きかが決まるんですよね!
翼の前先端が<<下を向いているとき>>は、循環の符号は(前の927での)
+で、揚力が下向きに働くんですよね。これでいいのかな?

だ・か・ら! 下向きの力は、揚力と重力。上向きはちっちゃい浮力だけ。
鉛直方向の合力は下向き!!
それで、鉛直方向には等加速度でsageって逝くんだと考えたんですけど。
あらら...6さんの言ってることと違うー。

そーいえば、サメは浮袋がないのでずっと泳いでないと沈んじゃうそう
です。
10ご冗談でしょう?名無しさん:2001/06/19(火) 21:53 ID:NmnrJgm2
もんだい

今、断面が非常に薄いベルト状の金属テープがあるとします。
両端は繋がっており、ちょうど女性が使うガーターベルトみたい
なものと想像して下さい。
これを翼に何本も巻き付けてそれを覆います。
ただし、ベルトの表面は十分滑らかで、ベルトが露出している
のは、翼上表面だけで、下半分のベルトは、翼の中に入り込んで
隠れているものとします。
下手な図ですが、一応この状態での翼の断面図を

           ↓ベルト
     ┌──────────────────────────┐
     │   ──   ↓翼上面(のつもり)                 │
     │ ──   ──────              │
     ├─           ─────         │
   ──└──────────────────────────┘
 ──                          ──────
─────────────────────────────────────
↑翼下面(のつもり)

それを翼の前後方向に、何枚も並列に並べて覆います。
上から見ると、翼は縦に沢山のベルトで覆われているように
見えるでしょう。
ベルトは、翼前端で翼の中に入り、翼後端で、翼から出て
再び上面に表れます。(ベルトは十分弾力があるとし、弛ま
ないとします。かつ表面も十分に滑らか)
今、ベルトは翼内部のモーターによって高速回転出来るように
なっているします。つまり、翼上表面は、静止状態でも任意の
速度で(表面だけが)動くことが出来るようになっているというわけです。
つまり、表面だけ動かすことが出来る翼(藁)を作ってみたというわけです。
翼の表面にベルトコンベアを何ラインも設置したということです。

境界層の理論から言って、ベルト表面の空気の速度はベルトの
移動速度と同じになる筈です。(空気が翼全体の移動速度を
認識できるというのなら話は別になりますが、この場合、明らかに
物理的証明が必要になります。空気はヴァカでしょうから、目についた
表面だけの速度を認識する筈です。)
飛行中にベルトを適切に動かすと、空気力学的には、より高速に飛ぶ
ことと同じになるでしょう。
この時、果たして循環は増大するでしょうか?
下降気流の増大についてはどうでしょうか?
揚力の寄与はどうなるのでしょうか?
もし寄与すると言うのならば何故(比較的ローコストで可能な
技術であるのにもかかわらず)実現されていないのでしょうか?
11名盤さん:2001/06/19(火) 22:31 ID:7eXgOrqA
それ自分も思った。興味Age
12ご冗談でしょう?名無しさん:2001/06/19(火) 22:52 ID:???
翼の上面はデリケートだから、そんなことしたら剥離して抵抗力が増えるよ。
13ベル厨:2001/06/19(火) 23:22 ID:???
8と10の質問に答えてチョ。
14ご冗談でしょう?名無しさん:2001/06/19(火) 23:39 ID:???
>>9 さん(=前スレの920)
ゴメン、あなたが何を知りたいかが今ひとつ分からない。
水中でも背面で進行してるって事かい?結局、前スレ920
の質問(背面飛行時の力の釣り合い)を解決したいのかい?

空中に話しを戻すけど、通常の飛行の場合は鉛直方向の
力は下向きの重力と上向きの揚力(空気の浮力は無視)で
あって、その二つの力は釣り合っている。
背面飛行の場合も、全く同じで、鉛直方向の力は下向きの重
力と上向きの揚力であって、その二つの力は釣り合っている。
だから鉛直方向には速度0乃至は等速(等加速度じゃないよ)
運動をする。

背面飛行の場合は、操縦者からみて機首下げにして、翼に
当たる空気流れの角度(迎え角)を定常飛行と同じ様に+
のままに(厳密には値はちょっと違うけど)してるだけ。
だから、この場合も揚力は鉛直方向上向きに働く。

「翼周りの循環」ってのは私は理解してないんだけど、
この説明であなたの疑問は解決したかな?
15ベル厨:2001/06/19(火) 23:44 ID:???
Age
16ご冗談でしょう?名無しさん:2001/06/20(水) 00:14 ID:r05TpwM2
>>8
ジェットエンジンや、プロペラを通ってきた空気は仕事をされているので、
エネルギーが保存しておらず、ベルヌーイの定理は成り立ちません。

前スレの650でも読むといいでしょう。
ベルヌーイの定理は、簡単に言うと、「単位体積あたりのエネルギー保存則」なので、
流体が仕事をされると成り立ちません。
単純に、流速が上がると圧力が下がるわけではありません。
17ベル厨:2001/06/20(水) 00:23 ID:???
10の方法で揚力得られますか?
18ご冗談でしょう?名無しさん:2001/06/20(水) 00:26 ID:iYa46Xgw
>>8
はじめに開発された翼した吊り下げ式の実用ジェット機は、
そもそもエンジンの吸気排気部が
翼から前後にはみ出していました。
だから翼にかかる空力には影響がありません。
旅客機等の吊り下げ式のエンジンは、
翼面から十分離れていて、
空気の流れと、
翼面の空気の流れはあまりかかわりをもたないように
なっていますよ。
19ベル厨:2001/06/20(水) 00:48 ID:???
10の方法で揚力得られますか?
2010:2001/06/20(水) 00:52 ID:???
>>12
そのとおり
21ベル厨:2001/06/20(水) 00:56 ID:???
10の方法を静止状態でやっても揚力は生まれないのですか?
22ご冗談でしょう?名無しさん:2001/06/20(水) 02:03 ID:???
揚力=ρUΓ (Uは翼の速さ)
23ベル厨:2001/06/20(水) 02:14 ID:???
空気に対しての速さですよね?だったら揚力はあるってこと?
24名無しさん@お腹いっぱい。:2001/06/20(水) 03:46 ID:rrAIJ9fY
age
2510:2001/06/20(水) 05:41 ID:???
>>23(=21,=19,...でしょ?)
こちらもそれなりに邪悪な質問をしているんだから、
そうせっつかず、それなりに説得力のある解答が出るまで
じっくり待ちましょうや
26ご冗談でしょう?名無しさん:2001/06/20(水) 10:51 ID:???
>>10
この方法では、揚力は得られないでしょう。
空気はそんなヴァカではありません。

摩擦抵抗の変化はあると思います。
27ご冗談でしょう?名無しさん:2001/06/20(水) 10:56 ID:???
>>25
そんなに邪悪でもないでしょ。
円筒を回転させて半分を軸受で覆った状態と考えてもいいかな。
半分だけお椀に入ったボールを回転させると浮き上がるかって
質問だな。お椀の少し前側を持ち上げた状態か。
28ご冗談でしょう?名無しさん:2001/06/20(水) 11:05 ID:???
>>27
>お椀の少し前側を持ち上げた状態か。
これは
 お椀の前側を少し持ち上げた状態か。
です。下降気流のことを念頭に置いたのね。

栄光(?)の27をゲットだな(藁
29ご冗談でしょう?名無しさん:2001/06/20(水) 12:05 ID:???
>>27
根拠は無いが27はとりあえず死ね
30ベル厨:2001/06/20(水) 17:29 ID:???
だれも答えられないのかな?
31age:2001/06/20(水) 20:25 ID:???
Age
32しゃらく:2001/06/20(水) 20:42 ID:r7OzP.Rc
正確かどうかわかんないですけど説明してみますね。

飛行機の羽は上が丸くなっていて、下が平たいですよね
羽の前の空気が羽の後ろに行くときには、
羽の下を通った場合より、上を通った場合のほうが長いので
同じ量の空気が羽の上下を通った場合、
上にある空気の密度が薄くなります。
このため、飛行機が速く進むほど、
空気が下から押し上げる力が強くなるわけです。
33:2001/06/20(水) 20:49 ID:???
前スレ読んできなさい。
34ご冗談でしょう?名無しさん :2001/06/20(水) 21:43 ID:???
>>32
うけた...(笑
わざとやってる?>>33と同じ忠告を君に送るよ
35ご冗談でしょう?名無しさん:2001/06/20(水) 22:27 ID:???
他の生き物はみんなおとなしく、地球という星を大切にして環境に適応して生きているのに、

どうして人間だけ、いろいろバカな事ばかりするんだろ。

社会不適合者とかを非難する前に、まず人間それ自体が地球不適合者じゃん・・・
36ご冗談でしょう?名無しさん:2001/06/21(木) 00:15 ID:???
35はあちこちのスレに柿混んでいる。無関係と尾もわれ
しかし意味深
37 :2001/06/21(木) 00:21 ID:???
>>10の方法を静止状態でやっても揚力は生まれないのですか?
3810:2001/06/21(木) 01:21 ID:???
君はどう思うのよ?>>37
39>10:2001/06/21(木) 02:15 ID:???
分かりません。だからだれか答えてよ。
40ご冗談でしょう?名無しさん:2001/06/21(木) 02:43 ID:???
煽ったら教えないって27に言われたんじゃないのかい?
41 :2001/06/21(木) 03:50 ID:???
煽ってないよ。
4226:2001/06/21(木) 09:40 ID:???
10の質問は前にいちおう答えたつもりなんだが。まあいいや。

静止状態のとき、ベルトは、擬似的な流体に対して平行にしか動いていない。
翼上面で斜めになっている気がするけど、ベルトが動いているんだから、
ベルトが傾いても、ベルトに対して流体は常に平行だ。

平行だと揚力は起きないのかというと、ずばり揚力は起きない。

ベルヌイを使うと叱られそうだが、ただの板を水平に飛ばした場合、
静止状態と比べて流速(動圧)が上がっているけど、その代わりに
静圧が下がっているわけではない。
ベルヌイの定理は、流体の流れ道の太さを変化したときの保存則だ。
43宗之(子持ち):2001/06/21(木) 11:25 ID:???
いいかい、あれだけ高く空を飛べるというのは
ものすごい抵抗があるからなんだよ。
だから君もこんな事でくじけちゃいけない。
もっとすごい抵抗のなかでがんばるんだ。
いいかい、がんばるんだ。
44名無し三等兵:2001/06/21(木) 11:40 ID:x67AhxC2
すいません、軍事板から来ました。


http://yasai.2ch.net/test/read.cgi?bbs=army&key=989654718
の、戦時日本のエンジン事情なるスレなのですが、
途中から翼面積と空気抵抗についての話が出てきて混乱しつつあります。
ずばっと結論を見、話を正常化させる為にもお力を貸していただけないでしょうか。
45ご冗談でしょう?名無しさん:2001/06/21(木) 11:55 ID:KCCE60eU
あれよあれよというまに2つめのスレですね。
1スレできた直後に読んだけどそれ以来よんでなかった。
そうとう盛り上がってるようなんで最初から読んで見ます。
46ご冗談でしょう?名無しさん:2001/06/21(木) 12:27 ID:???
>>44 混乱してるようにはみえない。スレの主旨については
わからんが、「翼面積と空気抵抗についての話」については
ほぼ結論がでてるみたいだけど?
47ご冗談でしょう?名無しさん:2001/06/21(木) 16:40 ID:nzXRdSHk
10は要するに理論的には翼の上に発生する筈の境界層を除去することができる
と主張しているのだろう。
境界層のある無しで、揚力はその存否も含めて大きな影響を受けるのかどうか
ってことを問題にしたんじゃないの?
クッタージューコフスキーの定理を基本に置く理論では境界層は、実在気体
が粘性を持つ以上必要不可欠な存在であり、これが無ければ抗力も揚力も発生
しない...んでしょ。
48ご助力感謝:2001/06/21(木) 17:28 ID:???
>>46
某あるつ変 でよいのでしょうか^^;;
49>47:2001/06/21(木) 19:40 ID:???
ということは10の方法で揚力は生まれるんですか?
50 :2001/06/21(木) 21:05 ID:???
Age
51 :2001/06/21(木) 23:05 ID:???
Age
52 :2001/06/22(金) 00:24 ID:???
なんかだれもかたえなくなっちゃった?
53ご冗談でしょう?名無しさん:2001/06/22(金) 00:38 ID:???
誰か、前スレの議論の進行を旨くまとめてくれたら、
議論復活するんじゃないの?
10の質問見ても何が問題なのか今市良くわからぬ。
54 :2001/06/22(金) 00:40 ID:???
10のは又別問題として考えて。
55ご冗談でしょう?名無しさん:2001/06/22(金) 01:53 ID:2tTV0K2Q
>54
どうして別の問題?
>>10 が言ってるのは変化球の原理とおなじことなのでは?とおもうんだが。
いわゆるマグヌス効果ってやつ。
揚力は得られる。ただし、主流速度があればのはなしだけどね。
56ご冗談でしょう?名無しさん:2001/06/22(金) 03:31 ID:???
前スレの27氏(個人・複数かは知らないが)は一応ストーミーな進行役かそういう役割を果たした。氏の主張を独断と偏見でまとめた。

・翼の上が下よりも長いから上を通る風の速度が大きい...というのは間違いか、非常に
不正確である。(上を通る風速が大きいのは実験事実だから、因果関係の説明として
不適切というのであろう。)もし仮にそれを認めても翼の上を通過する際に空気の粘性
によるエネルギー散逸や翼が為す仕事を無視することは出来ないので、単純にベルヌーイ
の定理を使って翼上の圧力低下を導き出すことは厳密性を欠く。

・揚力は本質的にクッタージューコフスキーの流れが、発生することによって起こる。
循環が0でない流れである。
(この流れは速さが非有界である、つまり最速点が無いことに注意)これによっても
抗力の存在が説明出来ないが、これは境界層理論を組み合わせて説明出来る。
(境界層理論では、翼付近で気流が翼に粘着し、それ以外の場所では粘性を無視出来る。
粘性の無い流体として扱うことが出来、また流速も、翼表面付近以外では遷音速領域以下
なので、圧縮性を無視でき、結局完全流体として扱えるので、クッタージューコフスキー
の流れが起こる為の要件を満たすだけでなく、抗力も説明出来る。
何故このような(特殊な)流れが発生するのかについては納得のいく言及が無い。

・揚力は、クッタージューコフスキーの流れが発生することにより発生する、翼上面の
下降気流の反動とも解釈できる。この下降気流は、空気と翼の粘性による相互作用
から発生する。
運動量の保存則から、翼+空気(地球全体)+地球のすべての運動量の
和が時間不変であるということは言える。
これは運動量の分布については何も言わず、非常に分散的になっている場合
可能性も含んでいる。
しかし氏の主張では、より強く、運動量の分布は翼の上付近の下降気流に集中してい
るとしている。つまり、翼の質量をMとした時、翼の上付近の下降気流の運動量は
Mgとなるという。

・氏は理想気体は完全流体であるとしている。
57age:2001/06/22(金) 13:01 ID:???
Age
58>>1:2001/06/22(金) 14:43 ID:PckpjJv2
前スレのどこにもまともなこのスレへのリンク貼っとらん。
継続スレたてるときには、前スレに次スレのリンク貼るべき。
しかも前スレで別の名前のスレがいつの間にか勝手に継続
スレにされている。(尻切れ状態になったようだが)
前スレ既に「嵐」によって、書き込み不能にされていて、
今さらリンク張れない。
このスレ建てた貴方は、ぜひとも、前スレの最後の方の
「嵐」部分を削除依頼して、リンク貼ってくるべき。
5958:2001/06/22(金) 14:48 ID:PckpjJv2
60>58:2001/06/22(金) 22:34 ID:???
ハァ?
61前スレの:2001/06/22(金) 23:44 ID:WfLqMfSI
27は突然どこへいったんだ?
62ご冗談でしょう?名無しさん:2001/06/23(土) 00:10 ID:???
ってより、前スレ削除されたみたいだよ。どういうわけか知らない
が。
63ご冗談でしょう?名無しさん:2001/06/23(土) 00:32 ID:???
http://cheese.2ch.net/sci/kako/988/988866699.dat
>>62
過去ログぐらい見れ(まだdatだが)
64ご冗談でしょう?名無しさん:2001/06/23(土) 00:33 ID:???
前スレ無くなったンでとりあえず、終了かな?
結論:
ベルヌーイの定理を安直に使わず、クッタージューコフスキー
等の先人にも敬意を払え。
安直に運動量だけで考察するな!
ヒコーキは失速が恐い

ってとこかな?
65ご冗談でしょう?名無しさん:2001/06/23(土) 00:52 ID:???
つーか「流体力学」では、揚力を物理的には理解できないらしい。
ゼミスレッドみれ
66ご冗談でしょう?名無しさん:2001/06/23(土) 00:57 ID:39j3dJTA
10の方法で揚力得られるのか〜。
67ご冗談でしょう?名無しさん:2001/06/23(土) 04:52 ID:weZXm1Is
>66
得られる。
得られない理由をいってくれ。
68ご冗談でしょう?名無しさん:2001/06/23(土) 05:32 ID:ui3F/05Q
回転凧って知ってる?
69ご冗談でしょう?名無しさん:2001/06/23(土) 06:16 ID:qvoVl/go
なんで揚力得られるのですか?>67

回転凧ってどういうものなんですか?検索してもいまいち分からなかったんですが。>68
70ご冗談でしょう?名無しさん:2001/06/23(土) 06:23 ID:ui3F/05Q
ソレこそ循環流を作って揚力を得る凧>回転凧
20年くらい前からあったよ、なんでwebで紹介されてないんだろう?
ま、しょせんWeb情報なんてソノ程度ってことか
71ご冗談でしょう?名無しさん:2001/06/23(土) 16:57 ID:cfvWm89A
なんで揚力得られるんですかAge。
72ご冗談でしょう?名無しさん:2001/06/23(土) 17:14 ID:???
>>70
実際Webの情報量って目茶苦茶偏ってるよね。
google での検索結果
ガンプラ 16,900件ヒット
地球ゴマ 234件ヒット
そりゃ今どき地球ゴマなんかで遊ぶやつぁいねーだろうけどさ・・
73 :2001/06/23(土) 18:39 ID:cfvWm89A
ここにいます。
742:2001/06/23(土) 18:54 ID:0cBhcfA.
>>14 (=6) さん
あうー。どうもありがとうございます。レスが遅くてごめんさい。
えっと、えっと、こっちの言っていることが変なので話が通じなくて
ごめんなさい。

えっと迎え角が+で、流れ→\翼のとき、揚力が上向きで重力とつり合う
のは、わかりますー。背面でもそうでなくても水平飛行ならこれでいいですー。

で、釣合が壊れたときはどうなるんでそう。
迎え角が−で、流れ→/翼のとき、揚力が下向きになって落下していくとき、
機体を支える力が無くって、<<等加速度>>で落下するのかどうかに関心が
あったのです。

流体が水だったら、速度に比例する抗力が働いて適当につり合う感じが
するんですけど。でもでも、いろんな量に依存してるので、実際は試して
みないとだめですよね....
752:2001/06/23(土) 19:11 ID:0cBhcfA.
隊長! googleからこんなの発見しました〜

http://www.geocities.co.jp/Technopolis/4764/52/530302.htm

10の翼は流れと平行じゃなきゃベルトの分の揚力ができるのデスカ?
762:2001/06/23(土) 19:29 ID:???
>>75

>ベルトの分の揚力が
→ ベルトの分の揚力「も」
7770:2001/06/23(土) 19:59 ID:???
>>75
えらい!コレコレ!S52年NHK教育なるほど記憶が私もよみがえってきた
ありがとう!激しく感謝
78ご冗談でしょう?名無しさん:2001/06/23(土) 23:42 ID:???
>>74 14(=6)
空中で「迎え角が−で、流れ→/翼」なる状態を瞬時に達成で
きるか判らないけど、そのような非定常状態は長く続かず、
機首が下がって垂直降下の後、定常飛行に回復すると思われ。
水中モーターの場合もほぼ同じ(頭から底に衝突)と推察される。

第2弾スレは「揚力の本質」以外のこのような話しを
やっても良いものなのかね?>ALL
79名盤さん:2001/06/24(日) 00:31 ID:G9.AGSQA
なんで揚力得られるのですか?>67
8067:2001/06/24(日) 03:53 ID:w/bJpxeM
>79
ベルトの回転にともなって発生する循環。
その典型例として回転凧があげられているとおもうんだけど。俺も回転凧ははじめて知ったよ。
前スレとか回転凧のリンク先とかよんでみた?それでも疑問があれば
   なにがわからないのか、どうわからないのか・へんなのか
整理してくれないかな?
ここには、いろんなわかりかたをする人がいるんだから、どうこたえていいか、こたえるほうだって
考えちゃうんだよ。
81名無しさん@お腹いっぱい。:2001/06/24(日) 05:26 ID:???
http://ikupara.pro.tok2.com/riron/_berunui.html

じつは揚力は
「ベルヌイの定理で説明するのが都合が良い」だけで、本当は「 ベルヌイの定理だけで計算しても、実際の揚力の10分の1程度にしかならない」そうです。普通の説明では「前縁で分かれた空気が後縁に同時に到達するから」と書かれていますけど、実際には上面の空気のほうが下面の空気よりも早く後縁に到達します。
ダランベールの背理やクッタ・ジュコフスキーの定理をもとに、結局は粘性と慣性で翼の周りの 境界層内の空気が循環することによるマグヌス効果で揚力を得ているそうです。つまりゴルフボールの弾道と同じ原理ってことです。深いですなぁ?
82ご冗談でしょう?名無しさん:2001/06/24(日) 07:57 ID:???
↑あなた、自分でこのページつくったでしょ?
83ご冗談でしょう?名無しさん:2001/06/24(日) 08:36 ID:???
ちゃいますよ。だれか知らん人のページ。
でも2chのスレみてた可能性はあるな。
84ご冗談でしょう?名無しさん:2001/06/24(日) 16:56 ID:???
>>81
>翼の周りの 境界層内の空気が循環する
わけではない。
スレのレベル下がりすぎ。
85前スレの1@中学生です:2001/06/24(日) 17:19 ID:dMLaDGHY
知らないうちにパート2が出来てたんですね。

相変わらず難しくて良くわからないですが
パート2ができてチョットうれしいです。

では引き続きどうぞ。
86ご冗談でしょう?名無しさん:2001/06/24(日) 22:56 ID:HKhvX5nM
じゃあ自分が上げればいいじゃない?>84
87ご冗談でしょう?名無しさん:2001/06/25(月) 11:21 ID:Ywbb403c
age
88ご冗談でしょう?名無しさん:2001/06/25(月) 12:00 ID:B1yrhZns


           ζ
       / ̄ ̄ ̄ ̄ \
      /          \
     /\    ⌒  ⌒  |
     | |    (・)  (・) |
     (6-------◯⌒つ
     |    _||||||||| |
      \ / \_/ /
        \____/
     ______.ノ       (⌒)
    //::::::::|-、 ,-/::::::ノ ~.レ-r┐
   / /:::::::::::|  /:::::ノ__ | .| ト、
   | /:::::::::::::::| 〈 ̄   `-Lλ_レ′
   レ::::::::::::::::::|/::: ̄`ー‐---‐′
89みんな馬鹿だなぁ:2001/06/25(月) 12:03 ID:nkhE8eps
ひこうきっていうやつはなエンジンとかプロペラついてるから飛ぶんだぞ!!
そんなこともわからない厨房共は逝ってしまえ!
90ご冗談でしょう?名無しさん:2001/06/25(月) 13:32 ID:???
>>89
ファン(CPUでも、換気扇でも良い)
の表と裏、どちらが風を作るのに寄与しているでしょうか?
表で負圧を作り、そこに空気が流れ込み、慣性で後ろ側に吹き出す
のか?
裏で、直接空気を押し出し、粘性で表の空気を引き込むのか?
前スレでは、こういった「管欄」議論が盛んだったみたい
91ご冗談でしょう?名無しさん:2001/06/25(月) 16:30 ID:ccXc7ejQ
ではなぜエンジンとプロペラがついていたら飛べるのですか?>89
92ご冗談でしょう?名無しさん:2001/06/25(月) 16:39 ID:nmUQ.ZXs
揚力はエンジンとかプロペラ関係無いでしょ。紙飛行機
・グライダーにだって揚力は発生するんだから。>>89
煽りだから相手しなくていいよ >91

>90
なんかうまい例えに思える。
93ご冗談でしょう?名無しさん:2001/06/25(月) 18:55 ID:???
管欄ってどういう意味?
94ご冗談でしょう?名無しさん:2001/06/25(月) 19:51 ID:???
管欄=kudarann
95名無しさん@お腹いっぱい。:2001/06/25(月) 22:24 ID:???
なんてこったい。
96ご冗談でしょう?名無しさん:2001/06/26(火) 00:35 ID:???
>>90
あのスレを読んでもそんな「管欄」理解しかできないドキュソ
97ご冗談でしょう?名無しさん:2001/06/26(火) 02:34 ID:???
前スレは結局オイラーの方程式(ナヴィエ・ストークスも含む)の
一部に妙に執着して、大切な連続の式(divU=0もしくは
div(m(p)U)=0とか)を無視すぎた嫌いあり。
これらの式、結構重要な働きをしており、流体内部での相互作用に
決定的な影響を与えている。(特に完全流体の場合)
粘性なんて、それに比べれば影響は小さい。
98 :2001/06/26(火) 07:22 ID:dU2nb.76
10の方法でどれくらい揚力得られるの?
静止状態のキャタピラの速度=飛行機の速度の時、両者は同じだけ揚力が得られる?
99ご冗談でしょう?名無しさん:2001/06/26(火) 07:28 ID:???
>>55を見たまへ。
結論として10の方法では揚力は得られない。
100 :2001/06/26(火) 07:54 ID:dU2nb.76
得られるって他の人がいってるのは?
101ご冗談でしょう?名無しさん:2001/06/26(火) 10:33 ID:GEPaoJg.
「ベルト」だか「キャタピラ」だか、それを空気摩擦軽減に利用は
出来るだろうに。だから同じ速度でも、
同じ揚力にはならないだろう。揚抗比は確実に上がるだろうに。
102ご冗談でしょう?名無しさん:2001/06/27(水) 00:12 ID:???
>>97
知ったかぶりの厨房が一撃離脱で恥をさらすのはこのスレの伝統か(藁
連続の式は単なる質量保存側。流体内部の相互作用には無関係。
つーか、相互作用してたら完全流体じゃないし…
低いぞ。
103why:2001/06/27(水) 02:21 ID:yzwddq/c
>>102
ハァ?
方程式の解の範囲に決定的な影響を与えているんじゃないんですかな。
質点系の力学では、質点相互の距離不変に対応する式。
流体を単なる質点系の速度場の方程式に堕落させない非常に重要な
式。
>>相互作用していたら、完全流体じゃないし
藁。お主は、流体と質点系の区別がついておらぬな?
104@@:2001/06/27(水) 02:35 ID:fl0thrN2
物理をかじったけど、
いまでもあんな鉄のかたまりが
そらをとぶわけないとおもってるし、
すべてのモノが原子でできているという
大前提ですら、しんじられない。
105103:2001/06/27(水) 09:45 ID:???
>質点系の力学では、質点相互の距離普遍に対応する式。
剛体の力学を質点系の力学の(一般には)極限とみる場合、
質点相互の作用反作用の法則に加え、相互距離不変を前提に置かなけ
ればならないのと同様。そういう制約条件に対応する式

言葉が足らなかった。変な突っ込みが入るかも知れぬから、一応
訂正。
106:2001/06/27(水) 12:10 ID:5tKzV3zk
>>104
>鉄のかたまり
空を飛んでるのはアルミだ。
アルミが飛ぶのが信じられないなら、そのへんのアルミフォイルで折り紙作って飛ばして見ろ。

>原子でできている
太陽は原子でできてないぞ。
107ご冗談でしょう?名無しさん:2001/06/27(水) 13:24 ID:???
流体と質点系の区別が出来ない人へ

流体ってのは、翼みたいな「障害物」にぶつかる際に得た
「情報」をこれからぶつかる流体部分に伝える性質がある。
つまり、流体は局所の情報を全体に伝えるという、著しい性質
を持つ。この性質は全体を理想化させる働きを持つ。
(障害物との相互作用を出来るだけ小さくするとか)
(密度一定という条件が緩くなればなるほどその性質は落ちるが)

(粘性は、平均化させる働きはあるものの、必ずしも
全体の情報伝達性には寄与しないの*かも*知れない。これは未解決)
だからこそ、流体中の流体である完全流体ではダランベーのパラドック
スとかそういう一見奇妙な現象が時刻無限大の極限では起こり得る。
(あくまでも定常状態の話だからね。)

流体は個々の自由度はかなりあるけど、全体で緊縛されている。
独立した質点集団とは根本的に異なる。

ヒコーキの翼と空気の関係。
完全流体とみなせるほど、レイノズル数に対する速度が小さいわけ
ではなさそうだし、第一、圧縮性があるので、なおさら完全流体
で議論するのはつらくなる。
だからと言って、独立した質点集団の統計作用で見るほど、空気は
非流体的じゃない。
だから、その中間のレベルでの考察が必要なんでしょ。やはり。
108ご冗談でしょう?名無しさん:2001/06/27(水) 14:50 ID:???
前スレでも嵐てたdデモ出現
流体はデムパで「情報」を伝えるのか (ワラ
全体で「緊縛」された流体って、縄師か (ワラワラ
流体粒子の運動に着目した記述はラグランジュ法だが、
定常状態しか考えられないドキュソには理解不能だとオモワレ
109ご冗談でしょう?名無しさん:2001/06/27(水) 18:34 ID:8XXF/wmg
いやアルミも鉄もおんなじようなもんだって。
110ご冗談でしょう?名無しさん:2001/06/27(水) 19:14 ID:BPki7Ib.
>>108
お前も嵐
111ご冗談でしょう?名無しさん:2001/06/27(水) 20:46 ID:nyzN5U6I
定常状態になる為に、障害物の情報を源に伝えて
あらかじめそれに備えてもらうことによって、
調和した流れに近付き定常状態になると思われ
その意味で流体は「繋がっている」(緊縛は大袈裟)
粘性が調和にどれだけ寄与するか?これは大問題
圧縮性については調和を邪魔する性質があることは
容易に予想できるが。
112ご冗談でしょう?名無しさん:2001/06/27(水) 21:41 ID:???
>>107=103=97? & >>108=102?
流体と翼が相互作用をする状況ではラグランジュ法で考えた方が
わかりやすいかもしれないな。流れを「場」でとらえた議論では
流体の圧力にとらわれすぎる傾向がある。>>107もそのようなこと
を書きたかったようだが、あれじゃトンデモだぜ。>>108の気持ち
もわからんでもない...
>>110のようなカキコはクソ。スレを腐らせるだけだ。
113ご冗談でしょう?名無しさん:2001/06/27(水) 21:48 ID:???
>>111
>定常状態になる為に、障害物の情報を源に伝えて
>あらかじめそれに備えてもらうことによって、

どうやってその情報を源まで伝えるんだい?
114111:2001/06/27(水) 23:59 ID:dAznLlLU
>>112
圧力の「波」でしょうか。やはり。(誤解を招く表現かも)
115ご冗談でしょう?名無しさん:2001/06/28(木) 01:36 ID:???
>>114
圧力の「波」って、音波のことかい?

定常状態ってのは、着目している場所で物理量の時間微分がゼロに
なる条件を満たせばそれでオッケー。別に「源」までその情報を伝
える必要はないんだ。
流体の下流側に影響を及ぼすことを「繋がっている」とするなら話は
わからんでもないが、上流側である源に影響すると考えるのは変だぜ。
116114:2001/06/28(木) 02:07 ID:???
別に変ではない。
確かに(D/Dt)u=grad(p)+grad^2(u)という方程式だけだと
因果関係の保存とか成り立ちそうだが、連続の方程式(の類)
が同時に成立するから。

「情報」を伝えるのは圧力とも限らないかも知れない。
(自分で言っていてスマソ)

要するに連続の方程式(の類)から発生する、任意の領域の
密度変化に対する制限を満たすために発生する、流体要素間に
働く力かな?
粘性とは独立したもの。
勿論、圧力もそういう役割を大いに果たすでしょう。
111の話は定常状態に至るまでの話。
敢えて標語的に言えば、「ある一般的な仮定の元で流体は
集団的な学習性を持ち、例えば抗力とか揚力とかの障害物に
対する作用を0に近づけようとする」って予想を言ったつもり。
というかそれを証明することが流体力学の基本的な問題なんじゃ
ないのかな?
完全流体だけでなく、圧縮性や粘性が入ったときにもこの「予想」
が成立するかとか。
粘性は、「自己学習性」に+に働くか、−に働くか?とか。
117ご冗談でしょう?名無しさん:2001/06/28(木) 02:42 ID:???
>>116
連続の方程式が好きだな。
 (∂/∂t)ρ + div(ρv) = 0
 領域内の質量の時間変化は境界を通して出入りする質量に等しい
ってごく当たり前のことしか言っていないんだよ。連続の方程式を
満足させるように、何か特別な力が働いているわけじゃない。

「流体の学習性」とかいったトンデモな考えを改められないのは何故だ?
118116:2001/06/28(木) 14:25 ID:ypcozjrI
>>117
そうとも言い切れんでしょうに。一般に密度ρ が変化する時は、ρ と圧力pは
関数関係で結ばれる。
つまり、pはvによって支配され、同時にvはpによって支配されるという関係
vと圧力pの因果関係を決定づける重要な式だと言える。
119ご冗談でしょう?名無しさん:2001/06/28(木) 18:56 ID:???
age
120ご冗談でしょう?名無しさん:2001/06/28(木) 18:57 ID:lcFEW7P.
age
121ご冗談でしょう?名無しさん:2001/06/29(金) 00:59 ID:???
>>118
そりゃ「関係」はあるよ。古典系であれば因果律は厳密に成り立つぜ。
だからと言って、連続の式を成立させるために「特別な力」が働くと考える
のは間違い。連続の式をチョー簡単に表現すれば「水が流れ込むとコップの
中の水が増える」ってことだが、これのどこに「力」が働くんだ?

流速と圧力(と密度)の因果関係はナビエ‐ストークス方程式で決まる。
ある時刻の流体の速度分布と圧力分布と密度分布と外力がわかれば、その
後の任意の時刻における速度分布を「計算」することができる。解くとき
には連続の式とエネルギーの式を連立する必要がある。

で、流体の「学習性」とかの妄想は消えたかい?
122118:2001/06/29(金) 04:02 ID:???
流体の方程式は、流体要素を質点として解釈して得られる。
(座標の取り方で、オイラー表現・ラグランジュ表現と変わる。)

また(完全流体の場合を除いて)、速度場が、密度分布を作り、
密度分布がその点を通過している流体要素に与える力を決定する
この関係も質点力学の言葉でいえば、結局は質点の全体的な位置関係
(とりわけ距離)で決まる内部相互作用を持つ質点系という風に言い
換えることが出来る。

確かにその意味では、質点の方程式に過ぎないかも知れない。
(非線形の結合関係を持つ無数に多い質点の方程式)

大抵の場合、流体要素に与えられる力は、ポテンシャル力、つまり
圧力ということになり、圧力というスカラー量は、密度の関数と
仮定される。(もしかすると、これが流体と質点系を決定的に区別
する性質なのかも知れない)

完全流体の場合も、初期値境界値問題に対する解は、圧力を密度の関数
と表した時、圧縮性を仮定した方程式(系)で例えば圧力p=密度ρ^mと
した時の解でm->∞をしたものと深い関係があるだろう。
(完全流体とは極端に「圧縮・膨張」に抵抗する性質を持つ流体
では無いか?)

で、その完全流体で、領域中に障害物がある場合でも、その障害物と
相互作用が無い解に、多くの初期値に対する解が収束していくだろう
という「予想」が、ダランベールのパラドックスの解釈だろう。

これがもし正しいなら、流体は、「密度分布(圧力分布・速度分布)
等の自分自身を支配するモノを、自分自身でオプティマイズする
「特別な力」」を持つ系であるともいえる。少なくともこの性質は、
独立した(相互作用の無い)質点系では見られないい。

特別な力と書いたが、これは通常の次元を持ったものでは無く、プロ
パティに近い。
そういった「力」が本当にあるかを調べることは、流体力学のひとつの
遠大な目標じゃないのかな?
確かに、通常の意味での「力」ではなく、あくまでも予想・想像だから
それを妄想だと思うのは自由。信仰かもね。
123ご冗談でしょう?名無しさん:2001/06/29(金) 16:16 ID:daDJHFv.
液体は表面積が最小の形となるよう自分自身をオプティマイズする
「特別な力」を持つ系?
124ご冗談でしょう?名無しさん:2001/06/30(土) 06:20 ID:L8bdmdZ2
age
125 :2001/06/30(土) 19:29 ID:el4kRm5c
age
126ご冗談でしょう?名無しさん:2001/07/01(日) 01:10 ID:1cJCcX6o
前スレで争論となった問題を再掲

争論:揚力とは

・翼の下の空気が、翼の真上の空気が圧力低下することによって「軽く」
なって、その上の荷重をまともに翼にかけない為に発生する。
つまり、動的に発生する浮力である。
なお、荷重という言葉が出たが、この力は翼が静止空気内を鉛直に移動
する場合にも、進行方向に対し垂直に発生する。(この場合も片側の
空気が「軽く」なること(圧力の等方性によって発生する一種の浮力
と考えられる。)空気自身が得る運動量は、非常に広範に分散し、
一部は、地上に圧力として作用する為、観測される値は、飛行機が
重力によって得る筈の運動量よりもかなり小さい。

これに対し、

・翼上面で翼と空気が相互に引力を及ぼす為に発生する。この時、
翼上面の空気は翼に引き寄せられ、下向きの運動量を得る。
その大きさは毎秒当たりmgだけ増加する。ここでmは飛行機の質量、
gは重力加速度。
引力の原因は、空気と翼の間の粘性である。

どちらかが正しいのでしょうか?それとも、同じことの言い換えなの
でしょうか?
127ご冗談でしょう?名無しさん:2001/07/01(日) 02:18 ID:???
>>126 まとめて話しを本筋に戻そうとする努力に
は敬意を表するが本当にそうかい? 俺の理解では、

”主たる”揚力の発生理由は、

・翼により空気の流れを下向きに曲げて、その
反作用として生じる (旧スレ 27説)

・翼により翼上面に負圧を発生させ、その負圧
 方向に引き込む力として生じる (ベル派)

という2説の戦いだと見ているんだが。粘性だと
か完全流体だとかいう言葉は、上記コンセプトを
より詳細に説明、検証するために使われてるだけ。

俺自身は10数年前に揚力の発生要因として27説
を説明してもらったが、それ以来27説を支持して
いる。前スレでは結局27説が優勢であったように、
思えるが如何?
128ご冗談でしょう?名無しさん:2001/07/01(日) 08:20 ID:???
>>127
そもそも「圧力差」では揚力を説明したことにはならないのでは?
 揚力→圧力分布でわかる
 圧力分布→速度分布でわかる
 速度分布→ナビエ‐ストークス方程式を解かないとわからない
とゆーことで、揚力の計算はできるが理解にはなっていない。で、
無理に速度分布の生じる理由をひねり出すと、「翼の上下の長さの
違い」になるんじゃないのかな?
ナビエ‐ストークス方程式の解を境界条件まで含めて物理的に
説明するのは難しいと思う。
ギョーカイの人の意見キボン。
129ご冗談でしょう?名無しさん:2001/07/01(日) 09:54 ID:???
127だけど、コンセプトの話をしてるだけだから
あまり難しく考えないでおくれよ。言葉足らず
だったかな。

・翼(の断面形状、迎え角の形成)により、翼
 上面の空気流れを下面に比べ相対的に速くする
 事により翼上面に負圧を発生させ、その負圧
 方向に引き込む力として生じる (ベル派)

これで良い?
ちなみに翼上面の長さが長いことは直接は関係な
いからね。上面平ら下面凸の翼でも迎え角を取れ
ば揚力は発生するから。

ちなみに私は業界人でもないし流体力学を学んだ
事も(ほとんど)無い。
130ご冗談でしょう?名無しさん:2001/07/01(日) 15:58 ID:???
>>129
そう、だから「ベル派」ではちゃんと理解できないってこと。

>・翼(の断面形状、迎え角の形成)により、翼
> 上面の空気流れを下面に比べ相対的に速くする

相対的に速くなるのは、結局ナビエ‐ストークス方程式
(その近似式も含む)を解いてみないとわからないってこと
でしょ。
「逆立ちゴマは何故逆立ちするのか?」って質問に対して
「運動方程式を解けばわかる」って答えているのと同じ。
計算はできるけど説明になっていない。
131ご冗談でしょう?名無しさん:2001/07/01(日) 17:09 ID:X34M3azI
でも、翼上面の速度が下面の速度の倍以上になるって話しは
実験事実のようでっせ。飛行機の教科書数冊開いてみれば、
アメリカかなんかの研究所のデータとして掲載されている。
翼によって、「仕事」されているし、粘性も考慮せんといかんから
安直にベルヌーイの等式を使うことは許されないことは、認めるが
ベルヌーイの等式の右辺(エネルギー密度の次元だっけ?)、
翼の上と、それに到達する前で極端に変わるのだろうか?
余り変わらないとすると、速度が倍になった時、圧力低下が
起こるということは、たとえ厳密でないにせよ、十分な説明に
なると思うのだが。(何故倍になるかは、形状の問題だから、
ここでは取り敢えず不問とする)
132ご冗談でしょう?名無しさん:2001/07/01(日) 17:14 ID:???
>>130
127(=129)だけど、フォローしていただいているのか
反論していただいているのか良くわからないが、俺の
考えでは、ナビエ・ストークスの式なんてどうでも良
いんだよ、所詮人間が考えたものだから。

自然現象として、上面の空気流れが速くなるんだから
それはそれで「ベル派」への否定材料にはならない。

もう一度書くけど、自然現象として考えた場合、"主
たる”揚力が

・翼により空気の流れが下向きに曲がり、その
 反作用として生じる (旧スレ 27説)

のか

・翼(の断面形状、迎え角の形成)により、翼
 上面の空気流れが下面に比べ相対的に速くなる
 ため翼上面に負圧が発生し、その負圧方向に
 引き込まれる力として生じる (ベル派)

のかどっちなのかという論争だと思うけど。
133ご冗談でしょう?名無しさん:2001/07/01(日) 17:25 ID:???
127だけど、つっこまれる前に断っておくけど、
>>132で書いた2つのコンセプトの論争であるという
点はご理解いただけると思うけど、これらを定量的に
検証するために人間が考えた流体力学の式を用いる
って事じゃないの。

私は実際の計算等詳しくないので、詳しい方どうぞ
お続け下さい。
134ご冗談でしょう?名無しさん:2001/07/01(日) 18:05 ID:???
前スレでも触れられていたが、いずれにせよ、翼通過後
空気の運動量が(トータルで)下向きの成分を得ることは
間違い無さそう。しかし問題はその量と分布。
翼上面付近に集中しているのか、それとも、周囲の空気全体に
広く僅かに分散的に(最終的には地面でブロックされるか)
なるのかってこと。
翼と空気の間の相互作用で引力発生というのは、どうも眉唾
っぽいから(他にも同様の原理で説明できる物が多数ある筈)
上面の下降気流とやらも、圧力差関連じゃないかと思うが。
135ご冗談でしょう?名無しさん:2001/07/01(日) 19:04 ID:???
>>131
だから、ベルヌーイの定理はそれで良い。けど揚力の説明をする
ときに「速度差が生じているから」では説明になっていない。
「ベル派」は速度差が生じる理由をちゃんと示さないと説明して
いることにはならない。
前スレでは「ナビエ‐ストークス方程式の解だから」といった
ものしかなかったよ。

>>132
「27説」は物理的な説明が完結している。
 揚力→空気の運動量変化で説明
 運動量変化→翼の形状と空気の粘性で説明
物理の素養に乏しい連中がこれを理解しないだけ。

「ベル派」は圧力差が生じる理由を「流体力学」に求めていた
のではダメ。って、そーしたら「ベル」じゃなくなるけどね。
流体分子間の相互作用(って粘性だ)に着目すれば、翼の上側の
圧力が下がることは説明できそうだが、だれもそーゆーアプローチ
はしてない。

>>134
微分方程式が解けなくなるとゆー理由で「粘性はワルモノ」と
たたき込まれた連中が、揚力の理解のジャマしてる。
前スレからの流れだな。
136ご冗談でしょう?名無しさん:2001/07/01(日) 23:06 ID:oSJ0jwBg
誰も粘性がワルモノなんていっていないようだ。
要するに粘性=分子間相互作用であり、完全流体=分子間相互作用無し
と心から信じちゃっているあなたが少しヘンなのよ。
分子間に相互作用が無い系は流体は流体でも、幾らでも圧縮可能な
圧力が無いヘンな系。普通そういう奴は単独の粒子に分けて扱う。
137ご冗談でしょう?名無しさん:2001/07/01(日) 23:07 ID:oSJ0jwBg
ゴメソ
分子間相互作用=粘性
ね。
138ご冗談でしょう?名無しさん:2001/07/01(日) 23:09 ID:oSJ0jwBg
もう一つ
幾ら圧縮されても、圧力が上がらない、ヘンな系
に訂正
139ご冗談でしょう?名無しさん:2001/07/02(月) 00:41 ID:???
ナビエ‐ストークス方程式を経由しないと流体を把握できない
マヌケが、統計力学も知らずに講釈たれてるぜ (藁

分子運動論の立場では理想気体(=完全気体)は
 構成分子間に相互作用の働かない気体
と定義されている。
「壁」に衝突すれば運動量は変化して、ちゃんと「圧力」を発生させる。

レベル低すぎだな...
140ご冗談でしょう?名無しさん:2001/07/02(月) 03:09 ID:8W7E9/kc
だからぁ〜
完全流体≠理想気体=本質的に圧縮性の(非粘性?)流体
恐らく粘性を仮定しても仮定しなくとも理想気体の定義には
矛盾しないでしょうね。(証明は知らんけど。)
理想気体を議論するときは、圧縮性を考慮した流体力学で
考えなければ。質点が独立に運動できるわけじゃないからね。
自由度は極めて高いけど。
141ぜっと:2001/07/02(月) 03:55 ID:Gx.cRr9.
完全気体は非粘性です。理想気体は熱力学もしくは統計力学で定義され
ているものなので、別に粘性とかは関係ないです。でも、飛行機とかの
高速気流はほとんど、完全気体で、理想気体を仮定しているね。つまり
非粘性の理想気体。いちばん簡単な期待です。
142140:2001/07/02(月) 04:39 ID:???
理想気体という言葉が良くないかも知れない。
熱力学的標準気体とかでも言うのが良いのかな?
143ご冗談でしょう?名無しさん:2001/07/02(月) 09:48 ID:???
>>140
「相対論は間違っている。なぜなら、速く走っても体が重くなる
 感じがしないから」って言ってるガキと同レベルだな。

言葉の定義をちゃんと書いておくと、
 理想流体=完全流体
 理想気体=完全気体
ってゆーことになってるが、知らねーのかよ。
「粘性のある理想気体」ってのは矛盾もハナハダしいぜ。
流体力学の前に統計力学を勉強しな。
144ご冗談でしょう?名無しさん:2001/07/02(月) 12:49 ID:???
誰も相対論は間違ってるとか書いて無いのに。....
すぐ直接関係の無い話を引出す奴は藁藁)
理想気体=完全流体とするんだったらそう思っておけば良い。
その変わり、理想気体を圧縮しようと思ってはならないが。
145ご冗談でしょう?名無しさん:2001/07/02(月) 12:56 ID:aL0vuGs2
>>144
じゃ何故わざわざ別の言葉で定義しているのですか?
統一するのが、物理の世界じゃ100年程前からトレンデーなんでしょw)
146ご冗談でしょう?名無しさん:2001/07/02(月) 13:13 ID:???
>>144
つまり「理想気体には粘性がある」って主張ね。ドキュソ決定だぜ。

流体を流れの場で捉えるオイラー法をちゃんと理解してないから、
完全気体でも「構成分子は互いに*圧力*を及ぼしあっている」と
思い込んでしまっている、可哀想なヤツ。

物理の基礎がちゃんとしてないところに流体力学を詰め込むと、
こーゆー惨めな結果になってしまうんだろーな。
147ご冗談でしょう?名無しさん:2001/07/02(月) 17:40 ID:GQs.cbtE
理想気体が圧縮可能という主張がなぜ粘性の存在に直接結びつく
のかにゃ?
148ご冗談でしょう?名無しさん:2001/07/02(月) 18:14 ID:???
>>145
前スレで、理想気体は粘性流体じゃと威勢よく主張していた
ドキュソさんも、似た文体だったな(稾)
宗祖替えでっか?
149ご冗談でしょう?名無しさん:2001/07/02(月) 19:37 ID:BiqRGUJg
>宗祖替え
宗主替えの間違いでしょ 爆)
150ご冗談でしょう?名無しさん:2001/07/02(月) 22:36 ID:???
>>147
これは>>140あたりのカキコで
>理想気体を議論するときは、圧縮性を考慮した流体力学で
>考えなければ。質点が独立に運動できるわけじゃないからね。
とマヌケなことを言ってるからだな。
「理想気体の質点(気体分子のことだろ)が独立に運動できない」
って、相互作用をしているってことなんだぜ。つまり粘性を持って
いるとゆー主張。
まっ、当人の頭の中では、ナビエ‐ストークス方程式が気体分子の
運動をツカサどっていると思い込んでいるようだから、オレの書い
ていることは意味不明なんだろーな、きっと。
151ご冗談でしょう?名無しさん:2001/07/03(火) 00:14 ID:CqauYUUw
東京工業大学掲示板
http://green.jbbs.net/study/398/toko.html
152ご冗談でしょう?名無しさん:2001/07/03(火) 02:52 ID:G0shLFho
統計力学の分子レベルの話と、分子速度の平均を流体要素の速度と
して扱う流体力学では話のレベルが違って当然じゃないの?
理想気体の流れを考えるときは、それとPV=nrtという公式が
縁深いことからわかるように、圧縮性を考えなければならない
でしょう。本筋では。
繰り返すが、流体力学での質点(流体要素)は統計力学の分子とは
スケール的に言って全然違う。一段高い統計量といったところか。
流体要素はその密度を勝手に取ることが出来ないので、全体から
束縛された運動をすることになる筈。
統計力学と流体力学の区別をつけてね。
153ご冗談でしょう?名無しさん:2001/07/03(火) 03:16 ID:???
>>152
そんな話じゃないだろ。
「理想気体に粘性があってもよい」などとゆー、
マヌケな考えを正してやったんだぜ。わかったか?
154141:2001/07/03(火) 05:31 ID:20YYMoJE
粘性があるってのは、ただ単に流体にせん断応力が働いているといっているだけです。それは流体力学ではじめてつかわれる言葉の定義であります。一方理想気体というものは、熱力学もしくは統計力学で定義されたものであって、粘性がどうのこうのということとは全く無関係です。つまり、理想気体に粘性があっても別になんの問題も生じません。そもそも粘性があるないは流体の運動状態によって左右されものなので、気体自身の性質をあらわしている’理想気体’というものとはなんの結びつきもありません。例えば、実際に粘性流体を扱うときは、まず、流体の応力がひずみ速度テンソルに一次まで比例すると仮定して、ナビエストークスの方程式を用いる場合が多いです。さらにこのとき、理想気体の状態方程式も仮定することによって、温度場について議論をしている研究もあるでしょう。
155ご冗談でしょう?名無しさん:2001/07/03(火) 10:20 ID:???
>>154=141
「理想気体(=完全気体)に粘性があってもよい」派の二人目か。
他にもいるの? 粘性が生じる理由を考えたことはあるのか?

「翼模型の前に飛行機スレのプリントを貼った研究室」の関係者に
聞きたいのだけれど、流体力学の講義では粘性はどのように教えて
いるのでしょう? ナビエ−ストークスの粘性項として導入するだ
けで、そのオリジンについてはナイショなの?
「ヘビ?」はオレ。
156ハシボソミナズキドリ:2001/07/03(火) 12:05 ID:???
昨日NHKでハシボソミナズキドリの特集がやっていました。

オーストラリアのタスマニアから北海道はオホーツクまで、20日間で
食べ物を食べずに飛行するという鳥です。

この飛行方法の解説がされていました。
飛行は主に滑空で、エネルギー消費を抑えて効率良く飛んでいます。
上空は風が速く、低空は波の摩擦により風が遅く、その速い風と遅い風
の差を利用しているというのです。

それって、どういうこと?
速い風と遅い風があると、効率が良くなるの?
157:2001/07/03(火) 12:06 ID:???
「ハシボソミズナギドリ」でした
158ご冗談でしょう?名無しさん:2001/07/03(火) 12:40 ID:???
前スレの840くらいを参考にしておくんなまし。
いわゆるダイナミックソアリングだと思うよ。

(前スレ含め過去ログに目を通してる?スレに何か
書きこむときは一応これを踏まえるのがエチケット)
159ご冗談でしょう?名無しさん:2001/07/03(火) 13:16 ID:???
>>158
ハァ? ここにその過去ログのリンク先を書いてくれ。

それに話が違うぞ。
160ご冗談でしょう?名無しさん:2001/07/03(火) 13:17 ID:???
158だけど、前スレの書き方じゃちょっと
わかりにくいかも知れないんで、東先生のわ
かりやすい文章を読んでくだされ。
(荒れた場のちょっとした息抜き)
http://www.japa.or.jp/kouen/soukai1.pdf

もうちょっと難しい話しはこれ。
http://homepage.mac.com/koichi_takasaki/lib/6thSSS.pdf
161ご冗談でしょう?名無しさん:2001/07/03(火) 13:21 ID:???
>>159
158だけど、前スレのリンクは通常は1番にあるもの。
このスレでも同様。(まだhtml化されてないけど)

話しが違うかな?俺はそう思わないけど、君の
説明は何なの?マターリと行きましょうや。
162ご冗談でしょう?名無しさん:2001/07/03(火) 14:39 ID:???
>>160
この説明、大変わかりやすいです。
ありがとうございます。
163163:2001/07/03(火) 14:46 ID:zBn0KAQ6
>155
分子運動論で、平均速度差が異なる、分子集団が明確に2つ以上の
領域に分かれている場合、その境界で平均速度差を小さくする現象
も説明出来るんじゃないの?理想気体でも。それからある程度粘性
を説明出来るような気がするが。
164ご冗談でしょう?名無しさん:2001/07/03(火) 17:32 ID:0lIoca6w
MSNにこんな記事が載ってました。
http://journal.msn.co.jp/worldreport.asp?id=010703s_suzuki&vf=1
でも肝心の所はおあずけになってます。
165ご冗談でしょう?名無しさん:2001/07/03(火) 18:25 ID:???
>>163
>分子運動論で、平均速度差が異なる、分子集団が明確に2つ以上の
>領域に分かれている場合、その境界で平均速度差を小さくする現象

理想気体では分子間に力が働かないので、速度差は小さくならない。
両方が合わさった速度分布になるだけ。

>>164
「揚力の説明は難しい。やはりナビエ−ストークス方程式を解かな
 いと正確にはわからない」ってオチだけはやめて欲しい。
166ご冗談でしょう?名無しさん:2001/07/03(火) 18:55 ID:???
理想気体では分子同士が衝突してもすり抜ける。
か....。
んな仮定があったかな?
167 :2001/07/03(火) 19:20 ID:???
ぶわっふわっふわ!
断面が上下対象でも揚力は発生する。
単なる板でも揚力は発生する。
168ご冗談でしょう?名無しさん:2001/07/03(火) 20:05 ID:???
>>166
「分子間に全く相互作用が無い」のだから、分子同士は衝突もしない。
「壁」とは衝突して運動量が変化する。つまり圧力が生じる。
これでもまだ「理想気体に粘性があってもよい」と思うのかよ。

そろそろ、翼周りに揚力を発生させるような圧力分布が生じる
理由を考えようぜ。
169ご冗談でしょう?名無しさん:2001/07/03(火) 20:18 ID:fy7YKk/o
>両方が合わさった速度分布になるだけ。
これは平均速度が変化することを意味し、両方の領域で
応力が発生したことを意味し、粘性が発生したことになるのでは?
170ご冗談でしょう?名無しさん:2001/07/03(火) 20:28 ID:???
>>169
ならないよ。
「理想気体に粘性がある」ってのは間違いだから、
もうこのスレに書くのは止めな。
171  :2001/07/03(火) 20:34 ID:???
164のMSNの記事は前スレででてきた記事とは違うものらしい。
http://journal.msn.co.jp/worldreport.asp?id=010703s_suzuki&vf=1
172  :2001/07/04(水) 02:02 ID:???
あげ
173ご冗談でしょう?名無しさん:2001/07/04(水) 02:53 ID:NdCTma1M
>>170
圧縮性はじゃあるのね?
174ご冗談でしょう?名無しさん:2001/07/04(水) 10:06 ID:???
>>173
あるよ。
理想気体って PV=nRT が厳密に成り立つ系だぜ。
175 :2001/07/04(水) 14:13 ID:???
揚力の発生原因は二つ
仰角による主成分と断面の形状による副成分。

仰角による成分は翼の上面と下面の空気の密度差による
形状による成分は速度差による。
じゃない?
176ご冗談でしょう?名無しさん:2001/07/04(水) 15:36 ID:9HLideLQ
2つの理想気体が同じ温度(平均運動エネルギーが同じ)で、平均速度だけ差を持っている状態で接しているとする。更に
両者とも速度分布が等方的であるとする。(分子レベルでは、ある特定の方向に
流れているわけでは無く、ランダムな方向に運動している)
この時、時間の経過と共に、両者の分子の平均速度が変化することは、分子同士
の衝突を仮定しない場合でも、容易にわかると思うのだが。
だから、分子運動論と温度・圧力・マクロな運動としての流体運動を結びつけて
考える場合は、粘性に相当する性質は、割合と簡単に出てくる。
177ご冗談でしょう?名無しさん:2001/07/04(水) 16:02 ID:???
>>176
かなり逝ってるドキュソ

「平均運動エネルギーが同じで、平均速度だけが違う等方的な速度分布」
ってモノを考えられちゃうところは、やはり真性のキティーちゃん?
178ご冗談でしょう?名無しさん:2001/07/04(水) 19:58 ID:6.OcmK.E
平均自由工程という言葉の意味わかっていますか? >177
179ご冗談でしょう?名無しさん:2001/07/04(水) 20:08 ID:???
分子の質量が違うと176の「平均エネ…云々」はクリア。(意味無いけど)
理想でも衝突はするだろ。
その際弾性で、混ぜてもポテンシャルには影響なけりゃいいんだから。
というわけで、粘性の話は打ち切り。
180 :2001/07/04(水) 20:34 ID:???
板状の翼モデルで考えていいんですか?

弾性衝突説。
空気分子が翼にぶつかって下向きの加速を得る。
翼は上向きの力を得る。
つうのはダメ?
181ご冗談でしょう?名無しさん:2001/07/04(水) 20:50 ID:???
前向きの速度があればいいんじゃないの。
あとは吸盤みたいなイメージで。
182ご冗談でしょう?名無しさん:2001/07/04(水) 21:35 ID:???
>>178
理想気体の分子(原子)は質点とみなすのだが、質点の場合の
平均自由行程はいくらになるんだい?

>>179
「等方的な速度分布」で平均速度が有限になるケースってあるのか?
粘性の話を打ち切るのには賛成。

>>180
オッケーだが、そんな消防な議論をしているスレではない。
時々ドキュソのノイズが入るのは無視してくれ。

>>181
「吸盤」って??? 意味不明だぜ。
183179:2001/07/04(水) 22:34 ID:???
有限ってどういう意味?
等方的なら、速度ベクトルの平均は0で、運動エネルギーは
温度比例(有限)でいいんじゃないの?
理想気体って質点だっけ?
圧縮した極限での体積要素をゼロにみなすために質点モデルを取ること
に意味はあるが、別にそれが互いに衝突できても問題無い。
もちろん、出来なくても問題無い。
184ご冗談でしょう?名無しさん:2001/07/05(木) 01:58 ID:iOXU3nSE
δ関数みたいな近距離のみに作用する相互作用で、しかも斥力ならば
相互に区別がつかない分子の衝突は、あたかも「すりぬけ」が起こったか
のように解釈される筈。
(古典)統計力学では、「分子の流れ」みたいなものは、考えないのが普通。
つまり、粘性うんぬんの話と、相互作用の形はまた別な話。
仮に相互作用が引力を含む場合は、気体状態方程式とのズレが発生する大きな
要因となる筈。粘性には影響するが、あくまでも粘性は、流れというマクロな
量で問題になってくる話であり、本質的な要因ではない。
分子の速度分布のズレが流れの本質。
粘性は、その相対的なズレが拡散する現象であり、理想気体で発生してもなんら
問題は無い。
185 :2001/07/05(木) 02:05 ID:???
速度は下面の方が速いとか
186 :2001/07/05(木) 02:16 ID:???
下面にだけ風送っても揚力発生するし。
187:2001/07/05(木) 03:51 ID:???
>>186迎角がついている場合だよね?
それは揚力というより浮揚力…?^^;
188ご冗談でしょう?名無しさん:2001/07/05(木) 04:24 ID:???
>>183
>等方的なら、速度ベクトルの平均は0で、
そう、これが普通。でもキティーちゃんはその平均速度に差がある
としているんだ。つまり有限の値を持つってことだ。変だろ。
理想気体の構成粒子が単原子の場合、有限の大きさがあると回転の
自由度が出てきてしまうので、質点とみなすのが普通。大きさが無
いのだから衝突も考えない。

>>184
>粘性は、その相対的なズレが拡散する現象であり
これが間違い。
粘性があるとエネルギーが散逸するのだが、これは単なる拡散
とは違う。何度も書くが理想気体に粘性は無い。
「理想気体の粘性」の話を続けたいなら、別スレでやってくれ。
歓迎はされないけどな。オレみたいに優しく相手してくれないぜ。
189ご冗談でしょう?名無しさん:2001/07/05(木) 04:41 ID:???
>>187
立派な揚力だぜ。
気流の向きを下向きに変えることで上向きの力を受けている。

で、ホントに知りたいのは翼の上面で下降気流が発生する理由だろ。
MSNの記事の「その2」が早く見たいよな。
190ご冗談でしょう?名無しさん:2001/07/05(木) 06:02 ID:mrbP.AO2
>等方的なら、速度ベクトルの平均は0で
そういう風に見える座標に対して速度vで運動する座標からみれば
速度ベクトルの平均はv(或いはーv)になるでしょうに。
191ご冗談でしょう?名無しさん:2001/07/05(木) 10:23 ID:???
>>190
それ、等方的じゃない気が・・・。
192 :2001/07/05(木) 11:27 ID:4aaLh0uE
揚力は板でも発生します。
飛行機は仰角ありの単なる板でも飛びます
仰角無しの形状だけでは飛びません。

揚力は仰角がメインです。揚力は仰角がメインです。揚力は仰角がメインです。
193 :2001/07/05(木) 11:35 ID:4aaLh0uE
板状の翼の例。

コンコルド、F104,F14,F15,F16,というか現用の超音速機全部。
194 :2001/07/05(木) 11:47 ID:4aaLh0uE
三角翼全部。
195 :2001/07/05(木) 11:59 ID:4aaLh0uE
背面飛行。
196ご冗談でしょう?名無しさん:2001/07/05(木) 16:05 ID:mrbP.AO2
>>191
等方的というのは、ある特別な慣性系から見た場合という意味
でしょ。
197 :2001/07/05(木) 16:17 ID:Dhzy6e56
流体理論派の失敗は
初期条件として翼断面に沿った流束を考え、その作用を考えてる点
にあると思う。

初期条件を何も障害物の無い空間として、そこに翼断面を置いた
場合の流束が翼に与える作用を考えるのが正解ではないか?

気流の向きが変わるのは翼断面が邪魔だから変わるのであって、
断面積が一定の曲がったホースの中の流体が何故曲がって進むのか
を流体力学で解明しようとしてるようなものだと思う。
198ご冗談でしょう?名無しさん:2001/07/05(木) 17:38 ID:rlGpJm2s
それもあるが、境界条件が単純過ぎるのが根本的原因じゃ
ないかな?
199ご冗談でしょう?名無しさん:2001/07/05(木) 20:01 ID:???
>>196
普通それを等方的とは呼ばないな・・・。
200ご冗談でしょう?名無しさん:2001/07/05(木) 21:08 ID:???
>>197-198
なかなか良さげな着眼点だな。
要するに、流れの中に置いた翼から流体は力を受けるはずなのに、
直接その力を考慮していないのがいけない、ということかな。
確かにナビエ−ストークス方程式の外力項でも取り入れてなさそう。
境界では単に速度の接線成分をゼロ(翼と同じ速度)にするだけか?
201ご冗談でしょう?名無しさん:2001/07/05(木) 21:12 ID:cWNGoXuE
188はそれでも、流体を勉強しているのかあ??等方的なら、速度ベクトルの平均はゼロだ〜あ?理想気体に粘性はないだ〜あ?さらには理想気体の単原子分子の衝突はないとは・・・??でたらめなことを書くな!かなり自分に自信をもって発言しているみたいだが、もっとちゃんと勉強をしなおしな!速度分布が等方的でも、平均の速度ベクトルはちゃんとあるぞ。マックスウェル分布を考えれば簡単にわかるだろ!そして、平均の速度ベクトルが巨視的にみたときの流体の速度だろーが!それに理想気体に対しても粘性を考えてもいいぞ!176さんとか184さんの考えが正しいんだぜ!100%正しいぜ。ここにくる人に間違ったことを教えるな。最後に理想気体の分子同士にも衝突は考えるぞ。184さんが書いてあるように理想気体の分子間力はδ関数みたいなものを扱っているんだよ!
188に忠告しとく!別に考え間違えをするのは、人間誰しもあるから許そう。しかし、それを指摘されているのに正そうとしないのは許せない!もっとしっかりと勉学に励みな!いいかげんなことを書くなよ。自分の言動に責任をもて!
202ご冗談でしょう?名無しさん:2001/07/05(木) 21:18 ID:???
>>201
ゼロにならんのは絶対値の平均だろ…
等方な速度分布のベクトルの平均取ったら
ゼロになるに決まってるじゃないか。
203横レス:2001/07/05(木) 21:18 ID:Dhzy6e56
>>199
そんなこと言ったら等方的なんて存在しなくなる。
絶対静止系を主張するつもりですか?
204ご冗談でしょう?名無しさん:2001/07/05(木) 21:19 ID:???
>>199
じゃあ世の中には等方的なものなど存在しないことになるねぇ。
205ご冗談でしょう?名無しさん:2001/07/05(木) 22:16 ID:oLC7SNTE
等方的の意味を誤解しているのか??そもそも等方的という概念は考えている座標系の原点による。もし、速度分布が原点を速度ゼロにとったときの座標系について等方的な分布ならば、202の言うとおり、速度ベクトルの平均はゼロになる。
しかし、ここで使っている等方的な速度分布とは、必ずしも原点を速度ゼロにとったものではないので、平均の速度ベクトルはゼロとはならないのだよ。マックスウェルの速度分布をみれば明らかだ。
206ご冗談でしょう?名無しさん:2001/07/05(木) 23:04 ID:???
>>205
じゃあ最低限その事実がわかる分布関数を示してみて。
最も簡単なものでいいよ。「等方的」で
「ベクトルの平均がゼロでない(つまり、偏りがある)」ような分布。
207ご冗談でしょう?名無しさん:2001/07/05(木) 23:25 ID:VkgEIudk
(v1,v2,v3)を速度座標としたときのマックスウェルの速度分布
f(v1,v2,v3)=A*exp{-(v1-c1)**2-(v2-c2)**2-(v3-c3)**2}
は原点を(c1,c2,c3)にしたときの座標系について速度の分布が等方的だよ。
この時の平均速度ベクトルは(c1,c2,c3)となり、ゼロでない。
208ご冗談でしょう?名無しさん:2001/07/05(木) 23:56 ID:???
今話してるのは当然一様等方だろ?
209ご冗談でしょう?名無しさん:2001/07/05(木) 23:57 ID:???
キティーちゃんが群れているのか、自作自演か?
流体力学も統計力学もダメ。得意分野は何だ?

>>201
速さと速度の区別のつかないドキュソ
「理想気体の分子間力」などと書いた時点で、逝ってよし。

>>205
ガリレイの相対性原理を知らないドキュソ
マクスウェル分布もちゃんと知らないとオモワレ

>>207
速度と座標を引き算してるドキュソ
そんな式はマクスウェル分布ではない

そもそもマクスウェル分布は、一つの粒子がある速度を持つ確率を
表したもの。統計力学の初歩の初歩なんだがな。
210ご冗談でしょう?名無しさん:2001/07/06(金) 00:15 ID:???
>>207=205?
原点の速度が(c1,c2,c3)か。

>この時の平均速度ベクトルは(c1,c2,c3)となり、ゼロでない。

つまり、(c1,c2,c3)方向に偏った「等方的でない」分布だな。
211 :2001/07/06(金) 00:35 ID:IqzhqPbk
さあ、粘性の話は打ちきって前に進もう。
212ご冗談でしょう?名無しさん:2001/07/06(金) 02:13 ID:lEeUQMbY
209=210
君はなぜ、静止流体しか扱おうとしないのかね??
よくわからん。上の例はちゃんと等方的じゃないのか?

確かに一つの粒子がある速度を持つ確率が統計力学ではマックスウェル分布として扱われている。
しかし、その確率に空間にある全粒子数Nをかけてみな。そううすれば、ちゃんと粒子分布になるだろ??
確率の二項分布の定理より、容易に証明できるぜ。気体を分子の集団で扱うときの基本的なものなのだが。
これもマックスウェル分布というのさ。
それに、そもそも確率ってどう測るんだ??ある速度を持つ粒子数nに全粒子数で割って、p=n/Nとするのが普通でしょ?だから、n=pNとなるのが自然だろ?

ガリレイの相対性原理なんてものとは何の関係も無い。
なぜ、相対性という言葉がでるのか理解できん。
速度をただの座標と考えただけだ。
運動論を述べたつもりはさらさらない。

君は理想気体を絶対に誤解している。理想気体の分子間にも分子間力は働いている。ひょっとして、統計力学で誤解しているのでは??確かに、統計力学では、理想気体をN個の自由分子、つまり分子間力が働いているないものとして扱っているが。しかし、それは分子間に分子間力が働いていないと言う事をいっているのではない。それは、気体が平衡状態にあるときは考えなくてもいいが、非平衡のときは考えないといけないのだよ。これも、ちゃんと統計力学の分子運動のところでちゃんと議論されているのだが・・。見落としているのか??
213ご冗談でしょう?名無しさん:2001/07/06(金) 02:51 ID:???
209=210は自分で自分のことをドキュソと呼んでいるように思われ
214ご冗談でしょう?名無しさん:2001/07/06(金) 06:16 ID:???
座標系を回転させるとどうなんるんですか?
等方なら結果は変化しないと思いますが。

>f(v1,v2,v3)=A*exp{-(v1-c1)**2-(v2-c2)**2-(v3-c3)**2}
>は原点を(c1,c2,c3)にしたときの座標系について速度の分布が等方的だよ。
原点て何の原点ですか?
215ご冗談でしょう?名無しさん:2001/07/06(金) 07:55 ID:???
>>212=dデモ

等方的な速度分布を持った流体が静止流体。流れ出したら速度分布は
等方的ではない。

マクスウェル分布に粒子数をかけても粒子分布にはならない。
「粒子がある速度をとる確率」なのだから、全ての可能な速度の
場合を足し合わせる(全速度空間で積分する)とその確率は1。

速度vで動く座標系から「静止流体」を観測すると、平均速度が
−vで流れているように見える。これは静止した座標系から平均
速度−vで流れる流体を観測した場合と同等。
ってのがガリレイの相対性原理。

>速度をただの座標と考えただけだ。
こんなことを書いている。やっぱりキティーちゃんだな。

「分子間力のある理想気体」について述べてある文献を示して
みろよ。キティーちゃんの妄想にはウンザリだぜ。
216ご冗談でしょう?名無しさん:2001/07/06(金) 14:18 ID:wkBMIEzs
最近の215さんを筆頭とする、前スレ27系の人の反論も
キレとコクにおいて精彩を欠いてますね。単なるいちゃもん
付けにしか思えない。関係の無いことにミスリンクさせて遊んでる
だけ。
217ご冗談でしょう?名無しさん:2001/07/06(金) 14:40 ID:JjenGelU
>>216 わしもそーおもう
218ご冗談でしょう?名無しさん:2001/07/06(金) 14:42 ID:???
>>216
んじゃ、キミのそのキレとコクで本筋の話題に戻してミソ。
それができないなら、くだらねーレス入れんなよ。

ベルヌーイの定理で翼の揚力を説明しようとするならば、
そのような速度分布の生じる原因について言及する必要がある。
ただし、ナビエ‐ストークス方程式を解けば速度分布はわかる
ってのは説明になっていない。

こんなところかな。これができないから「理想気体の粘性」の
話で誤魔化してるんじゃないの?
219 :2001/07/06(金) 14:59 ID:v7tCA31M
理想気体は粘性を持っていて群運動しちゃいけないということでよろしいでしょうか?
220ご冗談でしょう?名無しさん:2001/07/06(金) 15:28 ID:???
よろしくはないが、このスレを読んでそんなバカな結論で納得する
なら、それでいいだろう。その程度のレベルということだな。

翼の揚力の話にもどるから、誰か>>218以外の問題点をまとめてくれ。
221219 :2001/07/06(金) 15:46 ID:v7tCA31M
皮肉ですだw
222ご冗談でしょう?名無しさん:2001/07/06(金) 16:54 ID:M6SfBmOU
クッタジューコフスキーの話はどうなったんだ?
223 :2001/07/06(金) 17:48 ID:v7tCA31M
揚力Cl∝Aα+B
平面翼の場合は明らかにB=0だし、背面飛行を見るとA>>B。
Bは翼断面の非対称から来るベルヌーイだけ。
仰角比例成分Aは下面の弾性衝突による力。
仰角αが小さい時は
揚力Clは弾性衝突+ベルヌーイで計算できる。
偏向された流束の運動量は求めなくて良い。
つうことでどうでしょうか?
224NASAしさん:2001/07/06(金) 18:16 ID:QCuGjnOg
すみません。ちょっとお邪魔します。
こちらのスレには、航空、船舶板よりの避難民、移民の方はいらっしゃいませんか?
現在、スッチー、エアラインヲタ族と全面戦争中です。
出来たら、戻って論議にご参加お願いできないでしょうか?

関連スレ

航空船舶板「航空、船舶ローカルルール」
http://mentai.2ch.net/test/read.cgi?bbs=space&key=988123624

批判要望板「学問・理系>航空、船舶ローカルルール申請」
http://teri.2ch.net/test/read.cgi?bbs=accuse&key=994328331

批判要望板「航空船舶板からスッチー・エアライン板独立希望」
http://teri.2ch.net/test/read.cgi?bbs=accuse&key=993259456

お邪魔しました。腰を折って申し訳ありません。
でも、近々、この種のお話を航空、船舶板で出来るようにがんばって
おりますです。 ご理解、ご支援の程を。
225ご冗談でしょう?名無しさん:2001/07/06(金) 20:29 ID:???
>>223
だからベルヌーイの定理で圧力を求めるのには、速度分布を知る必要があるだろ。
その速度分布はどーやって決めるんだ?

式を書くときは、もう少しちゃんと記号の説明をしないとわからんぞ。
226ご冗談でしょう?名無しさん:2001/07/06(金) 20:37 ID:???
>>222
クッタ‐ジューコフスキーの定理の何が問題になってるんだっけ?

>>224
確かにスッチーネタは学問じゃないな。
独立して趣味板に行った方がシアワセと思われ。
227 :2001/07/06(金) 22:42 ID:v7tCA31M
>>225
対称翼断面で計算する。
228 :2001/07/06(金) 22:56 ID:v7tCA31M
工学の方の揚力の式です。
Cl∝Aα+B
Cl:揚力
A:仰角比例成分の係数
B:仰角独立成分の係数
α:仰角
229ご冗談でしょう?名無しさん:2001/07/06(金) 23:07 ID:yT.xzpuw
215はどうしてわかってくれないんだ!
じゃあ、別の角度から考えて、
本当にもし理想気体に粘性がないならば、
物体っと接触している流体は物体と同じ速度を
もつという、境界条件を用いることができないぞ。
粘性の無い気体が物体と同じ速度になるはずが無い。
つまり、理想気体という、単純で扱いやすいモデルが
使えないということになる。
飛行機の揚力を計算するときには理想気体を使えないって
ことになるぞ。
230ご冗談でしょう?名無しさん:2001/07/06(金) 23:13 ID:/EMgO9ow
前スレの27氏と、理想気体粘性無し論を主張する人との
対決をキボンヌ
前スレの27氏は、空気と翼の粘着性を揚力の原因としてい
たようだ。
231 :2001/07/07(土) 00:25 ID:iGUyFgVQ
理想気体スレたてれ
232ご冗談でしょう?名無しさん:2001/07/07(土) 04:49 ID:???
>>223
>揚力Clは弾性衝突+ベルヌーイで計算できる。
>偏向された流束の運動量は求めなくて良い。

ん? 弾性衝突の計算は偏向された流れの運動量の計算そのものだぜ。

>>227
「計算しないとわからない」では説明になってない、とゆーこと
じゃないの?

>>228
仰角の定義も知りたいぞ。この式の意味するところから考えると
「有効仰角」みたいなものなんでしょ。つまり揚力がゼロのとき
の仰角をゼロと定義している。そーすると仰角の小さい範囲では
揚力と仰角は比例するけど、仰角が大きくなると直線からズレる。
で、そのズレ具合を表すのがBかな。
とゆーわけで、「有効仰角」だと翼の下面と上面のどちらが大事
なのかわからないぜ。
233ご冗談でしょう?名無しさん:2001/07/07(土) 04:59 ID:???
>>229
粘性無しの気体で揚力を考えようとしてダメだった、って話が
前スレで出ているがちゃんと読んだのか?
揚力を求めるときは理想気体で計算しているわけではないぜ。
空気には粘性があるが、流速が十分速ければ翼から離れたところ
の粘性は無視できるってだけの話だ。

>>230
前スレで27が「理想気体に粘性がある」と主張したことは一度も無いが...
234ご冗談でしょう?名無しさん:2001/07/07(土) 05:09 ID:/gi8d3Co
>>230
全面的に賛成とりあえず、>>209さんと前スレの27さnの対決よろしく。
235228:2001/07/07(土) 05:28 ID:3s0.ber.
>>232
仰角は対称翼断面で考えれば明らか。
Bは定数。
236228:2001/07/07(土) 15:23 ID:DLUCCKNg
すまん、揚力はCLじゃなくて L でした。
Clは揚力係数。
237ご冗談でしょう?名無しさん:2001/07/07(土) 16:15 ID:???
>>235
対称翼断面のときは仰角がゼロのとき揚力はゼロ。だからBはゼロ。
仰角が小さいときには揚力は仰角に比例するって経験則を表したに
すぎないんじゃないの。仰角が大きくなれば比例しないでしょ。

非対称翼断面のときの仰角はどーやって測るの? 後端から前端の
下側に引いた接線と地表のなす角度?

で、巡航状態での仰角はいくらになるの? その仰角が大きいのなら、
ベルヌーイの定理で圧力差を用いた説明は大間違いってことになるが。
238弾性衝突派:2001/07/07(土) 16:51 ID:DLUCCKNg
>>237
仰角が大きくなれば失速する。
それまでは大体正比例する。
非対称の方は忘れましょう。
巡航状態での仰角はいろいろ。
239ご冗談でしょう?名無しさん:2001/07/07(土) 17:35 ID:StmOHrzs
カキコする奴は前スレからの流れを押さえてくれ。
27の示した参考文献を読んで、無駄レスを避けようぜ。

http://www.allstar.fiu.edu/aero/airflylvl3.htm

L∝Aα+Bってあまり見ない式だけど、αは有効迎角っぽい
ね。Aα >> Bだとおもうぞ。よって>>238の案に賛成。
ベル派はBが 主たる揚力だと言ってることになるんじゃない。
ちなみに旧スレ27はBを否定してないが、Aα >> B or
Aα > Bと主張してるはず。 >>197-200あたりから再開か?
240ご冗談でしょう?名無しさん:2001/07/07(土) 18:07 ID:NTS2rdvc
速度差→圧力差→揚力→飛ぶ
241弾性衝突派:2001/07/07(土) 18:14 ID:DLUCCKNg

        揚力 翼面が受ける力
      翼面\↑/
空気分子○→   \→抗力
         ↓\
         ○
        反射された空気分子

        揚力 翼面が受ける力
      翼面\↑/
動圧x投影面積→ \→抗力
         ↓\         
        偏向された気流の受け取る力
242ご冗談でしょう?名無しさん:2001/07/07(土) 18:18 ID:???
>>240
速度差の説明は無しか?
243弾性衝突派:2001/07/07(土) 18:19 ID:DLUCCKNg
        
        揚力 翼面が受ける力
       翼面\↑/
空気分子○→   \→抗力
            ↓\
         ○
        反射された空気分子
>>242無し
        揚力 翼面が受ける力
       翼面\↑/
動圧x投影面積→ \→抗力
            ↓\        
        偏向された気流の受け取る
244弾性衝突派:2001/07/07(土) 18:23 ID:DLUCCKNg
こんどこそ・・・
        揚力 翼面が受ける力
       翼面\↑/
空気分子○→   \→抗力
            ↓\
         ○
        反射された空気分子

        揚力 翼面が受ける力
       翼面\↑/
動圧x投影面積→ \→抗力
            ↓\        
        偏向された気流の受け取る
245弾性衝突派:2001/07/07(土) 18:25 ID:DLUCCKNg
空気分子がずれてるけど、
まあ、大体判りそうなんで止めとく。
246ご冗談でしょう?名無しさん:2001/07/07(土) 18:31 ID:???
>>245
何を言いたい?
247ご冗談でしょう?名無しさん:2001/07/07(土) 18:40 ID:NTS2rdvc
>>242
一緒に到着しようと頑張っると
翼の上面と下面で流速が異なってしまう。
がんばれ流体分子!
248ご冗談でしょう?名無しさん:2001/07/07(土) 18:55 ID:???
>>247
ネタか?
249ご冗談でしょう?名無しさん:2001/07/07(土) 20:05 ID:/dSbcrZE
前スレの27は、やはりコテハンとして「前スレの27」とでも
使うべきだ。そうしないと議論にならんぜ。
>>239当たり、非常にアヤしいが如何に?
幾らネットでも、自分の意見とちょっとでも違うと、すぐ他人を
トンデモとか極端な表現をするから、妙な煽りが入るんだ。
それを避ける為に匿名としたい気持ちも分からんわけではないが。
それなりに啓蒙の意思があるのなら、はっきりとコテハンで出てくる
べきだ。
250弾性衝突派:2001/07/07(土) 20:26 ID:DLUCCKNg
>>247
つうか、仰角考えると下面の密度の方が大きくなって
上面は密度低下、下面の流速>上面の流速となって具合が悪い。
251ご冗談でしょう?名無しさん:2001/07/07(土) 21:40 ID:???
>>249
前スレで27がトンデモだと決めつけていたのは、自分の意見と違う
からではなくて、物理的に明らかな間違いなのにそれを改めようと
しない場合だったと思う。今のスレだと「理想気体に粘性がある」
みたいな書き込みでしょう。
最近のレスは「低いぞ」ってレベルのものが多いので、
「議論にならんぜ」という気持ちもわかるよ。
252ご冗談でしょう?名無しさん:2001/07/07(土) 22:04 ID:UGPreJ2o
やれやれ。。
理想気体と完全流体とはちがうだろ?
これまでさんざん議論になってるんだから、それくらい確認してみれ。
253ご冗談でしょう?名無しさん:2001/07/07(土) 22:13 ID:StmOHrzs
>>249
俺が>>239だけど、旧スレ27じゃないよ。
もちろん、旧スレ27説支持派だけど。
(ちなみに考え方を指示してるだけで、ネット上での彼の
人格を指示してるわけじゃないから叩かないでちょ)
してる
254ご冗談でしょう?名無しさん:2001/07/07(土) 22:16 ID:StmOHrzs
239だけど、前スレで一時そうだったように
自分が27支持派かベル派か中立派か(それ以外か)
一言書き加えればいいんじゃない。
255ご冗談でしょう?名無しさん:2001/07/07(土) 22:51 ID:???
>>252
物理学辞典には「理想気体=完全気体」「理想流体=完全流体」と
出ていて、「理想」と「完全」は同義で使われています。さらに、
気体は流体であるとなっていますので、「理想気体=完全流体」な
わけですが、それを否定するのですね。
それは客観的に見てトンデモでしょう。
256ご冗談でしょう?名無しさん:2001/07/07(土) 23:54 ID:???
うみ=sea、ねこ=cat
だからって
うみねこ=sea cat
じゃあねえだろ。え、例えがようわからんって?スマソ。
257ご冗談でしょう?名無しさん:2001/07/08(日) 00:23 ID:i.zXhVqU
>>251
>前スレで、27がトンデモだと(私が)決めつけていたのは、(27が)
>自分の意見と違う からではなくて、物理的に明らかな間違いなのにそ
>れを改めようと しない場合だったと思う。
258ご冗談でしょう?名無しさん:2001/07/08(日) 00:24 ID:i.zXhVqU
今のスレだと「理想気体に粘性が無い」
みたいな書き込みでしょう。
最近の(27支持派の)レスは「低いぞ」ってレベルのものが多いので、
「議論にならんぜ」という気持ちもわかるよ
259弾性衝突派:2001/07/08(日) 01:16 ID:fscVEtpQ
動圧と静圧だけ考える。
翼に作用しているのは表面の流速0の静圧だけ。
境界層はこの表面の静止気体と動圧の相互作用によって生まれる。
途中の相互作用を考えず、原因の動圧と結果の静圧だけに着目する。

動圧x投影面積=下面の静圧増加x翼面積。
260弾性衝突派:2001/07/08(日) 01:16 ID:fscVEtpQ
動圧の式から投影断面積の受ける力を求める。
F=1/2xρxV^2xS
より、投影断面積(Sp)が受ける力は
Fp=1/2xρxV^2xSp
投影断面積は
Sp=Swsinα
投影断面積の受ける力は
Fp=1/2xρxV^2xSwsinα

翼面が受ける力Fw=1/2xρxV^2xSwxsinα
揚力Lは
L=1/2xρxV^2xSwxsinαxcosα
となる。
Sw:翼面積
α:仰角
261ご冗談でしょう?名無しさん:2001/07/08(日) 01:58 ID:???
前スレ27支持派です。

「弾性衝突派」さんの式はわかるんだけど、何を
言おうとしてるの・・・。しばらく静観してた方
が良いかも。

27支持派は、翼が空気に対して仕事をして気流に
下方向成分を与えるって事で良いんでないの。弾
性衝突が主因かどうかは疑わしい(翼上面が関係
なくなるでしょ)けど、粘性があるから気流を曲げ、
その反作用が揚力という事で一応完結してるんじ
ゃないの。

むしろ今はベル派の反駁の時間じゃない?
翼上面に負圧ができる理由を述べて、その負圧方向
に引き込まれる力だけで揚力が説明できるって話し
を展開しなきゃいけないんじゃないかと思うけど。
262弾性衝突派:2001/07/08(日) 02:28 ID:fscVEtpQ
完結してたのか。それは気付かなかった。
263ご冗談でしょう?名無しさん :2001/07/08(日) 03:43 ID:???
↑だから、カキコするなら前スレ+このスレ
の流れを把握してから書けっつうの、全く。
264弾性衝突派:2001/07/08(日) 04:09 ID:fscVEtpQ
前スレ見つからん。
このスレ見る限り完結しとらんように見えたが。
265弾性衝突派:2001/07/08(日) 04:09 ID:fscVEtpQ
皮肉でいったのにな。完結ねえ。
266ご冗談でしょう?名無しさん:2001/07/08(日) 09:17 ID:???
>>261
そう、27派が期待しているのは、速度分布の説明でしょう。
 揚力←翼の上下の圧力差←圧力分布←速度分布←その原因
方程式を解けばわかるってのは、説明になってないからね。

弾性衝突派は間違っていることを書いているわけでないが、
今更そんなあたりまえのことを言われても…ってカンジ。
翼上面で生じている下降気流については「弾性衝突」では
説明できないでしょ。
「巡航時の揚力の大部分は翼下面での空気の弾性衝突」
ってデータがあるなら話は別だけどね。
267ご冗談でしょう?名無しさん:2001/07/09(月) 04:15 ID:crB66bUw
>翼上面で生じている下降気流
この原因について、もう一度「27派」の人説明してちょ
268ご冗談でしょう?名無しさん:2001/07/09(月) 06:35 ID:???
27ではないが、前スレ27派

一言でいえば「空気の粘性」が原因
粘性がある→翼にまとわりつく
→翼上面に沿って流れる→下方向に曲げられる

じゃこれを分子論的に説明しろと言われると
俺はできない。ちなみに27は前スレ以来>>239
再掲してあるリンク先を参考文として説明している。
上の説明もここにある。ベル派はまずこれを読めば
如何だろう。そうすれば>>267のような質問はしない
か or 質問するにしても別の訊き方をするだろう。
269ご冗談でしょう?名無しさん:2001/07/09(月) 06:40 ID:???
268だけど、

煽るわけじゃないが、ベル派は27派の理論のツッコミ
をするのも良いが、それに終始せず、きちんとした
ベル派理論の説明をするのは如何か。
270>268:2001/07/09(月) 10:16 ID:???
27だって翼上面(下面)の流速が速くなる(遅くなる)のは認めてた。
それは粘性で説明できるのか?
271ご冗談でしょう?名無しさん:2001/07/09(月) 10:49 ID:???
>>270
ちゃんと話の流れを読み取らないとイケナイな。前スレで27は
「ナビエ‐ストークス方程式を解いて揚力を計算する」という
立場が間違いではない、とゆーことを「認めた」んでしょ。
で、それは間違いではないが「方程式が解けただけでは揚力を
理解したことにはならないから、ベル派はちゃんと説明してね」
ってのが今のスレの流れだよ。ノイズが多いけどね。
今のところ、その回答はゼロ。ベル派の論客はいずこへ?
272じゃん:2001/07/09(月) 11:26 ID:US6ESeJ.
空を飛べるから飛行機なんだよ。
273ご冗談でしょう?名無しさん:2001/07/09(月) 15:00 ID:FUhl4rUY
>ベル派
ベルヌーイの定理はあくまでも比喩であり、動圧と静圧が独立に値を
取ることが出来ないということを表現の具体例として使っており、
別にくだんの形に拘っているわけでは無い。もっと別のタイプのもの
が適切かも知れないということを認めながらも、恐る恐るそれを使って
議論しているだけ。
そもそもベルヌーイの定理は、ナビエストークス方程式からも導出され
るが、物理理論的には必ずしもそれに依存したものでは無い筈。
あくまでも、動圧と静圧の依存関係を言っている法則でしょう。
標準理論派をベルヌーイ派と同一視するのはもうやめて欲しい。

方程式をたてるのは、思考に最低限の枠組みを付ける為。
それを否定してしまうと、何でも説明できてしまって超ナンセンスな
理論が幾らでも出来る。枠に囚われない発想も大事だけど程度はある。
神・霊の存在まで物理で示されたら、TOE(Theory of EveryThing)に
なってしまって、物理理論の価値は0になってしまう。

>空気が翼にまとわりついて
確かに、その理由を圧力(圧縮性)以外の要因で説明すると、どうしても
奇妙な力が発生するからということになる。
しかし、物理現象であるそういう力は他の現象でも見られる筈である。
その具体例をキボンヌ
274ご冗談でしょう?名無しさん:2001/07/09(月) 19:44 ID:???
>>273
>標準理論派をベルヌーイ派と同一視するのはもうやめて欲しい。

では、ベル派とは異なる「標準理論派の揚力の説明」を書いてみてね。
275ご冗談でしょう?名無しさん:2001/07/09(月) 21:23 ID:???
268だけど、

>>273
スマソ、俺はベル派というのはベルヌーイの定理を使うと
いう意味でなく、とにかく翼の上下面で圧力差(分布)が
できて、その圧力差(分布)により発生する力が揚力の本
質であるという説の人を指していたんだ。他の人もそうじ
ゃないのかな?繰り返すが、この力は27も否定してない
がマイナーファクターだという事。「標準理論」と言う
呼び方は抵抗あるな。「ベル派」のままじゃダメ?

粘性に起因するまとわりつく現象は「コアンダ効果」だっ
てば。27は前スレ以来>>239に 再掲してあるリンク先
を参考文として説明しているから、これをベースに議論し
ようと言ってるじゃないか。しっかりしてくれ。
276ご冗談でしょう?名無しさん:2001/07/09(月) 21:41 ID:???
↑ スマソ、具体例って言われてたね。蛇口からの
水流にスプーンを入れると凸面方向に引かれるって
やつだよ。
277ご冗談でしょう?名無しさん:2001/07/09(月) 23:20 ID:???
>>275
圧力差に関して、その認識の仕方はちょっと違うんじゃない?
27が「圧力差の揚力への寄与」を問題にしていたのは、空気を
非粘性と仮定した場合じゃなかったかな。粘性を無視したために
「理論的に飛行機はムリ」と考えられていた100年前の話じゃないの?
前スレで27が主張していたのは
「圧力差(=流体の速度分布)にとらわれすぎると本質を見失う」
ってことだと思う。実際「揚力=圧力差」を主張する立場からは
翼周りの速度分布について納得できる説明はなされていないよ。
「圧力差は実用的に揚力を計算する手段」って認識だったはず。

このままでは「結局、圧力差では揚力を理解できないって」結論に
なりそうだけど、良いんですか? > ベル派、標準理論派
278 :2001/07/10(火) 14:13 ID:68LgeE.2

     →  
  空気 →   
     →    
279弾性衝突派:2001/07/10(火) 14:23 ID:68LgeE.2
上はミス
ベル派、標準理論派ではこの棒がどっちに動く事になるの?
     →  
     →   ┌─────────────
  空気 →   │   −>無限に長い棒        
     →   └─────────────
     →   
280ご冗談でしょう?名無しさん:2001/07/10(火) 14:34 ID:???
↑ うわっ、またコイツわけわかんない事を書きに
来やがった!嵐でやってんのかよ。でも強烈な個性
でスレに華を添える名脇役としては貴重な存在かも。

ボコボコにされて退場きぼんぬ。

P.S.
おまえ今まで何聞いてたんだ?
あと、前スレ読んだのか?
281ご冗談でしょう?名無しさん:2001/07/10(火) 14:49 ID:RKbT7cBM
>粘性を無視したために
>「理論的に飛行機はムリ」と考えられていた100年前の話
補足。これは非粘性流体の定常解では揚力も抗力も発生しない
だろうという予想から出てきたこと。
定常解は全て揚力・抗力が無いのか?
定常解にそもそも収束するのか?
という議論は、現在も研究中。(解の存在すらそう)
まぁ、揚力に関しては、定速飛行していても、時間の経過と共に
徐々に失われていくだろう...というのが100年前の予想。

定常状態に近づく過程では揚力は存在することを否定するものでは
無い。簡単に言えば、流れが障害物に「慣れ」うまくかわして
流れるようになるので、揚力・抗力が無くなるだろうということ。
粘性が、「流れの慣れ」を促進するのか妨害するのかは、実は
まだ分かっていない。抵抗の増大は間違いは無いだろうが。

でも、実際の飛行機では、例え粘性が無くとも加速したり、迎えを
変化させて気流を変化させる等の処置で、揚力を失わさせないよう
にも出来るだろうから、飛行機の研究は精力的に続けられて
20世紀に開花したということ。つまりダランベールのパラドックス
自体は飛行機の工学的研究には対して影響を与えず、純理論的な
興味の問題だったわけ。
282ご冗談でしょう?名無しさん:2001/07/10(火) 15:15 ID:???
「コアンダ効果」は現象論的には実存するようです。
その原因で意見が分かれているようです。
いずれにせよ、翼が空気中を移動するのに、翼上面に
「真空の間隙」を作らなければ移動できず、そこに空気が
侵入して埋め合わせなければならないことになります。
埋め合わせるのには空気を加速して流入させなければならない
のですが、空気を加速する力について議論が分かれているようです。
大気圧によって、(翼から見て)消極的に行われているのか?
それとも、強制的な束縛力として翼と空気の界面で相互作用が
起こっているのか?でしょう。空気の粘性の為、動かさなければ
ならない空気が膨大になり、束縛力は大きくならざるを得ない
という理由も付けられます。
でも、後者なら、翼の前後を入れ替え(迎えは同じ)た形状の
ほうが、揚力が大きくなって有利になるのではないでしょうか?
なにせ、通常の飛行機の翼の前縁は、結構急に切れ上がって
ますからね。それが後縁に来ると、そこで巨大な力で空気を急激
に引き込み、翼にはより強大な揚力が発生するでしょうに。
束縛力なら、一定時間に出来るだけ長い距離引き込めるかが
勝敗を分けることになるでしょうから。
飛行機ってバックで飛べたっけ?(背面飛行じゃないよ)
283ご冗談でしょう?名無しさん:2001/07/10(火) 15:53 ID:???
マイナーファクターって?
実際に翼面の圧力分布を測定すると、それは計算結果と合うし
その圧力差の総和は揚力とも一致するけど?
284ご冗談でしょう?名無しさん:2001/07/10(火) 17:15 ID:mY2v1gIg
285 :2001/07/10(火) 18:39 ID:???
例えば静止した流体中を急に翼が動くと、翼の後ろには反時計まわりの強い渦が発生するのが観測できる。これと反対の強さの渦がどこかに存在しなくてはならないが、それが翼にできていると考えられている。

-----------------------

この最後の部分の、翼にできているという渦ってどういうこと?
見えない渦ができてるの?
286 :2001/07/10(火) 23:30 ID:???
age
287ご冗談でしょう?名無しさん:2001/07/10(火) 23:39 ID:???
>>283

27派に近い中立派ですが、
その話し以前から気になってたんですよ。
ネット上にソース(情報源)ありませんかね、
勉強したいので、教えてください。
(このスレの過去ログって答えは無しね)
288ご冗談でしょう?名無しさん:2001/07/11(水) 00:00 ID:???
>>284
結論は予想通り
「揚力は難しくて説明できない。ナビエ‐ストークス方程式を
 解けば計算はできる。それも大変だけど」
ってオチでした。

途中の説明、変なところ一杯。

回転円筒の揚力の説明の部分は、粘性がありかつ仕事をされている
のに、いきなり「ベルヌーイの定理」を適用してる。
「ベル厨」みたい…

揚力には渦が重要なのはその通りだが、翼周りに渦が発生するのは
発進渦があるからだと思ってるみたい…

「下降気流は揚力が発生する理由ではなく、発生した結果」って
ことを書いて、「27派」に対して反論してるつもりみたい…

「翼の無い胴体だけの「飛行機」でもコアンダ効果は生じるが、
 十分な揚力は得られない。だからコアンダ効果では揚力は説明
 できない」だって。翼のコアンダ効果は無視か?
空気の粘性を忘れちゃってるみたい…

とても航空工学の教授が書いているとは思えません。
きっと研究室の院生あたりに書かせたのでしょう。
このスレを教えてあげましょう。
289ご冗談でしょう?名無しさん:2001/07/11(水) 05:53 ID:bRrBFu9.
マイナーファクターなら、穴があいた翼でも揚力が得られるのかな?
282じゃないけど、翼を前後逆に付けても飛ぶのかな?
290ご冗談でしょう?名無しさん:2001/07/11(水) 06:41 ID:???
気流に対して充分な量の下向き運動量を与えられれば
揚力は得られるんでないかい...例えどんな翼でも。

ただし、孔(にも依るだろうが)があいた翼や前後逆
の翼(充分なアップウォッシュが得られない+気流が
剥離してダウンウォッシュが得られない)は下向き
運動量を与えられるとは思えんよ。
291ご冗談でしょう?名無しさん:2001/07/11(水) 09:10 ID:bRrBFu9.
>充分なアップウォッシュが得られない
これと下向き運動量の関係は?
一見矛盾と感じるが如何に?
292ご冗談でしょう?名無しさん:2001/07/11(水) 10:25 ID:???
>>291
翼上面で効率よく下降気流を発生させる場合、滑らかに気流を
上昇させてそれを粘性で引張って下向きにするしか方法は無い
でしょ。
下降気流だけを作ろうとして、くさび形みたいな翼断面にすると
抗力が大きくなってしまうよ。

ナビエ‐ストークス方程式の解から翼周りの圧力分布を「計算」
する立場では、揚力を理解したことにはならないのだから、
ベル派もそろそろ「圧力差」から脱却したら?
293ご冗談でしょう?名無しさん:2001/07/11(水) 13:09 ID:???
>>291
290だけど、27の示した参考リンク先を理解した上で
レス書けって何回も言ってるじゃん。書いてあるよ。
頼むからしっかりした議論してくれ。
294ご冗談でしょう?名無しさん:2001/07/11(水) 16:55 ID:???
>ナビエ‐ストークス方程式の解から翼周りの圧力分布を「計算」
>する立場では、揚力を理解したことにはならないのだから、

なんで?
295ご冗談でしょう?名無しさん:2001/07/11(水) 17:01 ID:0JqMIBk.
だから、粘性があると何で翼上面の流速があんなに速くなるのか
(迎角をもった平板でもね)アホの俺にも分かるように
説明してくれ。
296ご冗談でしょう?名無しさん:2001/07/11(水) 17:38 ID:???
>>294
流体に関する運動の法則を微分方程式で表したのが、ナビエ‐ストー
クス方程式。方程式を解くとゆーことは、運動の法則にしたがって
計算しただけで現象を理解したことにはならない。

「逆立ちゴマ」の運動方程式をたててシミュレートすれば、コマが
逆立ちする様子を再現できるけど、「何故逆立ちするのか」に対す
る答えにはなってないのと同じ。

>>295
「あんなに速く」ってどのくらい速くなってるんだと思う?
297291:2001/07/11(水) 17:41 ID:???
リンク先何回も読んでる。わざわざプリントアウトして。
もう大分前から。
「コアンダ効果」の原因が、空気と翼の間の相互作用
(要するに力)だとすると、紡錘型(断面は翼と異なり
上下対称)の物体が空気中を移動
する場合物体全体を(トータルで)膨張させおそらく終局的
には円柱に変型させるような応力が働くことになるよね。
別に高度は上げていなくても。
こういうのって、工業的に応用されていてもおかしく無い?
実際は、逆の加工(円柱を紡錘状に変型加工する)は聞いた
ことがあるが、紡錘状を円柱にする加工は聞いたことないよ。
298291:2001/07/11(水) 17:46 ID:???
もう一つ
やっぱり逆に翼を取り付けたほうが、揚抗比は上がるよ。
「コアンダ力(界面接触力)」を認めるなら。
揚抗比が上がることが大事だから。
299ご冗談でしょう?名無しさん:2001/07/11(水) 18:01 ID:???
>>298
流線型って知ってるか?
これの前後を逆にしたほうが抗力は減るってことを
主張してるのかい?
300298:2001/07/11(水) 21:48 ID:???
翼型は流線形とは異なる。流線形の場合、それこそ揚力を得る
のはそれこそ大変。
逆に取り付けた場合
F1レーシングカーのように先端が低く、後端がやや切れ落ち
ている形状になるか。
これを考えると、翼は必ずしも抗力の減少を優先に作られた
わけでも無いことがわかる。車の場合、何故断面が翼型の
ように作られないのか?揚力が発生し、荷重がタイヤにかか
らなくなるからってのはダメね。より小さな抗力を犠牲に
して、ウィングで下向きの揚力を作ってキャンセルするから。
車は完全に抗力との戦いの世界でしょ?

逆に取り付けた翼の話も面白いけど、上下を貫通する小穴が多数
開いた翼が飛ぶかのほうが興味深い。
コアンダ効果がうまく起こるように斜に付けるとかすることは出
来そう。穴だからね。剥離など起こらないんじゃ
ない?剥離が起こったら翼から離れていくからね。
穴にうまく誘導するように、流れに平行な浅い溝を付けるって
のもテかな?穴の中で一気に下向きに誘導する。
301ご冗談でしょう?名無しさん:2001/07/12(木) 00:07 ID:???
>>300
そんなに思い込みを並べられても議論にならんぞ。
ちゃんと物理的根拠を書きな。

小穴のあいた翼ねぇ。
穴のあいた団扇じゃ風は起こせないのかい? 穴の数に依存するだろ。
302ご冗談でしょう?名無しさん:2001/07/12(木) 00:17 ID:???
>>295
どのレスに対する、あるいは何派に対する質問?多分、
議論の流れを把握してないが故のちぐはぐな質問に思
える。>>296のツッコミおよび>>287(>>283)の質問の
回答が欲しい所だ。


>>297-298, >>300 (=291)
すまんが、論理的展開が今一つわからない。
論理展開がわかるような日本語に直してもらえないか?

>>300の最後の段落だが、流れがどういう風になるのか
興味あるところではあるが、うまくいきそうにないと
感じる。だが、いろんなアイディアを出して試してみる
事は良いことだと感じる。
303ご冗談でしょう?名無しさん:2001/07/12(木) 02:52 ID:TQ6fxfv.
>>300
先っぽは尖っているほうが抗力は大きくなります。
レーシングカーでもつんつんには尖ってないでしょ?
ほかに、潜水艦や、新幹線、飛行機(超音速機は除く)の鼻先だって丸いでしょ?
304長文スマソ:2001/07/12(木) 02:54 ID:???
横槍長文スマソ(5レスに跨ります。)

[その1]
ベル派=標準派、粘性派、思い込みは両者にありそう。

問題の核心部分は、やはり翼の上を通過する空気が、どのようにして
翼から力を受けるのかという点に尽きる。

普通の考え方というか、世間に広く流布している一般的な考え方では
基本的に翼は空気の圧力(膨張力・内部の加速力)を介して空気と
相互作用していると考える。つまり翼に侵入出来ないという条件以外
ほぼ自由に運動している空気が内部で発生させる圧力が翼にかかる応力
に延長されるとするもの。

「粘性」が主張する「コアンダ効果」は、通常の圧力等の流体
力学で広く認められている形式の力以外に、更に特別な力が空気と翼
の間に働くということを暗示している。これがこの派の主張の第一の
特徴。圧力が応力に直接伝達されるのではなく、特別な力を介して
翼に応力として伝達されるらしい。だから上面の圧力が殆ど下面と
変わらなくとも、特別な力が大きければ揚力が発生することになる。

この力は純粋に界面だけに作用する力だということになる。
剥離に言及していることから、この力は、常に一定の大きさで発生す
るわけではなく斜面上を動く物体に働く抗力に代表される束縛力のよ
うに、両者の運動状態によってフレキシブルに変化するタイプ。
305長文スマソ:2001/07/12(木) 02:54 ID:???
[その2]
その条件を、物理的な現実と照らし合わせながらまとめてみた。
この力をコアンダ力と便宜的に呼ばせて貰う。一応敬意を表して。

・翼表面から一定範囲(コアンダ領域と命名)にあるとその力は作用
 する。
・コアンダ領域内でそこを通る空気塊を束縛する力であり、塊は隣接
 する空気塊との相互作用の他にこの力を受ける。
・上限はあるが、一定範囲に入るべく、コアンダ力の大きさはかなり
 大きくなれる。(その上限を超える力が必要な場合が剥離)
 例えば上部隣接空気塊がその空気塊を引き上げようとしても、一定
 範囲内に留めるべく、コアンダ力が増加する。

「コアンダ力」はポテンシャル力か?という問題はさておいて
実はこれだけでは、揚力を説明出来ない。隣接する空気塊との間の
相互作用が大問題。その相互作用は分子間力との相関性が高いと思
われ。翼上部空気の膨張(密度低下)を妨げる力であると特徴付け
られそう。この力こそが、コアンダ力を増大させる主因であると思
われ。これが大きい為に、コアンダ力を十分に大きくさせ、翼に上
向き引力を与えることになる。ではこの力の正体は?
306長文スマソ:2001/07/12(木) 02:55 ID:???
[その3]
前スレではファンデルワース力と強引に結論つけられている。
しかしやはりこれは強引過ぎる。なぜなら気体に働くファンデルワ
ールス力は分子間距離に比例して弱くなる性質が仮定され、その力
が膨張と共に強くなるとは到底思えない。

空気は内部では、内力や重力以外働く力が無いと考えるのが普通。
分子間力を考えるにせよ、流体力学では内力として、内力(通常は
圧力)と密度を関係付ける式の中に組み込まれ、表には現れない。
だから少なくともマクロ(流体)の視点で見る限り、この力は密度
と圧力の関係に反映する筈。

空気の体積を増大させることに抵抗すべく、内部で働く応力というこ
とになる。(このスレで盛んに主張されている「粘性」という言
葉をより的確に翻訳してみた。)
そういう応力を反映した、密度と圧力の関係はどのようになるか興味
深い。
307長文スマソ:2001/07/12(木) 02:56 ID:???
[その4]
もしかすると空気の膨張*速度*には制限があると仮定すると更にこ
の説明により説得力が出るかも。その場合も密度・圧力の関係が更に
興味深いものになる。早く膨張させることが出来ず、無理に膨張させ
ると全体がある方向に動くということ。

尚、この力は空気が一定体積の初期条件で突然真空中に放り出され、
膨張する際にも本質的に働く力であるとも言える。空気は真空中で
も膨張する速度に制限が発生することになる。「コアンダ」
力で境界を均等に引っ張って無理に膨張させてやるしか、理想気体
と同じ膨張速度を得ることが出来ないことになる。

急速に膨張している空気は、膨張を加速するような外力に対し、完
全流体のように、それを全体に瞬時に伝え、平行力に分解して、膨
張速度上昇を阻止する性質があることが揚力の根本原因であると言
うことが出来そう。これによってコアンダ力が肥大化し、大きな揚
力となる。但しこれは「粘性]派の主張を認めればの話だが。
(通常これに似た効果は、断熱圧縮による温度変化を加味した
場合で得られる。ただ残念ながら通常の速度で飛ぶ飛行機の翼に
それは適用出来ないとされている。)
ベル派も流体に拘らなければ、翼前部で翼上の空気荷重伝達を一時的
に遮断し、下からの(圧力を媒介とする)荷重を受ける為という
ことが揚力の本質であると変形することが出来る。この差は揚力は
上が原因か下が原因かに結局は分かれることになる。
308長文スマソ:2001/07/12(木) 02:56 ID:???
[Last]
下降気流の運動量増加による反力=揚力とする人は、空気の膨張時の
密度・圧力の関係をより適したものに書き換えてから主張しないと
説得力が無いんじゃないの?
「コアンダ効果」による揚力の本性を流体の言葉で表現すると、空気
の密度(変化)と圧力の間の関係が、多分未知の関係で結ばれている
から、と言い換えることが出来る。(この場合、未知の関係によって
発生するエクストラな圧力を応力としても良いことには注意。
コアンダ力は圧力を翼表面に伝達する役割しかしていないからね。)

まぁ、いずれにせよ、
圧力=応力とダイレクトに結びつける標準派に対するアンチテーゼの
意味では、それなりに価値を持つ主張かも知れぬ(藁)。
309弾性衝突派:2001/07/13(金) 00:33 ID:lK0wlWRE
揚力は翼の上面が主なのか?
地面効果(地面近くでは揚力が増す)は説明できるの?
310ご冗談でしょう?名無しさん:2001/07/13(金) 00:44 ID:???
↑ うわっ、またコイツわけわかんない事を書きに
来やがった!嵐でやってんのかよ。でも強烈な個性
でスレに華を添える名脇役としては貴重な存在かも。

ボコボコにされて退場きぼんぬ。

P.S.
あんた自身は言われてないかもしれないが
参考文献読めって何回言われれば気が済むのよ。
311ご冗談でしょう?名無しさん:2001/07/13(金) 00:48 ID:???
310だが、

ちなみに俺は前スレで、地面効果の原因は翼端渦
により発生する誘導抵抗が減るためだと書いた。
現在の認識では、その効果よりも、参考文献中の
効果が大きいと考えている。(っつうか、10数
年前に俺はそう習ったのだが忘れていた。)
312ご冗談でしょう?名無しさん:2001/07/13(金) 00:53 ID:???
310だが、

>>304-308の「長文スマソ」氏のご意見拝見したが、
結局、氏による揚力の本質とは何になるのですか?

私の理解では、氏は「27派」であり、コアンダ効果
が起きる原因を詳細に(私には氏の考え方は今のとこ
ろ理解できていないが)述べているように見えるのだが。
313弾性衝突派:2001/07/13(金) 01:28 ID:lK0wlWRE
参考文献中の効果ってなに?
314ご冗談でしょう?名無しさん:2001/07/13(金) 01:55 ID:???
>>312
「長スマ」はバリバリの「ベル派=標準理論派」だろ。で、熱力学
や統計力学をちゃんと勉強していないためか、「空気は連続体」と
ゆー概念に縛られすぎていて粘性の本質が理解できていない。
「何を主張したいのかよくわかんなーい」ってカンジだ。

「27派」の揚力の説明も「標準理論」に包含されていると思い込
んでいるよーなので、一度、「標準理論」による揚力の説明を披露
して欲しいと思うぞ。
315長文スマソ:2001/07/13(金) 02:10 ID:???
そうとも言えない。流体力学ってのは、本質的にニュートン力学の
極端な拡張であり、相当に包括的な体系だってことよ。
前スレ27氏はそこの点に誤解があると思う。つまり、ニュートン
に立ち返ったつもりになって、それでもって流体力学を批判している
が、実は皮肉なことに、その理論を延長すると結局流体の方程式の
変形を論じてってだけの話。
それにしても、「コアンダ力]って接着剤のハナシに近づいているな
我ながら(藁)
将来、この理論が正しいと検証されたら、空気接着剤「コアンダ]が
発売されたりして(w藁)
316長文スマソ:2001/07/13(金) 02:17 ID:???
密度も流体では速度場と同様に場でっせ!
だから、連続体に囚われてっていう主張には何の説得力も無い。
百歩譲って囚われているとしても、。連続体としての特徴を一
切無視することはこと、空気と飛行機の翼の関係を論じる場合
は出来ない。
少なくとも前スレ27氏の意見には、離散的な立場でも連続的な
立場でも並行で議論でき、どちらの特性も生かされているわけでは
ない。
317長文スマソ:2001/07/13(金) 02:29 ID:???
「正統派」の揚力の説明も「前スレ27理論」に包含されていると思い
込 んでいるよーなので、もう一度、「前スレ理論」による揚力の説明
特に粘着性(接着力)についての詳細を披露 して欲しいと思うぞ。
318ご冗談でしょう?名無しさん:2001/07/13(金) 04:44 ID:???
>>315
そんなことを書いてるってことは、ラグランジュ法とオイラー法の
区別がついてないんじゃないの?

>>316
流体分子は互いに圧力を及ぼしあっていると思ってるんじゃないの?
だから粘性の本質がわからない。圧力も同様に「場」なんだがな。

>>317
「標準理論」の揚力の説明はまだスレに出てきてないので、説明
して欲しいだけなんだが、何故しない? そんなに難しいのか?
そもそも「標準理論」って何が標準なの?
319ご冗談でしょう?名無しさん:2001/07/13(金) 06:04 ID:???
>>318
そんなに圧力以外の力が、翼に作用していると信じるなら
それを認めさせるような、より単純な現象に還元して説明すべき
じゃない?
いつも貴女が批判している、圧力の総和=揚力を前提とし、圧力(勾配)
を内部相互作用として含む質点集合体として空気を扱うのが標準理論
ってことじゃないの>>314
(しかし、扱う人達の立場によってその設定に相当に幅があることは
確かだが)

(再び)>>314
本質的に揚力発生現象にしか現れない力の存在を信じていて、それが
本質だと貴女はいつも言っているわけでしょ。何せ「標準理論」では
本質的に説明出来ない現象だと貴女の文脈じゃ言っているようだ
から。つまり、貴女は実は「標準理論」による「揚力」が説明できて
それでは不足するから、新しい力を導入したいわけでしょ?
ベルヌーイの定理という、ちょっとした道具をほのめかしただけで
実は正式には誰も持ち出したわけじゃない、つまりこのスレには存
在するわけじゃない「幽霊理論」があるものとして、貴女は自分の
「理論」がより正確だと主張するわけでしょ。どういうわけだか知
らないが、貴女は既に標準理論を知っているわけだ。
つまり、「標準理論」を説明するのは、筋からして貴女だよ。
その後に具体的にそれでは説明出来ないから、未知の力の存在が
あるということを主張するのが筋だ。
そうじゃければ、Woman logicというもの。
「好きだから好きなのよ!」
320ご冗談でしょう?名無しさん:2001/07/13(金) 07:25 ID:???
えっ、>>314,>>318って女性だったの?
(しかし、また新キャラ登場>>319(=長スマ?))
32127の妹(藁:2001/07/13(金) 10:50 ID:???
>>319
アタシが知ってるのはベル派よ。翼に発生する揚力を圧力分布で
計算する人たち。で、最終的にはナビエ‐ストークス方程式を解
かないとわからない、ってことで説明にはなっていないのよ。
標準理論派も似たようなものでしょ? ベル派とは違うのなら。
それをわかるよーに書いてね。

プププッ。「圧力以外の力」だって。マヌケの上塗りね。
「長スマ」は「圧力=応力」に縛られちゃってるから、空気の分子
が翼に衝突して圧力になるってことも忘れちゃってるんじゃないの?
流体力学よりも学部1年レベルの熱力学を勉強したら?
322長文スマソ:2001/07/13(金) 14:42 ID:???
>>318
すべて貴方の妄想です。
ラグランジュ法とオイラー法は観測座標の取り方の相違で物理的には
等価です。共通の出発点から異なる数学的手法で導出されてるだけです。
「流体分子」(不正確な表現です。正確には微少流体要素)は圧力場を通
じて全体とつながっていると考えます。実際に作用するのは、その圧力場の
勾配であり、その他にも外力や別の内部応力である粘性等が考慮されます。
>>319さん(女性?)が指摘されるように、翼には周囲の流体の圧力場が
延長されてそれが応力として作用するという前提に基づくものをここでは
標準理論と呼んでいるみたいですね。(私が言い出しっぺじゃありません
から)
>>319(≠私)
>>320
擁護してくれるのは有り難いけど、どうみても、世の中をすねて見ている
鬱屈中年オジが、念性派の正体じゃない?
その通り、念性派が圧力分布を最初に想定してからその総和を求める方法
では、不足すると言っているのだからそれを具体的に示してもらわないとね。
>>321
前スレの27の妹さん?かわいそうな貴女は、40過ぎても処女じゃない?
(余計なこと言ったか。物理関係者はクチが悪いからね。)

私は前スレを読ませてもらったけど(未だにhtml化されてないみたいだけど
1000に到達する直前にすべて保存しました。)

最初の方で念性派代表であるお兄さんは、クッタジューコフスキーの定理
という流体力学の中でも最もキワに近い完全流体の理論を持ち出していた。
ベルヌーイの定理というか方法(圧力と速度の間の制約関係を設定して
圧力場なり速度場を推定する方法)よりも、もっとラディカルですよ、
クッタージューコフスキーの定理って。
でも、お兄さんは、最近、批判対象であるベル派(=標準理論派?)
の依拠するのが流体力学であることを批判しているようですね。自分も
本当はそうなのに。熱力学で圧力の原因は確かに説明されてるけど、
正に流体の方程式に(圧力と密度の関係で)それが連立されているのにね。
分子うんぬんの議論を避けたいが為に、そうしているのだけど、そこら辺の
ことわからないみたい。

え?結局、揚力があるから、圧力分布は良く解らないけど、結局下からの
圧力が相対的に強いことが推論されることが揚力の本質だと言っているの?
お兄さん?
それじゃ標準理論の出発点に戻っただけじゃん!そこで思考停止して
しまっていたわけね。
否、出発点は同時に目標点だった。もしかするとお兄さん、既にそこに
到達していたの?じゃ、圧力分布がわかってるのね。きっとその分布の
説明をしてくれるでしょ。次レスで。
323NASAしさん:2001/07/13(金) 15:12 ID:nYZYBS/I

こんなの、立てて見ました。たまにはこちらにも遊びにきてや?

「□■□低Reでdj□■□!!」
http://mentai.2ch.net/test/read.cgi?bbs=space&key=994908706
324ご冗談でしょう?名無しさん:2001/07/13(金) 16:25 ID:jei90//.
287で一度登場した中立派(というよりどちらが正しい
か判らなくなってきた派)ですが、どちら派の方でも結構
なのですが、

・「翼面の圧力分布の測定結果、それの総和と実際に発生
している揚力との関係」を示した文献(できればweb上で
公開されているもの)ってありませんでしょうか。どちら
派にとってもこれがポイントなのかと思います。

・翼によって下向きの空気の流れが生じている事は正しい
ようですよね(27派の参考リンク先に提示)。この下向
きの流れというのは翼が空気に対してした仕事かと思うの
ですが、翼面の圧力分布というのは、この下向きの流れを
作り出した結果の影響を含んでいるという事で良いんでし
ょうか?

・結局、そのような(ってどのようなか知りませんが)
翼面の圧力分布を生じる理由というのは何になるんで
しょう。
325ご冗談でしょう?名無しさん:2001/07/14(土) 00:43 ID:???
>>324
>翼面の圧力分布を生じる理由というのは何になるんで
>しょう。

27派は「分子間力」でしょ。でも、その詳しい説明は27は提示していない。
(出張中か?)ちなみに前スレで27は圧力差を否定していないし、圧力差
以外の力が働くとかは書いていない。圧力差にとらわれていると揚力の本質
を見失うと主張してたはず。

標準理論派(=ベル派なのか?)の回答はいまのところ無し。方程式を解け
ばわかる、とゆーのではない斬新なレスを期待しよう。>「長スマ」
326弾性衝突派:2001/07/14(土) 02:22 ID:BmEgl8.I
真空に近いほど分子間引力が強くなるとは、恐れ入りました。
327ご冗談でしょう?名無しさん:2001/07/14(土) 02:45 ID:???
↑ うわっ、またコイツわけわかんない事を書きに
来やがった!嵐でやってんのかよ。でも強烈な個性
でスレに華を添える名脇役としては貴重な存在かも。

ボコボコにされて退場きぼんぬ。

P.S.
多分、君が論理をちょっと取り違えてるよ。
328長文スマソ:2001/07/14(土) 03:02 ID:???
>>325
で、アナタの立場は一体?
やけに前スレ27氏の理論を良く知っておられるようだが、
アナタにとって揚力の本質とは?圧力差以外にあるという
わけですね。まず出張中の前スレ27氏に代わって、方程式を
明示的に使わない説明をよろしくどうぞ。
まず、そちらが「ベル厨]と呼ぶ説明力に乏しい理論の欠点を
具体的に指摘し、それをどのように改善したのか?(具体的には
空気と翼の間に働く力をどのように修正したのか)を明確に教えて
下さい。

ちなみに標準派の説明の共通点は、要するに翼の上面で空気の圧力
が(下面よりも)下がることが揚力の原因であるということです。
その理由が正にスレの中で明らかになるんじゃないかな?という
のが、前スレの最初の部分の雰囲気だったように思えます。

そこに粘着という「斬新]なアイデアを持ち込んでくださった方
がいらしゃって、混乱に陥ったという次第でございます。
最初は境界層の理論の粘着条件のことを言われているのかと思いま
したが、結局は粘着はずばり空気が翼に対して接着作用を持つという
ことに落ち着いたようです。
329長文スマソ:2001/07/14(土) 03:06 ID:???
>>327
君のいつものワンパターンのレス、何とかならん?
貴重な意見をカキコしてくれているかも知れないのに。
弾性派さんも、揚力の原因をわかりやすく、詳しくまとめて
からのカキコキボンヌ。
330ご冗談でしょう?名無しさん:2001/07/14(土) 04:02 ID:???
>>揚力の本質を見失う
本質はもともと見えず、文字通り空理空論。
そんなことは最初から見えている(藁)
331ご冗談でしょう?名無しさん:2001/07/14(土) 04:35 ID:???
>>328
このスレの流れを見返してみたけど、27派(と思しき人々)
は粘性の事を粘着性・接着性と言ってないように見えるが。
むしろ、あなたが始めて>>315で使い始めて、強引に結論づけ
ているように見えるが如何?

個人的には>>324氏の質問の回答が気になる。
ちなみに、私も中立派。
332弾性衝突派:2001/07/14(土) 10:23 ID:amNizptc
判りやすくまとめます。
空気分子の持ってる熱運動+群速度による、
空気分子-空気分子間の弾性衝突と空気ー翼間の弾性衝突
が揚力の全てだと思います。
333すんませんm(__)m:2001/07/14(土) 14:03 ID:WGG8ja7w
あの〜マジで物理とは縁遠い素人のオヤジなんですが・・・。飛行機が地上近くで加速して浮力を得て飛び立つのはまだ解るのですが、何故空気の薄い(浮力の得られない)上空で飛べるのでしょう?昔から不思議で・・・うちの奥さんは飛べると思うから飛べるんだと言ってますが・・・。
334 :2001/07/14(土) 14:07 ID:amNizptc
空気が薄いと言っても半分くらいでしょ?
スピード上げれば大丈夫。
335弾性衝突派 :2001/07/14(土) 14:14 ID:amNizptc
>結局は粘着はずばり空気が翼に対して接着作用を持つという
>ことに落ち着いたようです。

この理論だと飛行機は空を飛べませんねえ。
実験しようとして飛行機を格納庫から出したとたんに、周囲の
空気が翼に集まってきて液体空気となって翼の周りに付着、堆積
してしまうので、重量オーバーでうごけなくなってしまいますねえ。
336ご冗談でしょう?名無しさん:2001/07/14(土) 14:25 ID:cSlbbiBU
う〜ん、>>332,>>335を見ていると、弾性衝突派が出てくる度に
同じ文言で煽ってる奴の気持ちが判ってきた気がする。

>>333
飛行機は浮力(アルキメデスの)を利用してないよ、揚力だよ。
337折衷派:2001/07/14(土) 14:46 ID:k7hgkaN6
取り敢えず左右の対象性から、「流れ」のことは少し忘れて
考える。翼の前縁上側で急に翼が厚くなる。ここを通るとき
に空気が跳ね上げられる。翼の上側の空気は一瞬自由落下運動
を始める。モトの高さにまで落ちた時、翼はすでに通り過ぎて
しまっている。下降気流は跳ね上げられた時の速度が重力に
よって向きを180度変えられただけ。
翼は、その上の空気が跳ね上げられているので、その荷重を
まともには受けない。上側の「気圧」はそれほど変わらなくと
も、翼にはまともにそれが伝わらない。下側の空気はそうでなく
それまで通り翼に作用する。この結果が揚力となる。
338弾性衝突派:2001/07/14(土) 15:59 ID:amNizptc
>>336
>>332はどこにもおかしいところはないよ。
>>335は皮肉だ。

浮力=揚力のつもりで質問してるんだよ。
揚げ足取りはやめよう。

>>337は判りやすいね。
339弾性衝突派:2001/07/14(土) 16:04 ID:amNizptc
んで、「標準理論」では物体表面の流速はどう説明されてるの?
「物体表面の流速が0になる=翼が力を受けている」
がどんな形であれ盛り込まれているの?いないの?
340折衷派:2001/07/14(土) 17:25 ID:k7hgkaN6
境界層については取り敢えず、揚力とは独立したものと見なしている
んじゃないの?存在したとしても、それは翼と一体化したものと見な
せると考えてね。
取り敢えず、翼の表面の流速の翼に沿う成分に関しては、無視って
ところじゃない?垂直な成分のみ0と置くってところか。
341弾性衝突派:2001/07/14(土) 18:06 ID:amNizptc
物体表面はこんな感じで流速が0になるはず。
                 /
                ○    \  
    \    \       ○ 空気分子○  
      ○    ○   /      ○
                        \
●●●●●●●●●●●●●●●●●●●●●●●●●●●●●
●●●●●●●●●●●●●●●●●●●●●●●●物体表面
342折衷派:2001/07/14(土) 19:02 ID:k7hgkaN6
統計力学の分子の概念を無造作に流体力学に持ち込むと混乱するよ!
まだ、統計力学と流体力学(マクロなね)の接続は理論的に完全じゃない。
空気の場合、速度が音速に近づけば確かに、弾性体というか質点の集合体
のように振る舞う部分が出てくるけど、やっぱり分子集団がパチンコ玉の
ように衝突するというイメージからはほど遠い。統計力学的な説明が良いのか
流体力学的な説明が良いのかは、この問題に関してはどちらとも言えないって
ところかな。
>>空気全体の圧力場を翼に延長して
ここらへんが、実は問題の核心部なのかもね。
343弾性衝突派 :2001/07/14(土) 20:12 ID:4KwlnaSA
>統計力学と流体力学(マクロなね)の接続は理論的に完全じゃない。

なるほど。
344ご冗談でしょう?名無しさん:2001/07/14(土) 22:08 ID:???
う〜ん、>>332-343を見ていると、弾性衝突派が出てくる度に
同じ文言で煽ってる奴の気持ちがもっと判ってきた気がする。

>>332 の説に基づくと
 ・翼の上面は揚力に寄与してないってことだね。
 ・空気の流れは翼に沿わずに、翼で反射するような流れに
  なるってことだね。

人に皮肉を垂れられるほどわかっちゃいないみたいだから、
しばらく静観してるが宜しい。
345弾性衝突派:2001/07/14(土) 22:18 ID:4KwlnaSA
>>344
>・翼の上面は揚力に寄与してないってことだね
寄与してると思う。でも主因ではないと思う。

>・空気の流れは翼に沿わずに、翼で反射するような流れに
  なるってことだね。

ミクロで見た場合の翼の表面ではそう。


>人に皮肉を垂れられるほどわかっちゃいないみたいだから、
>しばらく静観してるが宜しい。

判ってる人に聞いているんだけど、物体表面で流速が0になるのは
何故?
346ご冗談でしょう?名無しさん:2001/07/14(土) 22:53 ID:???
>>345
それこそ(この場合は)空気に粘性があるからじゃないの。おま
けに、これがコアンダ力の起源じゃない。ただし、粘性の生じる
メカニズムを分子論的に・・・と言われると俺は知らん。なんか
話しが堂々巡りしてるような気がする・・・。
347長スマ:2001/07/15(日) 00:36 ID:???
あのー、「コアンダ力」ってのは、私が便宜上前レスのみで
使用していた「仮の」力なんで、まだその存在すら明確でない
架空のもの故、注釈無しに使うことは止めたほうが良いと思う
のですがー...
ここではどんな意味で使っているのですか?
私は前レスでは、下降気流を積極的に起こす空気と翼の間に
働く力で、空気から見て外力と解釈される力という意味だった
んですが...
348ご冗談でしょう?名無しさん:2001/07/15(日) 01:08 ID:???
>>346
その通り、うっかりしてたスマソ。
そんないい加減な言葉使った俺ってバカだね。>>346訂正します。
× コアンダ力
○ コアンダ効果

で「長スマ」氏(名前変えたのね)は、物理現象として
実在している「コアンダ効果」が生じるのは何故と考えるの?
ある架空の力を自分で考え出したんだけど、これを使えば
説明できるという事で宜しいのかな。

疑問なんだがごく一般にはコアンダ効果のメカニズムとは
どう説明されてるんだろう。ちょっと調べた範囲では粘性
で説明されてると思う。詳しく知ってる人教えてちょ。
349ご冗談でしょう?名無しさん:2001/07/15(日) 14:59 ID:qNIf6Y6E
コアンダ効果というのは、失速が起こらない為の十分条件って
とこじゃないの?これが起こらないと、失速する可能性が極めて高くなる
って感じ。
翼下面でも同時に強い下降気流が発生すれば、上面に回り込む流れが起こり
にくくなる。(慣性及び粘性)
(だからある程度迎え角が必要。下面下降気流自体は、揚力に
大きく寄与するわけでは無い。音速を越えない場合についてのことだが)

超音速領域では、翼前縁付近に高密度が発生し、その密度が低下しない
まま、翼上に流れ込むという。だからこの場合、下面で発生する下降気
流が重要になってくるみたい。
350ご冗談でしょう?名無しさん:2001/07/16(月) 02:03 ID:/6Jd13f.
>>349
最近の高揚力装置は、翼上面でジェットエンジン由来の高圧空気
を噴射し、コアンダ効果でそれを下向きに曲げ強い揚力を得ることを
原理としているらしい。
もうコアンダ効果は百科事典にも載っているから、それほど極端な
専門用語でも無いか。
でも、何故下向きに曲がるのか?つーか、何故翼が空気の流れを
引っ張るのか(でなきゃ反動を得ることが出来ない)それは不明
というか、どこ探しても出ていない。
誰か知っている人解説キボン
351ご冗談でしょう?名無しさん:2001/07/16(月) 13:32 ID:AyBkIm1M
ちょっと皆さん休憩に入ったみたいなんで、僭越ながら私の理解
で中間まとめをさせて宜しいでしょうか。ベル派(標準派)、27
支持派、折衷・中立派とも派内部でも意見が違う方もいらっしゃる
ようなので難しいのですが... (違ってたらツッコんでください、
多分まとめるだけで内容に賛否両論でるでしょうから難しいです)

ベル派(標準派)
 とにかく翼の存在により翼表面に圧力分布が生ずる。この圧力
 分布を足し合わせる(積分する)と上向きの力となり、それが
 実際の揚力である。
     弱点1. 圧力分布のできる理由を説明できてない。
     弱点2. 圧力・流速測定データ等、資料を提示してない。

27支持派
 翼により空気の流れを下向きに曲げ、その反作用として揚力が
 生じる。 空気流れが下向きに曲がるのは空気の粘性に起因す
 るコアンダ効果である。(とする人が派内に多い)
     弱点1. コアンダ効果の原因をはっきり説明できていない
     弱点2. 流れの下向き成分等の(数値)客観データがない。

折衷派・中間派(すんません、これ良くわからないです)
 1. 圧力分布の総和=下向き空気流れに必要な力 説
    (ベル派と27支持派は同じ事を別の側面から言ってる説)
 2. 圧力分布の総和+下向き空気流れに必要な力=揚力 説
    (ベル派と27支持派の力を合わせたものが実際の揚力説)
     ただし、この説はその比率を明らかにしなければならない?
352ご冗談でしょう?名無しさん:2001/07/16(月) 13:33 ID:???
イカロスモナー
353ご冗談でしょう?名無しさん:2001/07/16(月) 14:01 ID:3EtJsQys
>>351
ベル派は浮力「歩麺デス(なつかしい)の原理」を揚力の特別な
場合と考え、それを根本原因とする人達のことでしょう。多分

非ベル派は、気流の運動量変化が重要だと考える人達のことじゃ
ない?
354ご冗談でしょう?名無しさん:2001/07/16(月) 14:36 ID:???
↑ で、あなたのお考えは?
355弾性衝突派:2001/07/17(火) 00:35 ID:W8CuOdSU
>>351
そりゃ飛行船だ。
356ご冗談でしょう?名無しさん:2001/07/17(火) 01:46 ID:k0uImR8c
>翼により空気の流れを下向きに曲げ
解釈1:

翼の存在により、空気の存在できる場所に制限が出来、さらに粘性や
圧縮性の為、翼の上の空気が*自分で*曲がり、その関係で翼の下
の空気が*自分で*持ち上がり翼に当たるので揚力になる。

解釈2:
理由ははっきりしないが、空気と翼の間に働く粘性(というより
摩擦力といったほうが良いかも)により、相互作用が発生し、
特に翼上の空気と翼上表面の間に引力が働き、それが揚力の大部分を
占める。

前スレ27派の「コアンダ力]の解釈は解釈2に近いと思うが、
27支持派はどう思っているのだろうか?
357弾性衝突派:2001/07/17(火) 02:29 ID:W8CuOdSU
351じゃなくて353だった ゴメン
358ご冗談でしょう?名無しさん:2001/07/17(火) 07:31 ID:???
351ですが、私も「弾性衝突派」さんが出てくる度に同じ文言で
煽ってる方の気持ちが理解できてきたような気がします。

>>358
「コアンダ力」なんていい加減な言葉は使わない方が良いですよ。
長スマ氏から注意されますよ。「コアンダ効果」の間違いですか?
コアンダ効果の解釈は私の理解では以下のようなものだと思います。
(ただし、これでコアンダ効果の説明になっているかは不明、
私としては納得しているのですが。)

空気の粘性により、翼面近くで流速の勾配がつく。これは、空気と
翼面に一種の摩擦力のような力が働いた事になり、空気の流れは
翼面に沿って流れる(=下向きの運動量を与える)

私自身は上面が効いているのか下面が効いているのか判りません。
両方効いているようなイメージを私は持っています。さて、コア
ンダ効果にあまり拘らず、圧力分布を示す等、本質的な話しから
再開しませんか。
359ご冗談でしょう?名無しさん:2001/07/17(火) 09:04 ID:???
航空学科に行け!
360ご冗談でしょう?名無しさん:2001/07/17(火) 14:50 ID:???
太陽電池動力(電気洗濯機程度)
超高空飛行(高度30000mに挑戦)
長時間巡航飛行(数日から数十日、それ以上)
平均速度50km以下

無人飛行機「ヘリオス」の存在も無視しないでね!
361ご冗談でしょう?名無しさん:2001/07/17(火) 17:58 ID:HrUFg.qw
age
362弾性衝突派:2001/07/17(火) 22:28 ID:Xm1n/Fa2
異端者を叩くからスレが下がる下がるw

水中翼で考えて、圧縮、膨張を切り捨てられないかな?
363ご冗談でしょう?名無しさん:2001/07/18(水) 02:04 ID:8NbnVPqE
飛行機の翼は、空気をその中に侵入させないように抵抗するだけ
なニョ?
それとも、積極的に空気を引っ張ったり、押したりする
ニョ?

それが最も本質的な問題だとオモワレ

あ、誤解しないでケレ、
飛行機に動力がついているからといって
それだけで、翼が空気を引っ張れるとか
押せるってこと言えニュァイってことだけは。
364ご冗談でしょう?名無しさん:2001/07/18(水) 10:11 ID:???
>>362
>水中翼で考えて、圧縮、膨張を切り捨てられないかな?

初めて良いこと言ったね、俺もそれ考えていた。このスレ
の頭の方にもあったけど、水中モーターを持った(持たなく
ても良いんだけど)有翼体(飛行機模型)を考えると揚力
発生するよね。27支持派説が説得力あるように見えるけど
どうなんだろ。それともレイノルズ数とか違うから、こんな
単純に考えてはいけないんだろうか?
365ご冗談でしょう?名無しさん:2001/07/18(水) 14:00 ID:7GA8L8qI
水だって圧縮膨張するでしょ。それなりに。
まず27支持派と反27支持派(標準派?)の違いというか争点を
はっきりさせて欲しい。でないとワケわかんなーい!
>>356が指摘している解釈の相違点を27支持派かそうでないかを分ける
点として良いのかな?
366ご冗談でしょう?名無しさん:2001/07/19(木) 02:02 ID:C5JnqpSM
ついにこのスレも元スレ27派の圧勝で終わったようだ。
=========================終了==================
お疲れさん。
367ご冗談でしょう?名無しさん:2001/07/19(木) 05:45 ID:Wp5QS22U
>>366
ハぁ?
368弾性衝突派:2001/07/20(金) 01:36 ID:OAIJB0mQ
深度77。
こーんな下の方に埋もれてるのをハッケソ。
>>366は数式にまとめろや。
369ご冗談でしょう?名無しさん:2001/07/20(金) 03:19 ID:LbVQNZHk
まんこまんこまんこまんこーけーこっこ
370ご冗談でしょう?名無しさん :2001/07/20(金) 05:43 ID:???
>>368
うわっ、またコイツわけわかんない事を書きに
来やがった!嵐でやってんのかよ。でも強烈な個性
でスレに華を添える名脇役としては貴重な存在かも。

ボコボコにされて退場きぼんぬ。
371弾性衝突派:2001/07/20(金) 14:09 ID:bkuHbh1M
>>370
お前も結論出せや。
372ご冗談でしょう?名無しさん:2001/07/20(金) 14:54 ID:???
結論:「弾性衝突派」はウザイ、邪魔、迷惑。
373弾性衝突派:2001/07/20(金) 18:10 ID:ofsa1TAs
結論:「粘性下降気流反動派」はウザイ、邪魔、迷惑。
374ご冗談でしょう?名無しさん:2001/07/20(金) 22:59 ID:???
同じ27支持派(なのかな?)内部でモメないように。
>>365の続き(結局 >>351 から?)から続ければどう?
375ご冗談でしょう?名無しさん:2001/07/21(土) 00:01 ID:0gDhUtHI
みなさん、まんこをなめなめしてますか??
376ご冗談でしょう?名無しさん:2001/07/21(土) 00:09 ID:???
>>374

>>362,>>364あたりの水中翼の話しからでも良いんじゃない。
377ご冗談でしょう?名無しさん:2001/07/21(土) 00:29 ID:???
一口に水中翼と言っても、水面が波立つような浅い
(例えば水中翼船の翼)奴から潜水艦のような深く、
水面が波立たないものまで色々ある。
水面の波立ちの原因は、翼の前後で発生する上昇、下降水流
の影響でしょうね。
深く沈んでいる場合と、浅い場合では、水が得る運動量に
大きな違いが発生している感じがしない?
378ご冗談でしょう?名無しさん:2001/07/21(土) 00:38 ID:???
>>377
浅い場合だといろいろ面倒な条件がついてくるから、
簡単に、深く沈んでいる場合を考えることでどうでしょ。
379ご冗談でしょう?名無しさん:2001/07/21(土) 01:42 ID:leTNrkXo
圧縮性の話であれば空気でもM<0.3位までの範囲であれば無視してもほとんど
誤差出ないんじゃ無かったっけ?
380ご冗談でしょう?名無しさん:2001/07/21(土) 02:00 ID:???
今前スレの27が示してたサイト見に行ったけど別にベルヌーイの定理
否定してるわけじゃないような気がするんだが…
単に同時到着理論の間違いを指摘してるだけじゃない?
ベルヌーイの定理で十分だと思うんだが。
381ご冗談でしょう?名無しさん:2001/07/21(土) 06:41 ID:XFyy6hEw
ベルヌーイの定理を直接使えるかどうかはわからないが、
圧力(内部ポテンシャルも含められるか?)と流速の間に
一定の制限(特に圧力が下がらなければ、流線上では速さを
獲得出来ない関係等)は成り立つだろうね。それを利用すれば
そう悪い説明でも無い気がする。
が、欠点もある。翼の周りの気流は定常状態には普通ならない。
(だからこそ乱流うんぬんの話と絡む)
時間的な平均をとれば、ある程度ベルヌーイlikeな定理が成り立つ
んだろうけど、問題はそれで飛行機が落下しないことを説明出来るか
だね。
水の中の翼が浅い水深のもとで、揚力と重力がつりあった状態で
水平運動している場合、翼はその上の水を跳ね上げつつ、それによ
って真上の水面が「下がる]ことによって、大きな「浮力]を得てい
ると見なせる。(アルキメデスの原理のように、翼の体積分の水
の重量=浮力となっていない。この場合)
揚力ってのは、こういう状況が本質なんじゃないかな?と私は
思う。多分、これが飛行機の発明の基本的アイデアじゃないかな?
382弾性衝突派 :2001/07/21(土) 18:46 ID:r5HJ7AF2
翼の下面の水流は翼がどんどん下がって来るから押し出されて下に方向を変える。
翼の上面の水流は翼がどんどん下がって来ゆくから真空を作るまいとして下に
方向を変える。

んー判りやすい。
383弾性衝突派:2001/07/21(土) 18:47 ID:r5HJ7AF2
来ゆく>ゆく
384竹内 美継:2001/07/21(土) 19:10 ID:naU6wtG6
「膜」があるから飛行機は空を飛べるのです。
「膜」があるから地球はまるいのです。
「膜」があるから満潮・干潮という現象があるのです。
「膜」があるから月は地球の周りをまわっているのです。
「膜」があるからあらゆる自然現象があるのです。
385ご冗談でしょう?名無しさん:2001/07/21(土) 21:33 ID:HIKoxaag
おもしろそうな話しで復活したのかな?

>>381
「水の中の翼が・・・」以降がちょっとイメージ掴めな
いんですが、もう少し詳しく教えていただけませんか?
翼前面でアップウォッシュが起きて、翼中央から後端に
かけての上方でダウンウォッシュが起きるという意味と
は違うのですか?「水面」を考える必然性もお願いします。

>>382
「翼が下がる」の意味が判らないのですが??
言いたいニュアンスは判ります。「翼下面では水流が
翼面に沿って下向きに曲げられる。翼上面では、一旦
跳ね上げられた水流が下降流となり、これも下向きに
曲げられることになる」という事ではないですか。翼
上面の下降水流の原因は、翼上面の水圧低下の補償,水
の粘性によると思いますが、これは議論の余地があると
思います。
386弾性衝突派  :2001/07/21(土) 21:58 ID:y/oYdsx.
ああ、止まってる水の方から見ると翼が下がってくるということです。
粘性は含まれてません。
>翼下面では水流が
>翼面に沿って下向きに曲げられる。翼上面では、一旦
>跳ね上げられた水流が下降流となり、これも下向きに
>曲げられることになる
はその通りの意味です。

翼から回転翼に話が飛びますが、
スクリューを高速回転しすぎると真空ができてある回転数以上では
効率が落ちるそうです。
387ご冗談でしょう?名無しさん:2001/07/21(土) 22:04 ID:???
ボートのオールは漕ぐ時に、その真上の水を跳ね飛ばす
すると、オールはその下の水面が平らになるまで圧力を
下から受けます。オールは漕いでいる時、沈めようとしても
なかなか沈まず、跳ね上げられる力を受けます。
そういうことを381は言いたいのではないでしょうか?
オールが完全に水面の下にある場合でも、極端にその力は
落ちません。(むしろ経験上はそれは強まる。)
388381:2001/07/22(日) 05:22 ID:kYpL6mng
>>387
はぁー?
アナタ、ボート漕いだことあるの?
逆のこと言っている感じ。
でも、普通なら水に沈んでしまうモノも、水面近くの水を剥がすように
前面のエッジで跳ね上げると、結果的にそのモノの上には水が無くなり
下から水圧しか受けなくなり沈まなくなるよね。私はそのことを言った
までなのだが。
389竹内 美継:2001/07/22(日) 07:14 ID:LfriZUmc
* あらゆる物質の階層毎に「膜」がある。

*地球という階層の「膜」と水分子という階層の「膜」と水の集合としての「膜」(水面)とを総合して考えれば「物」の性質というものが説明できるのではないかと考えています。

*「量」が「質」をつくっているようですが、「質」と「量」はちがうと思います。
390ご冗談でしょう?名無しさん:2001/07/22(日) 08:02 ID:???
よそでやってください>竹内 美継
391押しのけ吸い込み理論派:2001/07/22(日) 16:12 ID:Mbf1ofXs
こっちの方が簡単だから転向します。
392ご冗談でしょう?名無しさん:2001/07/22(日) 20:46 ID:???
↑わかってると思うけど27支持派って事だよ。
393押しのけ吸い込み理論派:2001/07/22(日) 21:10 ID:Mbf1ofXs
そうだったのか!
394ご冗談でしょう?名無しさん:2001/07/22(日) 21:43 ID:moCmAseM
27支持派って...
>>27のレスってそんなに意味深なの?
395ご冗談でしょう?名無しさん:2001/07/22(日) 21:44 ID:???
27って、まだ生きてたの?
396押しのけ吸い込み理論派:2001/07/22(日) 21:50 ID:Mbf1ofXs
前スレ27だよん。見られないけど。
397ご冗談でしょう?名無しさん:2001/07/22(日) 22:11 ID:???
392だけど、ゴメソ、先走った。

「押しのけ吸い込み」って>>285の下半分に書いてある事で良い
よね。多分これはベル派(標準派)、前スレ27支持派ともに合意
事項だと思うんだよね。(違ったらスマソ。)

で、こういう事が起きた結果として翼上下面にできる圧力分布の
総和が揚力であるとするのがベル派(標準派)で、これによる
下向き流れの反力が揚力であるとするのが前スレ27支持派だと
思うんだけど。
398ご冗談でしょう?名無しさん:2001/07/22(日) 22:13 ID:???
何度もスマソ。

× >>285の下半分
○ >>385の下半分
399押しのけ吸い込み理論派:2001/07/22(日) 23:07 ID:RwP7pqsY
いや、圧力など考えずに、翼が通りすぎた体積が下に移動すると
考えるの
400ご冗談でしょう?名無しさん:2001/07/22(日) 23:51 ID:???
>>399
この論理性のない書き方といい、コテハン多用と
いい、君は NOTORIOUS 「弾性衝突派」か?

相変わらず翼上面を考えてないが、下向き流れを作り
出す反力が揚力の本質であるという点においては広い
意味で27支持派に含まれると思う。

(27支持派の大多数からは嫌がられてると思うけどね)
401 :2001/07/23(月) 01:37 ID:0RKSgkWg
>>400
翼上面も含めてるよ。
402ご冗談でしょう?名無しさん:2001/07/24(火) 00:01 ID:QJoIsBhM
age
403ご冗談でしょう?名無しさん:2001/07/24(火) 01:40 ID:1MqKr6dM
翼上面上の負圧も重要とゆうことか?
404押しのけ吸い込み理論派   :2001/07/24(火) 09:24 ID:gEyOYtSw
翼上面も膨張を割愛して、できた空間に水が押し寄せると考えた方が簡単だ。
405ヘリ屋:2001/07/24(火) 09:34 ID:.vu98maI
おじゃまっす。へりの世界じゃ、「揚力」を説明すんのに、ブレードエレメン
トセオリと、モメンタムセオリーの二つで語る場合が多いです。ブレードエレ
メントセオリーからじゃ、例のベルヌイの定理を使いますが、モメンタムセオ
リーだと、ニュートンの「フォースはアクセレーションにマスを掛けたもん」
で語りますから、揚力(フォース)はマス(ローターデイスク)がアクセレー
ション(ダウンウオッシュ)をつくりゃ、勝手に生まれるモンとされるです。
こうするとヘリはダウンウォッシュで飛んでるんだと勘違いするやつが出ます
が、ダウンウォッシュはインデイユースドベロシテイー(これが増えるとイン
デユースドドラグが増えるから、エンジンがもっとお仕事しないとローター回
転が落ちる)の化身だから、ダウンウオッシュは「フォース=揚力」を生むと
きのうんこであります。
406ご冗談でしょう?名無しさん:2001/07/24(火) 14:28 ID:???
>>405
固定翼も回転翼も基本は同じだと思うんだけど、読みにくい
書き方のせいもあって最後の4行の意味がいまひとつ理解で
きない。おもしろそうな話しなので、最後の4行を言い換え
るか何かして、もう少し判りやすく書いてもらえないか?
407ご冗談でしょう?名無しさん:2001/07/24(火) 18:04 ID:aEvBD10w
要はダウンウォッシュは派生現象であり、ヘリの場合はうざい存在
だということじゃない?
408ご冗談でしょう?名無しさん:2001/07/24(火) 21:12 ID:???
>>407
いやそれは分かっているんだけどさ、その説明だよ。
409ご冗談でしょう?名無しさん:2001/07/25(水) 04:06 ID:n1fI0GXo
空気は翼の荷重を伝える媒介物に過ぎないのでしょうか?
(荷重は地球表面が分散して支える)

それとも、運動エネルギーを与えられて、その反動を翼に与えている
のでしょうか?
どちらも重力に抗する力を与えられるでしょう。十分に丈夫な翼と十分に
強い水力があればの話ですが。
410ご冗談でしょう?名無しさん:2001/07/25(水) 04:07 ID:n1fI0GXo
ゴメソ
水力->推力ね。(鬱氏)
411ヘリ屋:2001/07/25(水) 07:23 ID:bbkf2kTw
ダウンウォッシですか? ダウンウオッシュはインデユースドドラ
グにプロポーショナルです。エアフォイル自体のドラグは、ヒコー
キの翼もヘリのブレードでも、プロファイルとインデユースドドラ
グです。プロファイル自体はマッハがらみの速度やストールが発生
しない限り、速度の増加に応じてのドラマテイックな増加はありま
せん。インデユースドドラグは、リフトやスラストが増えますと、
それにプロポーショナルに増加します。だから、プロファイルは役
立たずのドラグで、インデユースドドラグは揚力やスラストを生み
出したときのウンコなんで、お役に立った後のドラグです。この理
屈はヒコーキだって同じですから、地面効果の説明は「地面にダウ
ンウオッシュが跳ねかえる=ダウンウォッシュベロシテイーがフル
にデベロップしない=インデユースドドラグが減る=同一ベロシテ
イー時の揚力が増える」です。
412ご冗談でしょう?名無しさん:2001/07/25(水) 08:04 ID:???
カタカナ読みにくい>>ヘリや
413ご冗談でしょう?名無しさん:2001/07/25(水) 10:33 ID:???
>>411
406です。わざとだと思うけどカタカナの大文字小文字
の区別無く書いていただきありがとうございます。
非常に読みにくかったのですが、大体は理解しました。
(補助資料)
http://www.aero.kyushu-u.ac.jp/birdman/01/What/Theory/flight.htm

この場合のダウンウォッシュ(=誘導速度:Induced
Velocity)と27派の参考文献におけるダウンウォッシュ
って同じモノを指しているんですかね。27派のダウン
ウォッシュは誘導抵抗(Induced Drag)渦に起因してない
と思いますので何か違うような気がするんですが・・・。
どなたか教えていただけませんか?
http://www.allstar.fiu.edu/aero/airflylvl3.htm

で、結局ヘリ屋さんの意見はいわゆるベル派(標準派)
ということなんですよね。
414ご冗談でしょう?名無しさん:2001/07/25(水) 23:48 ID:dj4QYD82
ageman
415ヘリ屋:2001/07/26(木) 05:52 ID:vmfDamRA
カタカナ多くてやっぱりウザかったですね。ごめん。ところで、自分自身は
ベル派でも27派でもありません。ヘリの世界じゃ神様になってるプルーテイー
という先生の意見では、モメンタムセオリー(27派)もブレードエレメントセ
オリー(飛行機なら「翼」でベル派)の二つは、同一の事象を異なるスタート
点から探る方法論であり、結末はおんなじとされてます。

たとえば、ある一定の対気速度で一定の降下率で降下中のヘリのリフトとウ
ェイトは完全に均衡(飛行機だって同じ)してますが、これをどう説明するか
です。

ベル派の説明:
エンジン出力を弱めると(ブレードのピッチ角を下げる)降下をするが、同
一の対気速度であっても翼型へのベロシテイーは下方から来るので、ここに迎
角の増加という「加速」が発生し、ベルヌイの定理で追加揚力が発生し、その
うちウエイトとリフトが均衡する。だから降下率は一定。

27派の説明:
エンジン出力を弱めると(ブレードのピッチ角を下げる)降下をするが、降下
すること自体は高度エネルギーとして機体に蓄えられている「ポテンシャルエ
ナジー(高度X重量)」を消費するので、これがブレードを廻すエネルギーに
変換される。だから降下中は少ないエンジン出力でブレードの回転を維持でき
、降下率はそのときブレードの回転に必要な総ドラグと拮抗した分の「ポテン
シャルエナジー」分で落ち付く。だから降下率は一定。

ところで27派の根拠になっているHPでのお話ですが、ここでのモメンタムを
リフトの根拠にされている部分は、内容的には405で発言させていただいたニ
ュートンのお話とおんなじです。

「B747の翼型はもっと分厚くないと」って部分ももっともですが、ベル派的
に考えれば、例の第2次世界大戦時に数学的に解明されてたっていう、「円筒
理論」、つまりウイングスパン(ヘリならデイスクの直径)分の円筒のなかの
モルセルが影響を受けてるってやつに反します。つまり翼型なんか前面投影で
考えれば(ヘリならデイスク)剃刀みたいに薄いけど、それでも翼の遥か上方
のモルセルにも影響を与えるからこそリフトが出るって話です。これはリフト
その物よりも、「パワーカーブ」に出てくる「インデユースドドラグ」に影響
します。インデユースドドラグは対気速度の増加に伴って減っていきます。な
ぜならば、単位時間当たりに翼型を通過する空気量が増加し,これによって単
位時間当たりの通過空気量の中に占めるドラグ量の割合が減るからですが、こ
こでの空気量は「円筒」分です。前面投影だけではこれは成り立ちません。だ
からここはベル派が好き。

ところで、所定の対気速度を維持した大型飛行機が最終進入にいる場合、その
ときの降下率が所定レートよりも大きい場合は、ゴーアラウンドしなくてはな
りません。なぜなら、着地寸前にちょっと機首を上げただけで思いっきり浮き
上がってしまうからです。でもこれはベル派じゃ説明できない。だってここで
のアクセレーションは、ほんのちょっとの機首上げに伴うベロシテイーの変化
だけで、とてもじゃないけど大型機をバルーニングさせる説明にはならないで
しょ。ここはやっぱ27派ですが、ここでの鍵は「ポテンシャルエナジー」と「
フォアワードモメンタム」の合成モメンタムです。

ところで27派のHPのダウンウオッシュはインデユースドドラグでありインデ
ユースドエアフローとおんなじです。だから27派は迎え角の増加はリフト増と
ダウンウオッシュ増をもたらすけど、それに打ち勝つエンジンパワーが必要と
言ってますよね。ところでダウンウオッシュは、フルデベロップドインデユー
スドエアフローと言ったほうが良いかもしれません。

長文+カタカナ、ごめん。
416ご冗談でしょう?名無しさん:2001/07/26(木) 14:54 ID:???
27派?の揚力の原因:空気と翼の摩擦力、及び空気の粘性
非27派?の揚力の原因:空気の密度と速度の間の制約関係
(圧力がそれを媒介する為、圧力差が生まれる)
非27派?の説明は、今一つ納得出来ない要素がたくさんあるが
27派?の説明は、空気と翼の間の摩擦力(粘着力とか粘着条件
の原因となるもの)という揚力と直交する成分の力がどうして
揚力に結びつくのかということの説明が無いというより、わけ
わかんない。この部分の説明がきちんと出来ないと、27派?
の説明は非27派よりタチが悪い。
417ご冗談でしょう?名無しさん:2001/07/26(木) 15:03 ID:GEPaoJg.
>>415
降下時には高揚力装置(フラップ)を全開しているからってことも
あるんじゃないの?つまり通常時(巡航時)とは翼の形状が違う
418ヘリ屋:2001/07/27(金) 01:35 ID:iu1lCnvU
結局、http://www.allstar.fiu.edu/aero/airflylvl3.htmのオヤジが言ってる
ことは、単にリフトやスラストをニュートンの第2で言ってるだけだと思いま
すよ。これは全く目新しいもんじゃありません。ニュートンの第2を航空ネタ
にするときは、「フォースはマスとアクセレーションを掛けたもん」とします
が、フォースをリフトにするんなら、マスは翼面積(ヘリならデイスク面積)
で、アクセレーションはダウンウォッシュです。もうちょっと細かく言えば、
「マス」はMass Flowであり、翼またはデイスクを「ポンプ」のよう
に通過して行く、毎秒当たりの空気量。アクセレーションは、翼またはデイス
クのかなり上面でSteady Vaueのとこから、翼またはデイスクのか
なり下面でこれまたSteady Vaueの所に向けてのベロシテイーの変
化。ところで「ベロシテイ」って言葉は、空気速度だけでなく、角度の概念が
必要なんで、ゼロヨンで疾走する車に「ベロシテイー」なんて言葉を使うと笑
われモンですよね。だって車は真っ直ぐ走ってるから。だから「ベロシテイー
」って言葉上の「アクセレーション」の意味は、単に風の速度の変化だけでな
く、角度の変化も含まれるわけですが、この角度の変化にコアンダさんが登場
しているわけです。
419アンチ文部科学省:2001/07/27(金) 01:56 ID:DR6HBT9I
正しい社会科教科書を作る方法については
http://www.geocities.co.jp/Technopolis-Mars/3422/mat.htm
を見てください。
420ご冗談でしょう?名無しさん:2001/07/27(金) 04:15 ID:pESBHV3E
つーか、前スレの27こと、http://www.allstar.fiu.edu/aero/airflylvl3.htmの
「おっさん」の言いたいことは要するに、
揚力の計算をベルヌーイの定理と速度差理論だけで計算すると、一見説明出来た
ように見えるが、上面下面の速度差の説明が(圧力差の説明と循環するので)明確
じゃないから問題だということらしい。速度差を認めて、それを使って揚力を計算
すると、(それはクッタージューコフスキーの定理で計算することと同じであること
は間違いない。クッタージューコフスキーはそれを洗練された形で表現したという
こと)、もっともらしい解答が出てくるが、実は循環論法だというのが論点。
確かにそれは言えてる。
速度差があるから、圧力差が出るのか。それとも圧力差が出るから速度差が起こる
のか。鶏と卵の関係に似ている。それには同意。
ただ、粘性が無い空気の流れは、いずれは、揚力も抗力も及ぼさなくなるだろう
という流体科学の基本法則を逆手に取って、粘性(要するに、流体内部の摩擦力
流体とその周囲との摩擦力)があるから、揚力が起こるんだ!という「裏」を
主張している感じだが、これは非論理的な推論だと思われ。
「AならばB」が正しくても、「Aでないなら、Bでない」という主張は間違い。
「アメリカ人なら英語をしゃべれる」という仮定が正しいとしよう。
「アメリカ人でなければ、英語はしゃべれない」という主張が正しいなら
(「おっさん」はそう言いそう」)英国がしゃべれる人は皆アメリカ人で
なければならないことになる。
421ご冗談でしょう?名無しさん:2001/07/27(金) 04:34 ID:???
self-consistentにうまく収拾つけれられないのか
422ご冗談でしょう?名無しさん:2001/07/27(金) 05:37 ID:???
>>415,>>418

406(=413)です。Momentum理論が27派と等価という事は理解
しました。すると、やはり、翼端渦に起因するダウンウォッ
シュ(=誘導速度:Induced Velocity)と27派の参考文献に
おけるダウンウォッシュは別物ですね。

誘導速度は揚力を生み出すのに有効な迎角を小さくする働きが
あるのに対し、27派のダウンウォッシュ(Full-developed
Airflowと言うのでしょうか、流体力学を勉強していないので
よく判りません)は、その反力で揚力そのものを生み出す訳
ですね。前者は2次元翼では考慮されないのに対し、後者は2
次元翼でも考慮されるのだと思います。

ベル派と27派が同じ事を別の側面から言っているだけという
のもうなづける気がします。(>>351の中間派になります。)
翼によって空気流れが下方に曲げられる
→ その際に翼周りに流速分布・圧力分布ができる。
→ その圧力分布の総和が揚力(正確には揚力+抗力)である。
→ ただし、その揚力は空気流れが曲げられた反作用と見なせる。

ちょっと27派の肩を持つ発言をすると、前スレ27氏にしろ、
参考リンク先にしろ自分達がこの説の考案者だとは言っていない
と思います。また、これは私個人の捉え方なのですが、揚力の
「本質」は何?という質問の答えは「空気を下向きに曲げる反
作用」という方がしっくりきます。(ベル派否定ではありません)
423ヘリ屋:2001/07/27(金) 07:14 ID:iu1lCnvU
417でのご意見は、415で書きこませていただいた「地表付近で機首
を上げると、降下率の高い航空機ほどバルーニングする」に関して
のご意見だと思います。たしかにフラップも理由だと思いますが、
まったく同じ現象がヘリでも起きるんです。ヘリでエンジン停止時
には、「オートローテーション」で降りてくるのですが、同一の対
気速度で同一の機体であれば、降下率の大きい場合の方が、地表付
近で機首上げをするとバルーニングするんです。この手の説明は、
やっぱモメンタムセオリーじゃないと出来ないと思います。
424ご冗談でしょう?名無しさん:2001/07/27(金) 14:03 ID:IhFu5esY
まだやってたのか・・・ちょっとちがった切り口で。
@物体(空気)
A物体(翼)
☆力(相互作用)
1. →@ A静止
2.  @☆A接触
3.  @← →A

2.の☆A部分に着目して衝撃力というか作用力(応力(圧力))を積分し、
その結果として3.の→Aを求めるのがベル派的な現象の見方。
3.で@← →Aを運動量保存の観点から→Aを求めるのが、前スレ27氏
的なものの見方。
で、1.2.3.の各段階で各物理量(速度、圧力)のふるまいの法則を
数式で表現したのが、ナビエストークスの方程式&連続の式。
さらに、実際の流体の場合は1.2.3の各段階は無段階的にというか
連続に同時に起こっているので、「分離して考えることはできない」と
思う。ベル的見方も27的見方も現象の一側面は捉えているが、
それが全てではないと思われる。
似たQ&Aに、スウィングバイでなぜ衛星の軌道が変わるのか?
A1.重力が働いているから。
A2.星と衛星の運動量が保存されから。
というのがあると思う。A1,A2どちらが正しい??
425ご冗談でしょう?名無しさん:2001/07/27(金) 17:24 ID:qWrNU8sg
>>424
「ちょっと違った切り口で...」「ベル的見方も27的
見方も現象の一側面は捉えているが、それが全てでは
ないと思われる。」 と言っているが、君は今までに
スレ1or2においてその旨主張したのか?それとも
話しの流れを見て出てきただけか?前者なら君のその
旨の発言内容(発言番号)を教えてくれないか?

前者なら大したもんだと評価するが、後者なら「何を
今頃と」負の評価しかできない。

また、スウィングバイについては、物事の一面しか見
てない例を挙げたいようだが、スウィングバイで衛星
の軌道しか変わらないような書き方を意図的にしてい
るのか?あまりうまい例とは思えないが。
426再度整理:2001/07/28(土) 04:09 ID:ef41XoT.
今一度整理

ベル派(標準派?)
揚力の主体は、翼の上下に発生する圧力差である。
圧力差の存在は、上下の気流の速度差(観測値)とベルヌーイの定理を用いて
説明出来る。完全流体近似で翼との粘着条件無しで、しかも翼が2次元とみな
せる位十分長ければ、複素関数論で系をまとめることが出来、クッタージューコ フスキーの定理のようなエレガントな数学理論が使える。(クッタージューコフス キーの定理はベルヌーイの定理と深い繋がりがある。)
速度差を上下の翼の弦の長さの相違で説明する人も居る。
流体の一般論で障害物がある流れでは、次第にその障害物に仕事をせずに
流れるであろうという予想もしくは経験的事実がある。(ダランベールの
パラドックス)これによると、完全流体(非粘性圧縮流体も実はそう)と空気を
みなした場合、最初は揚力や抗力があっても次第に失われるであろうと
予想されるが、現実の翼は揚力が失われない大きな理由に粘性(内部摩擦)
と翼と空気の境界における摩擦を挙げる人が多い。

運動量派(前スレ27派)
揚力の主体は、気流の運動量変化である。気流が下面で翼にぶつかる場合に
発生する応力の存在はわかりやすいが、その他に下面を通った気流が、翼
後ろで、その上の空気を粘性で下向きに引き込むことも考えられる。
揚力は、これら翼周りに発生する空気運動量の変化で説明され、単位時間
に受ける揚力の力積と、これらの気流の運動量変化の和は0に等しい。
(つまり反動)
それだけでなく、翼と空気の境界を強制的に真空にしないように、翼と空気
の間に束縛力が働き、「コアンダ効果(非剥離流の発生)」が生まれ、さらに
翼上方の空気の粘性による集団同調性により、束縛力が増幅され、大きな
上向引力を及ぼすからだとする意見もある。(特に前スレ27氏)
その根拠として、ベル派(標準派)でも用いられる粘着条件を挙げている。

折衷派:
揚力の主体は、何らかの原因で発生した、上下の気圧差(動的浮力)を
翼の形状で特に上面の形状によって、主に上面の圧力増加に必要な
気流の流入が妨げられる為に、維持される点にある。
下面の迎え角により発生する下降気流は、上面への気流の回り込み
を防ぐだけでなく、動圧上昇をもたらし、揚力に寄与する。
下降気流自体は、揚力の反動の一部が運動エネルギーに化けたもの
であって、それから直接揚力の大きさを推定することは出来ない。
下面の圧力増加は、大気中を音速(超低周波)で伝播し、広く地表面に
応力を与える。つまり、翼にかかる荷重の大部分は大地が支え、空気の
運動量変化になるものは一部である。
427ご冗談でしょう?名無しさん:2001/07/28(土) 05:15 ID:???
未だに前スレが読めない状況にあるのが原因なのか、最近の論点はフォーカス
がボケてるなぁ。前スレの最後の方では
 「揚力の原因は翼で発生させた下降気流の反作用で、27派は正しい」
 「揚力を‘計算’する時のベル派の方法は近似的に正しい」
ということになっていたはず。だから揚力の「現象を記述する」上では
「27派 vs ベル派」なんて構図はない。

今問題なのは
 「ベル派の‘圧力差’では揚力を理解したことにはならない」
でしょ。実際に翼の周りの圧力分布は計算できるけど、何故そうなるのかは
まだ誰も答えていない。だから今のところ「飛行機は何故飛ぶのか」という
疑問に対して「圧力差があるから」と言っても、その原因が不明なのだから
回答にはなっていない。

一方、27派の説明だとコンシステントに揚力を理解することができる。
静止している大気中で飛行機を前進させて下降気流を発生させれば、
大気の運動量を下向きに変化させた力積の反作用として、翼は揚力を受ける。
この揚力が機体にかかる重力とつり合えば飛行機は大気中に「浮いて」いられる。
もちろん翼に発生するのは下降気流ばかりじゃなくて、いろいろな流れが
生じるから「抗力」も受けるけど、揚力を理解するためには考えなくても
オッケーでしょ。

27派で理解しにくいのは翼の上面で生じる下降気流かな。これは空気の
分子間に作用する引力(Van der Waals力)で定性的に理解できるので、
敢えて「粘性」という言葉を持ち出す必要は無いね。
428押しのけ吸い込み理論派:2001/07/28(土) 08:22 ID:dHCzUwYY
翼の上面の表面には擬似真空が出来るのです。
それが水を引き付けるのです。
429ご冗談でしょう?名無しさん:2001/07/28(土) 10:26 ID:rDTtJN6Y
>>426
文章には細かいツッコミどころ満載なんだが、それよりも
何よりも>>422,>>424のような「同じ現象を別の側面から
見てる派」(中間派とでもしましょうか)を落としてない
かい?ところで「粘着」という言葉を使ってきているのは
今まで約1名しかいないと思うが、あなたはその方?

>>427
「ベル派の‘圧力差’では揚力を理解したことにはなら
ない」は言いたい事はわかるけど、圧力差のできる原因
というのは翼上前面で跳ね上げられた流れが翼上面後部
でまた翼に沿おうとするから(下面は割愛)で定性的に
は説明できると思うけど、これではダメなの?こういう
流れが起きるから上面負圧で下面正圧になるって事。これ
って、あなたが最後の段落で言っている現象そのものだと
思うけど。(もちろんあなたが示した(?)参考リンク先
にも書いてあるよね。)下降流の反力が揚力であるという
説明は賛成。Van der Waals力での説明が適切か否かは
疑問が残ると思う。個人的には「粘性」を推したい。

>>428
もうそろそろ水は終わっても良いんでないかい?っつうか
あんたのそれ理論じゃないんだよ。「押野家吸込」てのは
ベル派(標準派)にしても「27派」にしても現象として
認知してるんだからさ。その現象によりなぜ揚力が発生す
るかの説明があんたには無いんじゃない?
430427:2001/07/28(土) 10:47 ID:???
>>428
擬似真空って低圧部分のことかい?
低圧部が流体を引き付けるという話は、その低圧部に向かって
いろいろな方向から流体が流れ込むから、流れのかたよった下降流
が生じることを説明できないって、前スレで何度か出てたぜ。

ところで、翼上面の下降流を水で考えるのはグッドだな。空気の
分子間に働く引力よりもずっと強い引力が水分子間に働いている
ので(水分子の自発的な分極に由来)、コアンダ効果と合わせて
 「揚力=流体の運動量を変化させた力積の反作用」
ということが、さらに理解しやすくなる。
ここでも敢えて「粘性」を使うのは止めとくけどな(藁

さあ、ベル派は早く圧力分布の生じる原因を考えてね。
(このスレの前の方にヒントがちょこっと書いてあるよ)
431427:2001/07/28(土) 10:57 ID:???
>>429
>圧力差のできる原因
>というのは翼上前面で跳ね上げられた流れが翼上面後部
>でまた翼に沿おうとするから

これで翼の上面の静圧が低くなる理由をもう少し詳しく書いて。
それがちゃんとしていれば、圧力差で揚力を理解できたことになるよ。
でも、それはきっとベル派の説明じゃないね。
432ご冗談でしょう?名無しさん:2001/07/28(土) 11:29 ID:rDTtJN6Y
>>431
速度と圧力の関係の事?
まあいいや。俺は元27支持派(というより揚力は流れ曲
げの反力派)で現在中間派だから、説明はベル派(標準派)
に任せる。

あまり派閥にわけるのは適切じゃないかも知れないけど、
あなたも(登場初期はちょっと違ってた気もするけど)
中間派じゃないの?揚力の'本質'が下向きに曲げられ
た流れの反力であることを(あるいは反力で考えた方が
理解し易い事を)盲目的なベル派にケーモーするために
敢えてベル派(標準派)を敵にまわしてるんでしょ。
まあいいや、こっちも後はベル派(標準派)に任せた。
433ご冗談でしょう?名無しさん:2001/07/28(土) 12:16 ID:???
過去ログ(前スレ)がhtml化されたみたい。
読み返したい方は>>1のリンクからどうぞ。
434押しのけ吸い込み理論派:2001/07/28(土) 13:46 ID:dHCzUwYY
>>432
ワシも流れ曲げ派です。
435押しのけ吸い込み理論派:2001/07/28(土) 13:47 ID:dHCzUwYY
曲がらないと上面に真空が出来てしまうから曲がる。
実に簡単。
436ご冗談でしょう?名無しさん:2001/07/28(土) 14:47 ID:???

では今後は「27支持派」と名乗りたまへ、
NOTORIOUS弾性衝突派君
437流れ曲げ派:2001/07/28(土) 15:08 ID:dHCzUwYY
27派じゃ判りにくい
438ご冗談でしょう?名無しさん:2001/07/28(土) 16:42 ID:fjdr.Y8I
>低圧部が流体を引き付けるという話は、その低圧部に向かって
>いろいろな方向から流体が流れ込む
そう。だからこそそうならないように、翼を長く、翼末端を鋭く、そして
適切な迎え角をとって、翼末端下面の気流が大きくなるように設定するわけ。
翼が長い理由は面積を稼ぐ為では無く、定圧部が流体を引き付け、翼端渦流
(下から上へ空気が回り込む)による低圧喪失を防ぐ為なの。(長過ぎると
今度は、ヨー軸回りの慣性モーメントが大きくなって操縦性が悪くなり過ぎ
るから、無闇には長く出来ないけどね)
フラップも結局は、低圧部に空気がなるべく流れ込まないする工夫ね。

気流曲げ派(前スレ27派)の方は、「失速」という翼の揚力を語る上で
欠かせない重要な現象が何故起こるか説明出来るのかな?
”「コアンダ効果」が失われるから。”じゃダメでしょ。もしそうなら
「コアンダ効果」の起こる原因をもっと詳しく説明しないと。
439ご冗談でしょう?名無しさん:2001/07/28(土) 20:06 ID:xLs9X61I

ビクッ. ∧ ∧ ∧ ∧   / ̄ ̄ ̄ ̄ ̄ ̄ ̄ ̄ ̄ ̄ ̄ ̄ ̄ ̄ ̄ ̄ ̄ ̄ ̄ ̄ ̄ ̄
    (゚Д゚;≡;゚д゚) < うお!なんかすごいところに迷い込んじまったぞゴルァ!
     ./ つ つ    \______________________
  〜(_⌒ヽ ドキドキ
 ブッ ω)ノ `Jззз
440流れ曲げ派:2001/07/28(土) 22:14 ID:d4yiOUug
失速は流体力学にあげる。
441ご冗談でしょう?名無しさん:2001/07/28(土) 23:38 ID:qjr2dOw6
>>427
>「ベル派の‘圧力差’では揚力を理解したことにはならない」
>でしょ。実際に翼の周りの圧力分布は計算できるけど、何故そうなるのかは
>まだ誰も答えていない。

って、翼回りに循環(束縛渦)があるからって説明じゃだめなの?
442ご冗談でしょう?名無しさん:2001/07/28(土) 23:59 ID:???
>>441
翼周りの循環がゼロじゃない理由を示さないとダメでしょ。
ちなみに「発進渦があるから」もダメ。
「ある現象のために揚力が生じる」では理解できたことにはなら
ないよ。その現象が起こる理由を説明しないとね。

前スレで27は、翼で気流を下向きに変えれば循環はゼロじゃなく
なることを示したよ。

どーでもいーけど「流れ曲げ派」は前スレを読んでから出てきて
欲しいぞ。
443ご冗談でしょう?名無しさん:2001/07/29(日) 00:06 ID:YgTGfpe6
循環は絵に描いた餅であり、数学的に理想化された体系の
中で議論する時は意味があるが、現実の流れの循環の大きさを
定量するのは極めて難しいんじゃないの?
完全流体と密接なつながりのある、循環というベクトル量に
こだわり過ぎないほうが良いのでは?
「コアンダ効果の為に揚力が生じる」では、理解できたことに
ならないの?
444ご冗談でしょう?名無しさん:2001/07/29(日) 00:14 ID:1D7YJ3iQ
>>442
「発進渦があるから」がなぜ駄目なんだ?
何で発進渦があるかって事?
445ご冗談でしょう?名無しさん:2001/07/29(日) 00:29 ID:???
>>443
>「コアンダ効果の為に揚力が生じる」では、理解できたことに
>ならないの?

それは27派でしょ。その説明ならば理解できるけど、ベル派は速度
分布を考えてしまうからちゃんと説明できない。

>>444
> 「発進渦があるから」がなぜ駄目なんだ?

発進渦を打消す渦が、飛行している翼のまわりにくっ付いてくる
理由を説明しないとダメ。
446ご冗談でしょう?名無しさん:2001/07/29(日) 00:46 ID:1D7YJ3iQ
>>445
発進渦に対応する渦ってのは翼表面の境界層部における速度差による渦度として
存在してるから翼についてくるんじゃないの?
447ご冗談でしょう?名無しさん :2001/07/29(日) 00:58 ID:???
>>442
>どーでもいーけど「流れ曲げ派」は前スレを
>読んでから出てきて 欲しいぞ。

スマソ、これの意味が判らないのだが、何を言いたい?
何か勘違いしてない?
448ご冗談でしょう?名無しさん:2001/07/29(日) 01:08 ID:???
447だけど、非常にスマソ。
スレ汚しをしてしまった。

>>442
>どーでもいーけど「流れ曲げ派」は前スレを

これって個人名(旧「弾性衝突派」)の事ね。
ワシもそう思う。html化されたから読んでこいよ。
449ご冗談でしょう?名無しさん:2001/07/29(日) 01:23 ID:M0TGK/cs
えーん、むずかしいよう。
450446:2001/07/29(日) 01:27 ID:1D7YJ3iQ
そういえばそもそも翼周りに束縛渦がついてこないとクッタ条件満たさなくなる
んじゃない?そしたらそこでまた剥離が起こって発進時と同じ事が起こる気が。
451ご冗談でしょう?名無しさん:2001/07/29(日) 01:49 ID:???
>>446
そう、現象論的にはそれで良いのだけれど、翼まわりで rot v≠0
で、かつ循環が有限になる(揚力が発生する)流れの場ができる
理由も「境界層内部の速度勾配」で説明できるのかな。もう少し
詳しく書いてね。
結局プラントルの境界層方程式とかを解かないとわからないって
気がするんだが...

話はちょっと違うけど、ストークスの定理で面積積分を周回積分
に直すとき、内部に境界を含んでいてもオッケーなんだっけ?
452ご冗談でしょう?名無しさん:2001/07/29(日) 01:56 ID:???
>>450
クッタの条件って、循環を一意に決めるために翼の後端で速度が有限
になるってヤツでしょ。
束縛渦と関係あるのかな?
453ご冗談でしょう?名無しさん:2001/07/29(日) 02:14 ID:1D7YJ3iQ
>>451
とりあえず境界層内部の速度勾配はおいといて>>450で翼周りに
渦がついてくることは説明できるんじゃない。

>>452
発進時に翼の後縁以外で流れが流出(でよかったっけ?要するに流線が
出てくること)したら翼後縁で急激な速度増加・減少が起こって剥離が
起こってしまい、その結果流れは翼の後縁から流出する。その為には
翼周りに循環が存在しないといけない。って事じゃなかったっけ。
454446:2001/07/29(日) 02:17 ID:???
>>451
ストークスは…どうだったっけ?
455ご冗談でしょう?名無しさん:2001/07/29(日) 04:27 ID:kLT2QxtM
循環という量は、定量的な量では無く、定性的なものだと思われ。
定量的な支配関係(力とか、圧力とか)から、定性的な流れの場の性質を
導くことなどそもそも出来るのか甚だ疑問。

例えば、2次元平面で極座標表示で、2つの僅かに異なる速度ベクトル(分布)
が次で与えられる場合、単位円周りの循環は、一方は0であるのに、他方は0
では無い。それでいながら、両者の各点における速度差は1%にも満たない。

分布1: v(r,φ)=r^2(cos(2φ),sin(2φ))
分布2: w(r,φ)=v(r,φ)+1/100*r^(-1)(cosφ,-sin(φ))

完全流体で揚力が発生したのを確認して、初めて「ああこの流れの場の循環は
0でないんだなー」とわかる。揚力の確認と循環の存在の確認は同じこと。
同様に、今のままじゃ、「コアンダ効果」の存在の確認と揚力の確認は同じ
ことになってしまい、説明という意味では大いに不満と思われ。
456ご冗談でしょう?名無しさん:2001/07/29(日) 04:31 ID:kLT2QxtM
ゴメソ。訂正
誤)単位円周り
正)任意の原点中心の円周り
457ご冗談でしょう?名無しさん:2001/07/29(日) 08:33 ID:???
>>453
クッタの条件は境界層の剥離点が翼の後端付近にあるってことを
言ってるだけで、この条件があると翼周りの循環を一意的に決める
ことができる。だから循環が生じるための説明にはならないよ。

>>455
循環は定量的だよ。
一様な流れでは循環はゼロ。流れが周回積分のパスの内部で曲げ
られるとゼロじゃなくなる。

流体の密度をρ、翼の速さをV、翼周りの循環をΓとすると
翼の単位長さあたりの揚力はρVΓになるってのが、クッタ‐
ジューコフスキーの定理。
物理的には、流体の曲げられた方向の単位時間当たりの運動量
変化に等しいって、前スレで27は説明してたよ。つまり揚力は
下降気流を生じさせた力積の反作用ってことだな。

前スレをちゃんと読もうぜ。
でも>>1のリンク先に行っても「そんなスレない」と言われて
しまうのはオレだけか?
458ご冗談でしょう?名無しさん:2001/07/29(日) 09:39 ID:???
↑現れたページの一番下に「発見しました」ってある。
459ご冗談でしょう?名無しさん:2001/07/29(日) 18:26 ID:???
>>457
定量的性質か、定性的性質かというのは、定義にもよるから、私はコメント
しないが、少なくとも>>455>>456が言っていることは、流速のわずか1%
(もっと小さくても良い)が変わるだけで、循環が0になったりならなかったり
する。局所的な量関係で決まるものではないことということでしょう。
物理法則は局所的な量関係の積み重ねで全体を把握する傾向があるから、
循環という全体的な性質をそれでわかるかな?というのが、>>455の疑問なんだ
ろうね。それには同意。
460ご冗談でしょう?名無しさん:2001/07/29(日) 23:44 ID:???
しかし流れが流出する場所が翼の後縁であるって事はすなわち循環が存在する
ってことだろ。無揚力角を除けば。違うの?
461ご冗談でしょう?名無しさん:2001/07/30(月) 02:00 ID:???
流れが与えられたときに、循環を目で見てわかることが出来るかってのが
問題のようですね。
462ご冗談でしょう?名無しさん:2001/07/30(月) 07:00 ID:LrafsoqI
age
463ご冗談でしょう?名無しさん:2001/07/30(月) 10:10 ID:???
>>459
言ってることがよくわからんが、循環がゼロでなければその大きさ
に比例する揚力が得られるってことなんだが…
後半についてはストークスの定理を思い出してねってゆーのでは
ダメか?

>>460
別に後縁で流出しなくても循環はあるよ。翼の進行方向と垂直な
方向に流体の運動量を変化させればオッケーだ。

>>461
循環流と循環を混同してないかい?
途中で流れが曲がっていれば(=運動量が変化していれば)
循環はゼロではないよ。

実際の飛行機は静止した大気中で翼を前進させるから、下降気流が
見えれば循環が見えたことになるかな。
飛行機じゃ無理だけど、ヘリコプターを大きな台はかりの上に
乗せてちょっとだけ浮き上がらせれば、下降気流の大まかな
運動量も測定できるんじゃないの。
464ご冗談でしょう?名無しさん:2001/07/30(月) 22:36 ID:LrafsoqI
循環は一種のスペクトルとみた。流れのある種の成分と共鳴した時のみ揚力が
発生する。その成分は見た目じゃわからない。
揚力の存在でその成分を確認出来るって感じかな。
つまり、揚力は循環を計る道具であり、その逆は物理的には難しい。
465ご冗談でしょう?名無しさん:2001/07/31(火) 00:11 ID:???
ところで循環を直接測ってどうするつもり?間接的にでも
流れの様子から存在が分かれば良いんじゃないの?
とにかく翼の後縁から流れが出てれば循環の存在は明らかだと思うけど。
466流れ曲げ派:2001/07/31(火) 01:21 ID:???
循環を計ろうとして、飛行機を格納庫から出した途端、
翼の存在に気付いた周りの空気が循環しはじめ、
実験屋は機材もろとも吹き飛ばされましたとさ。
467ご冗談でしょう?名無しさん:2001/07/31(火) 09:23 ID:???
>>465
>とにかく翼の後縁から流れが出てれば循環の存在は明らかだと思うけど。

ええと、よくある断面がカマボコ型の翼まわりの流線で、翼の下側は流入す
る前も流出した後もまっすぐ、翼上面は翼に沿って滑らかに流れて後縁で下面
の流れと合流するってヤツでは、翼周りの循環を計算するとゼロです。

流れが翼で曲げられない限り、循環の大きさはゼロすなわち揚力は発生しない
ってことです。

>>466
循環≠循環流なんだが…
468ご冗談でしょう?名無しさん:2001/07/31(火) 23:35 ID:XNXoqopw
他人に意見するならsage進行はやめようぜ。初心者じゃないんだからね。
469ご冗談でしょう?名無しさん:2001/07/31(火) 23:48 ID:???
なんかそういうしきたりなのかと思ってたよ。スマン。
470ご冗談でしょう?名無しさん:2001/08/01(水) 00:10 ID:5jctj4OI
>>467
流れが翼で曲げられて循環が発生した場合Kutta条件はどうなんの?
循環がゼロでもゼロじゃなくても満たされてるわけなのかな。
ところで半円周りの流れってどうやって求めたっけ。
誰か教えて頂戴。
471ご冗談でしょう?名無しさん:2001/08/01(水) 05:05 ID:iV/dXGpI
スプーンの後ろを水流に当てた時、水がスプーンに沿って
流れて、水流にスプーンが曲げられるという例が、気流曲
げ派の「ウリ」なら、気流曲げ派なんてインパクトの無い
コテハン辞めて、「スプーン曲げ派」に変えたら?
472ご冗談でしょう?名無しさん:2001/08/01(水) 08:51 ID:???
>>470
クッタの条件って、複素速度ポテンシャルで突角をまわる流れを
考えると(翼とか平板の後縁ね)、突角のところで複素速度が発散
してしまうので、それを有限にするって条件。これを使うと循環の
大きさが決まって揚力が求まるんだな。
計算上の問題なんで、実際の循環がクッタの条件で生じると考える
のは間違い。

以下、上級者向け。
27派のオレが複素速度ポテンシャルでちょっとだけ解説。

半径aの円柱まわりの複素速度ポテンシャルは
 f(z) = Uz + iκ ln z/a + Ua^2/z
で、右辺の項はそれぞれ一様流、原点まわりの渦糸、湧き出し。
渦糸の強さ(循環のことだ)2πκは不定。円柱の回転の角速度に
対応してる。回転円柱には揚力が発生するってヤツだ。

一様流に対して仰角θで置かれた長さ2aの平板まわりの流れは、
ジューコフスキー変換(z=ζ+a^2/ζ)で円柱まわりの流れに置
き換えられて、
 f(z)=F(ζ)=exp(-iθ)Uζ + iκ ln ζ/a + exp(iθ)Ua^2/ζ
この複素速度を求めると
 df/dz = df/dζ/dz/dζ=(分子は各自で計算)/(ζ^2-a^2)
となり、ζ=aで発散してしまう。クッタの条件はこの発散を無く
すためにζ=aで(分子)=0にするってことだな。
そーすると不定だった循環2πκも決まって、揚力が求まる。
473流れ曲げ派:2001/08/01(水) 22:35 ID:???
円柱が最も効率の良い翼形というのが物理板の結論ということで
航空板の人に伝えて来ていいですか?
474ご冗談でしょう?名無しさん:2001/08/01(水) 22:53 ID:X34M3azI
sage進行はやめなよ。書き逃げみたいでウザくて女々しい。
475ご冗談でしょう?名無しさん:2001/08/01(水) 23:11 ID:???
>467
>ええと、よくある断面がカマボコ型の翼まわりの流線で、翼の下側は流入す
>る前も流出した後もまっすぐ、翼上面は翼に沿って滑らかに流れて後縁で下面
>の流れと合流するってヤツでは、翼周りの循環を計算するとゼロです。

って具体的にはどんな翼型でcl,cdはどれぐらいなんでしょうか。
単純なカマボコ型だと流れが滑らかに流れませんよね。
その翼型の性能が分かれば従来の説明による揚力と流れが翼によって
曲げられて発生する揚力の大きさの比較ができるんでは。
476ご冗談でしょう?名無しさん:2001/08/01(水) 23:42 ID:???
>>475
いや、そーゆー実際の話じゃなくって、「翼の後縁から流れが出て
れば循環がある」だけでは正しくないことを言ったのが>>467
気流が曲げられないとダメ。

下降気流が無い流線を示して「揚力は翼の上下の圧力差」って、
間違った説明をしているウェブページは多いよ。
477457:2001/08/02(木) 00:07 ID:fP3/WsNE
>>476
これまでの話からの循環のできる理由の自分の解釈は
 ・翼の後ろの端が鋭いことによって流れが回り込む際に剥離するため
 ・流れが曲げられるため
ってことなんだけど、前者と後者は別に矛盾してるわけじゃないし、両方の
効果によって循環・揚力が発生してると考えてもいいんではないかと。
そしてかまぼこ型の場合(前者が存在しない場合)の揚力が分かれば両者の
揚力への貢献の割合を知ることができると思ったわけです。
476さんは前者の解釈は完全に否定されているのでしょうか?
478475:2001/08/02(木) 00:08 ID:fP3/WsNE
間違えました。>>477は457ではなく475です。
479476:2001/08/02(木) 01:02 ID:???
>>477
>翼の後ろの端が鋭いことによって流れが回り込む際に剥離するため

滑らかに剥離して下降気流になれば、循環はゼロじゃないけど、
剥離しても気流が翼に流入した時と同じ方向を向いていれば循環はゼロ。
気流の向きが変わる=運動量が変化するってのが大事。
480ご冗談でしょう?名無しさん:2001/08/02(木) 01:48 ID:4GnRUYnI
ここに居る人は飛行機が飛ぶ理由がわかるって事は
F1マシンがダウンフォースを得る理由も分かるわけだな。
昨今流行のハイノーズの理由を教えてくれ。
481流れ曲げ派:2001/08/02(木) 02:36 ID:???
>>480
もう少し解説を求む。
F1マシンって空飛んでもOKなのかな?
482ご冗談でしょう?名無しさん :2001/08/02(木) 02:56 ID:???

うわっ、またコイツわけわかんない事を書きに来やがった!
嵐でやってんのかよ。 ボコボコにされて退場きぼんぬ。
483ご冗談でしょう?名無しさん:2001/08/02(木) 03:45 ID:???
>>482はコピペ
484ご冗談でしょう?名無しさん:2001/08/02(木) 07:58 ID:???
>>480
F1マシンは,もしスピンして逆方向にまっすぐ進めば飛ぶ事が可能。
ちなみにダウンフォースの大きさは車重を越えているので
スピードの乗った状態で天井に張り付けば落ちずに走れます。

ハイノーズとは先頭の尖がった所を
持ち上げて大量の空気をマシンと地面の間に送り込み
真中付近は地面ギリギリまで車高をさげて
後ろにあるデフューザでまた一気に車高をあげます。
こうすることでマシン下部に流れの速い空気を作りだし
負圧をかけてダウンフォースを得るという発想です。
現在フェラーリを設計しているロリー・バーンが考案したものです。

それまでの発想ではもっと安直にボディ下部をウィングの逆形状をした
ものにしてダウンフォースを得ていたのですがルールで
禁止されて下部は平らにしなければならないので
それならは車高は低いほど良い
(コーナリング中は重心が低い方がロールが起こりにくい)
という発想からノーズはギリギリ下まで下げられていることが殆どでした。

これらの方法で得られたダウンフォースはウィングを起こすだけという
単純な方法に比べドラッグが少ない,つまり直線でもカーブでも速く
レース向きというわけです。
485484:2001/08/02(木) 08:19 ID:???
適当なF1HPからパクってきました。
これがどうやら一般的に知られている理論だけど
正しいのかどうかをここの人に判断して欲しい
486ご冗談でしょう?名無しさん:2001/08/02(木) 08:22 ID:???
>>484
>ハイノーズとは先頭の尖がった所を
>持ち上げて大量の空気をマシンと地面の間に送り込み
>真中付近は地面ギリギリまで車高をさげて
>後ろにあるデフューザでまた一気に車高をあげます。

これは、途中で内径が絞られた管の中に空気を送り込んだ状態と
大体同じか。ベルヌーイの定理で真中付近の静圧が下がりダウン
フォースになるんだな。でも、後ろ側から出て行くときには
「上昇」気流もありそうだが…
487ご冗談でしょう?名無しさん:2001/08/02(木) 16:08 ID:JiZzT1vk
>>486
>でも、後ろ側から出て行くときには「上昇」気流
>もありそうだが…

つまり、コアンダー効果によるダウンフォース向上
の効果もあるって事になるんだろうか。飛行機と同じか。
488ご冗談でしょう?名無しさん:2001/08/02(木) 16:22 ID:JiZzT1vk
ところで、そろそろ本題に戻らないかい?

「翼により空気流れが曲げられて初めて揚力が発生する」
というところまではベル派(標準派)も納得したと思う。
で、問題はその下向き流れの反作用が揚力なのか、それと
も揚力の一部にしか寄与していないのかという事だと思う。

私は前者の意見(ただし、この反作用が揚力の本質であ
るだけであり、翼全面の圧力の総和を取っても揚力が求
まる= 中間派)なんだけど、ベル派(標準派)は後者の
意見という事で良いだろうか。そうすると、厳密な意味
でのベル派(標準派)というのは居なくなり、「折衷派」
という事になると思うけど如何?
489ご冗談でしょう?名無しさん:2001/08/02(木) 18:14 ID:???
>>487
>つまり、コアンダー効果によるダウンフォース向上
>の効果もあるって事になるんだろうか。飛行機と同じか。

う〜ん、これが飛行機と同じではないってことは、前スレで議論
されてたことなんだが…
F1のボディー下面の構造は、ベルヌーイの定理の応用例で出て
くるキャブレターと同じで、流速の速いところで静圧が下がるか
らダウンフォースが生じる。でも、これは流れを曲げているわけ
ではないので「揚力」が働いていると考えてはいけない。単に外側
の大気圧によって押されているだけ。
F1にしろキャブレターにしろ、大事なのは低圧部分が閉鎖され
ているとみなせるってことだ。低圧部が開放されていたらどーな
るか? 単に低圧部に空気が流入してくるだけで力は働かない。
もし、これで力が生じるなら掃除機のホースの先を上に向ければ
空を飛べる(by前スレ27)。ホースの先を天井スレスレに近づけ
た場合は低圧部が閉鎖されているとみなせるので、天井に吸付く。

だから、閉鎖されているキャブレターのアナロジーで、開放され
ている翼の周りに圧力差が働いて揚力となると考えてはいけない
ってのが前スレ27の主張。
で、これがわからない連中のことを「ベル厨」と呼んでましたな。

前スレ後半あたりで登場してくるのが「標準理論派」だが、
長くなったので後は次レスだ。
490489:2001/08/02(木) 18:42 ID:???
さて「標準理論派」の主張するところは
 「翼周りの圧力分布を積分すれば揚力になる」
ってことで、これは翼が受けている力は大気からの圧力のみなので
まぁ、当たり前のこと。で、「ベル厨」との違いは、
 「翼上面の低圧部めがけて大気が流れ込むことはない。流体の
  運動の法則を考えると、ちゃんと低圧部が定常的に存在できる
  速度分布になっている」
これは確かにその通りで、全く正しい、と前スレ27も認めていた。
では、「標準理論派」では何が問題かってゆーと、
 「なんでそのような速度分布になるんですか?」と訊かれても
 「ん? それはだな、方程式を解かないとわからないんだ」
としか答えられないところでしょ。つまり、
「標準理論派」では揚力の計算はできるが、揚力を説明できない。

とゆーのが、これまでのスレの流れ。
491489:2001/08/02(木) 18:56 ID:???
>>488
>で、問題はその下向き流れの反作用が揚力なのか、それと
>も揚力の一部にしか寄与していないのかという事だと思う。

クッタ‐ジューコフスキーの定理を認めるならば、下降気流の
反作用が揚力そのものでしょ。

「中間派」とか「折衷派」ってのはよくわからん。

27派の説明で圧力が下がるってことを示すのはちょっと大変。
だから前スレ27は「圧力差」を認めながらも、敢えてそれには
言及しなかったんでしょうな。
492 :2001/08/02(木) 19:24 ID:???
前スレ27はこのスレでは名無しとして来ているね。
493ご冗談でしょう?名無しさん:2001/08/02(木) 19:53 ID:XWwRLMkA
モーターが充分強力なら、掃除機だって浮くだろ。
494ご冗談でしょう?名無しさん:2001/08/02(木) 20:28 ID:f1Xb.Ufw
つまり実際に起こってる現象としてはこういうことか?

 1.翼の後ろの端が鋭いために流れは粘性によって回り込めず剥離する。
 2.このため流れは非粘性流れの場合と姿を変え、循環が0で無くなる
   =流れが下向きに曲げられる

2の循環によって生ずる速度差に着目したのがここでいうベルヌーイ派
で、流れが下向きに曲げられることによる運動量変化に着目するのが27派
である。俺としては両方とも同じ事を別の視点から見てるだけだと思う。

ちなみに三次元翼の場合に随伴渦によって起こる流れの下向き成分は誘導抵抗の
元でグライダーなんかではこの成分を極力減らすために高アスペクト比の翼を
使っている。また、翼の平面形を楕円形にするとこの誘導抵抗を小さくする
ことができる。ってことじゃない。
495流れ曲げ派 :2001/08/02(木) 21:18 ID:???
静止している空気の中を翼が通り過ぎて行くのを考える方が簡単だよ。
通りすぎた体積を考えれば良いだけだし。
496ご冗談でしょう?名無しさん:2001/08/02(木) 21:48 ID:ncEox8.s
487,488です。

>>489-490
F1ボディの下面の話しは了解、私が勘違いしてました。
ベル派、標準派の違いは単にベルヌーイの定理を「厳密」
に用いるか「近似的に」用いても良いと見なすかの違いかと...。

>>491
下降気流は見とめるけど、クッタ‐ジューコフスキーの
定理を認めない人もベル・標準派には居るのでは?速度
圧力分布の測定値からその定理を用いて下降気流の大き
さ(=揚力)を計算できる訳ではないよね。ちなみに
  中間派:揚力=圧力差=下降流の反作用
  折衷派:揚力=圧力差+下降流の反作用

>>494
前半部の記述は正しいのかな?良くわからない。
誘導抵抗の話しは揚力の発生、本質に基本的に関係
ないと思う。ただ個人的には、この話しが落ち着い
たところで地面効果の原因の話しをしたいと思って
いる。27派参考文献が納得いかないんだよね。
497ご冗談でしょう?名無しさん:2001/08/02(木) 21:58 ID:THQBxVZw
クッタ−ジューコフスキーを認めないで、どうして
速度差(圧力差)が生じることを説明するのよ?
>速度圧力分布の測定値からその定理を用いて下降気流の大き
>さ(=揚力)を計算できる訳ではないよね。

できるよ。でなかったら飛行機が設計できないじゃない。
498ご冗談でしょう?名無しさん:2001/08/03(金) 01:32 ID:???
>>493
空気の粘性がメチャ高いとか、掃除機の密度が空気と同じくらい
じゃないと浮かないでしょ。

>>494
流れが翼の後縁で剥離するのと流れが曲げられるのは無関係。

>2の循環によって生ずる速度差に着目したのがここでいうベルヌーイ派

循環があるから速度差が生じるわけではない。
流れの運動の法則から翼周りの速度分布および圧力分布が決まる
ので、そこから揚力を求めるのが標準理論派(=ベル派?)。
圧力分布を積分しても良いし、速度分布から循環も計算できるの
で、クッタ‐ジューコフスキーの定理を用いても良い。
499ご冗談でしょう?名無しさん:2001/08/03(金) 01:52 ID:???
>>496
>  中間派:揚力=圧力差=下降流の反作用
>  折衷派:揚力=圧力差+下降流の反作用

こんなのは無いって。

前スレで27は、気流を下向きに曲げたときの運動量変化から求まる揚力と、
クッタ‐ジューコフスキーの定理で循環から求めた揚力が同等であることを
示した。つまり、27派も標準理論派も揚力を計算する立場では同等である
ってことだ。

だから、中間派ってのは無意味だし、クッタ‐ジューコフスキーの定理が
運動量変化そのものなのだから折衷派は間違い。

27派と標準理論派の違いは、揚力をちゃんと*説明*できるかってことだけ。
500ご冗談でしょう?名無しさん:2001/08/03(金) 04:21 ID:LsZa8rm.
あらためて前スレ(HTML化された奴)はここ
http://cheese.2ch.net/sci/kako/988/988866699.html
501ご冗談でしょう?名無しさん:2001/08/03(金) 09:44 ID:HSfO9VLU
>>499
どうもありがと。私自身、10数年前に、揚力が空気流れを下向き
に曲げた反作用で生じることを聞いて以来27派だったけど、この
スレのおかげで結構スッキリしてきた。

ベル派(標準派)は圧力差で揚力を説明しているつもりなのだが、
その圧力差が生じるための本質であるところの空気流れの曲がり
に言及していないという点において「説明」になってないという
理解で良いかな。
502ご冗談でしょう?名無しさん:2001/08/03(金) 09:47 ID:HSfO9VLU
↑ゴメソ、496でした。
503whoami:2001/08/03(金) 11:50 ID:???
>>501
標準理論派でもベル派でも揚力の説明で、圧力分布の生じる原因が
速度分布にあるとした時点でアウト。結局はナビエ‐ストークス方
程式を解かないとわからないんだ。この第2スレでもう少しその辺
の理解が進むかと思ったけど、何もでてこなかったな。「長スマ」
に期待したんだが、いつも逃げてばかりでちゃんと応えていなかったな。

そーすると
「飛行機ってなんで空を飛べるんですか?」
に対する回答は、やはり
「翼で空気に下向きの運動量を与えているから、その反作用が
 機体にかかる重力と釣り合う」
でしょ。

標準理論派は
「工業的に翼をデザインするにはどーするんですか」
って質問の回答になっていて
「流れの運動の法則を表した方程式を解くと、翼周りの圧力分布や
 速度分布が計算できるので、それで揚力(や抗力)を求めます」
なんでしょう。

標準理論派(ベル派)は揚力を精密に求める方法を示しているので
工学的には非常に重要。だけど、それで揚力が生じる理由も説明で
きると考えるのはダメ。
運動の法則がわかっていて運動方程式が解けたからって、現象を
理解したことにはならないんだが、翼の揚力に関しては、それで
理解できたと勘違いしている場合が多いのはどーしてでしょう?

運動量変化を用いた説明ならば、揚力の本質を理解しやすいのだ
から、大勢がこちらの説明に変わるのをキボンヌ。
翼下面では空気の分子の弾性衝突、翼上面では分子間の引力で
下向きの運動量を与えてるってことだが、翼上面の話はわかりに
くいかもしれないな。でもちゃんと勉強すればわかるんだから、
「速度差」で理解したつもりになるよりはマシでしょ。

とゆーことで、標準派の反論が無ければ、これで「まとめ」かな。
504ご冗談でしょう?名無しさん:2001/08/03(金) 12:18 ID:flkeGgVc
>>503
501です。ますます理解が深まりました。(ただし、
翼下面が弾性衝突なのかは議論の余地があるかと
思いますが...) 大筋では大円団というところでし
ょうか。私もこの説明が広まる事を希望します。

回転翼と固定翼は基本的に同じ原理で揚力を生じる
と思うのですが、空気に与える運動量の反作用で揚
力が生じる事について、回転翼はイメージし易いの
ですが、固定翼はイメージし難いと言う事でしょうか。

前スレでも多少誤解があったので、再確認しておく
と、下記いづれもAが「本質的」な答えですよね。

飛行機のプロペラが前方に推力を持つのは何故か?
A. プロペラ翼により空気に後方運動量を与えた反作用
B. プロペラ翼前後面の空気速度差に起因する圧力差

ヘリが飛ぶのは何故か?(揚力を持つのは何故か?)
A. ローター翼により空気に下方運動量を与えた反作用
B. ローター翼上下面の空気速度差に起因する圧力差

飛行機が飛ぶのは何故か?(揚力を持つのは何故か?)
A. 翼により空気に下向き運動量を与えた反作用
B. 翼上下面の空気流れ速度差に起因する圧力差
505ご冗談でしょう?名無しさん:2001/08/03(金) 14:34 ID:zBn0KAQ6
待ったぁー!
運動量で安易に理解したつもりになってはならぬ。トンデモな世界に
突入じゃ!
クッタージューコフスキーの定理と浮力をもう一度考察キボンヌ。
それらの間の関係が、今一つだから。速度U->0の極限で、浮力に
一致しなければならないだろうが、この時循環は一体どうなるのか?
506ご冗談でしょう?名無しさん:2001/08/03(金) 14:46 ID:???
>>505
待たないよ。
翼の速さUがゼロなら循環がゼロでなくても、
クッタ‐ジューコフスキーの定理から揚力はゼロ。
「U→0の極限で浮力に一致」って意味不明。
で、何が知りたい?
507ご冗談でしょう?名無しさん:2001/08/03(金) 20:17 ID:BFEzfPTo
なんかまとめかけてるところに申し訳ないが。

なぜ運動量変化が起こるのかって説明に不満がありますね。
翼下面では空気の分子の弾性衝突、翼上面では分子間の引力で
下向きの運動量を与えてるっていうのは少しいいかげんではな
いでしょうか。これについても計算しないことには運動量変化
がどうなるか分かりませんよね。結局方程式を解かなくてはな
らないって点では両者とも同じです。

自分が考えるに揚力について正しく説明しようとすればどうしても
途中で何らかの計算が必要なのではないでしょうか。したがって、
圧力差による説明と運動量による説明を両方しとくのが無難ではな
いでしょうか。どちらが良いとは一概には言えないでしょう。
508ご冗談でしょう?名無しさん:2001/08/03(金) 21:23 ID:???
>>507
運動量変化でどんな「方程式」を解かなきゃ説明できないと思う?

翼周りの速度分布と圧力分布が方程式を解かずに説明できるの?

揚力を「計算」するのが標準理論派の方法だが、それは揚力が
発生する「説明」にはなってないってことはわかった?

「俺なら翼周りの速度分布はこう説明する」ってカキコは歓迎
なんだが、それ以外はもっと良く読んで、ちゃんと考えてから
書き込んでね。
509流れ曲げ派:2001/08/03(金) 22:57 ID:???
前スレ1の前で、その方程式解いてみせれ。
510507:2001/08/03(金) 23:17 ID:uhxF6r0M
>>508
申し訳ないが分からん。流れが曲げられたと仮定したときの運動量の変化
なら簡単に計算できますが。それじゃ循環を仮定するのと同じだし。
流れがどのように曲がるかは計算しないと分からないのでは?
それだったらどっちの説明でも結局同じではないかということが言いたい
のですが。
511ご冗談でしょう?名無しさん:2001/08/03(金) 23:51 ID:???
>>510
だから、流れがどのように曲がるかを計算するのは標準理論派。

運動量変化の大きさをマジで計算しようとすると、気体の分子運動
論+統計力学的な考察が必要になるよ。でも、そんな計算をしなく
ても、「空気の分子に下向きの運動量を与えた反作用が揚力」って
言えば納得できるよね。

標準理論派が速度分布を計算するのはナビエ‐ストークス方程式と
連続の式とエネルギーの式。で、計算しなくても「翼周りの速度
分布(圧力分布)は、こーなる」って説明ができれば良いのだが、
なかなか難しそう。つまり「計算はできるが説明はできない」って
ことで、27派の場合とは事情が異なる。

何か良いアイデアはある?
512507:2001/08/04(土) 00:11 ID:ClpQCbx.
いや、それって流れが曲がるのはとにかく前提条件って事?それなら別に
「翼の形状により循環が生まれて翼の上下で流速が違ってきてその結果
圧力にも差が出てそれが揚力」
って説明と変わんない気がするんですよ。
「何で循環が生まれるか」ってのと「なんで流れが曲げられるのか」
っていう点を明確に説明しない点において。両者ともあいまいな点が
あるというか。そのあたりが気になるなと。
まあ一般向けなら自分はどっちの説明でもいいと思ってます。
513ご冗談でしょう?名無しさん:2001/08/04(土) 00:40 ID:9HLideLQ
>>506
ハズレ!
重力の影響は無視出来ない。重力を考慮した場合のクッタージューコフスキー
の定理は、浮力+密度×循環×速さ
速さ->0の極限で、浮力に収束します。

空気の運動量変化なんだが、これは一筋縄ではいかない。
クッタージューコフスキーの定理は、そもそも
2次元の非有界領域で渦無しの流れを前提としています。
翼を取り除いた平面全体(質量∽!)を考えて?成り立つか
成り立たないか、微妙なところでしょう。平面全体の質量なんて
定義出来ますか?
翼の周りの有限領域以外は取り敢えず無視しても、その領域全体
の運動量の変化についてのみ言え、翼周辺の局所的な気団の運動量
変化に(揚力の反作用が)集約されるかは結論できません。
そもそも、現実の航空機の翼との接続性を無視しない、このスレの
立場から言って、少なくとも領域は半平面で考える必要があります。
この場合、クッタージューコフスキーの定理(循環で揚力を表現する)
が成り立つかについては、恐らくNoでしょう。
そしてこの場合、地面に相当する領域の境界にかかる圧力も無視できま
せん。(この圧力の反動は、確実に上昇気流を作るでしょう。)
514ご冗談でしょう?名無しさん:2001/08/04(土) 01:10 ID:???
>>512
要するに「空気は流体」って概念から脱却できないだけね。

気体分子の運動量変化を考えるときに「流れ」なんて関係ないよ。
衝突(斥力)か引力で分子の運動の方向が下向きに変化すれば、
揚力が発生するってことなんだけどな。
実際には統計平均として翼周りの空気は下向きの運動量を得ている
わけで、この状態をマクロにみると「流れが曲がった」ように見え
るだけ。
27派の説明は明快でしょ。

それから
>翼の形状により循環が生まれて翼の上下で流速が違ってきて
この書き方もダメ。循環が生まれるから流速が異なるのではない。
「翼の形状により速度差が生じ、循環を計算するとゼロじゃない」
ってのがホント。だから「どーしてその速度差が生じる」って
疑問が出るのだが、標準理論派は解決していない。
515ご冗談でしょう?名無しさん:2001/08/04(土) 01:28 ID:???
端折り過ぎてるせいか意味不明なカキコがあるな。
デムパが出ているよーな気がするが...
516ご冗談でしょう?名無しさん:2001/08/04(土) 01:43 ID:Hko20Wuw
>>507
循環と呼ぼうが速度差と呼ぼうがどっちでもいいが、循環が発生する理由
については誰も説明していないな。
前すれでstarting vortex の話をしていた人はいるけど。
あと、運動量収支と圧力分布の計算をしてどっちも半々って結論だした
人もいたよ。
27派もベル派もそれにはちゃんと答えてないよ。
517ご冗談でしょう?名無しさん:2001/08/04(土) 02:01 ID:???
>>516
>循環が発生する理由については誰も説明していないな。

周回積分のパスの内部で流れが曲がっていれば循環は発生するよ。
>あと、運動量収支と圧力分布の計算をしてどっちも半々って結論だした
>人もいたよ。

この人は物理をよくわかっていたのですが、計算のミスを指摘され
てから消えてしまいました。もう少し続けてくれれば良かったと思う。
518507:2001/08/04(土) 02:04 ID:ClpQCbx.
たびたびすいません。
要するに翼の下面を気体分子が押し上げ、上面を(相対的に)引っ張って
いて、その反作用として流れが下向きになってるという事でいいのでし
ょうか。すると揚力の存在が前提となってしまい堂々巡りのような気が…
つまり、"翼の下面を気体分子が押し上げ、上面を(相対的に)引っ張って
いる"というのは揚力そのものではないですか?
519ご冗談でしょう?名無しさん:2001/08/04(土) 03:19 ID:???
>>518
何が疑問なのかよくわからないよ。
目覚めてからもう一度考え直してみて。
520507:2001/08/04(土) 03:52 ID:???
>>519
スマン。そうする。
521ご冗談でしょう?名無しさん:2001/08/04(土) 06:11 ID:rMfMGaRs
循環を考える限り、空気の流体性を無視するわけにはいかんだろうに。
連続の方程式という制約条件が課されていることを忘れてはならない。
(そこに実は問題の本質が隠されていたりして)

クッタージューコフスキーの定理に拘りすぎる(循環なる数学的量も
含む)と、飛行機の翼に働く揚力の本質が見えなくなるんじゃない
かな?
クッタージューコフスキーの定理の成り立つ条件を、列挙すると。

1:翼にかかる応力は、圧力(地上!で、大気圧!に一致するように
正規化した量)の表面積分となる
2:空気は完全流体で、渦無しの定常解に収束する。
3:その定常解は循環を持つ。

その前の大前提として、領域が二次元で、遠方の流れベクトルは
一定って条件があった。
513さんの指摘の通り、かなり強い条件の元に成り立っている
ような感じ。少なくとも、循環のある流れに漸近するってことが
証明できなければ飛行機の翼に働く力の説明にはならない。
つまりオイラー方程式(ナビエストークス方程式よりは簡単とい
われてはいるが真相は定かでない。)を解析しなければダメという
点では標準派の理論そのもの。
結局のところ、循環があるというのは、上下に速度差が出来ること
と言っても(大抵の場合は)良いだろうから。ベルヌーイ派の説明
と、実は本質的に同じ。ベル過信派への批判は、そのままクッタージュ
ーコフスキー過信派への批判として通用する。

513さんが指摘したように、運動量変化で、飛行機が落下しない
理由を説明する人も居るが、これも一つの説明ではある。そういう
状況も有り得るだろう。しかし空気を下に「噴射」する(結局の
ところはそう言い切っても良いだろう)ことによってのみ揚力を得
ているのではなく、空気に荷重を伝達分散させて、地上で支えてい
るという見方を、過小評価している点は問題。
伝達しているのは、ズバリ音波(超低周波)である。
飛行機は音によって飛んでいるという人さえ居る。
では、何故そのようなことが出来るのかという説明は、空気を
完全流体とみなさず圧縮性流体とみなし、翼の最大翼高位置直後
上面に発生するCavity(キャビティ)を認めることに尽きる。
キャビティはキャビテーションという言葉で、機械系の人なら
必ず知っている言葉。要するに真空が発生することね。
どうして、その真空(に近い低圧状態)が維持されるのか。どうして
それが発生するのかについてだが、前者は翼の形状の工夫。
後者は、翼が移動する為。
ピストンのシリンダーを引っ張ったら、中の気体は一様には圧力
低下しない。理由は粘性とか、慣性とか色々ある。特に押して
圧縮した直後等はその傾向が著しい。
こうして、上面に出来たCavityに由来する低圧部と下面の大気圧より
も高圧な部分との応力差によって、翼は動的浮力を受けるのだと
考えられる。
522ご冗談でしょう?名無しさん:2001/08/04(土) 09:08 ID:???
sigh...

>>513
循環の定義を誤解しているとオモワレ
空気の場合、翼の浮力は考えないよ。

>>521
翼上面に低圧部ができるのは、シリンダー内のピストン上面が
低圧になるのと同じって主張かな? シリンダーは閉鎖されてい
るけど翼の周りは開放されているわけで、その違いがわからな
いのは、やはりアレか。低圧部が生じたとしても、下降気流には
ならないってのも、前スレでガイシュツ。
個人的には「飛行機は音波で飛ぶ」ってヤツの参考文献をキボンヌ。
523押しのけ派:2001/08/04(土) 14:02 ID:QpVrnfh2
>>522
新説だからないだろ?
524押しのけ派:2001/08/04(土) 14:05 ID:QpVrnfh2
>>521は式立てておくれ。
525ご冗談でしょう?名無しさん:2001/08/04(土) 16:04 ID:???
気体分子の運動量変化のマクロな視点が圧力
526ご冗談でしょう?名無しさん:2001/08/04(土) 16:47 ID:76miWbL2
いずれにせよ、一番わかりにくいのは、翼の回りの空気が
地面との相対速度一定の時、時間的に変化しない流れになる
ってことじゃないのかな?その前提で議論されている限り、
結局は方程式の解がある特殊な性質を持つので、循環なり
速度差なり、運動量変化(?)なりが発生して圧力差となる
というのが、結論のようだ。
527513:2001/08/04(土) 20:32 ID:9HLideLQ
>>522
別に誤解していないよ。重力を考慮した条件では、クッタージューコフスキー
の方法で速度場からベルヌーイの定理(オイラーの方程式から導かれる)
を使って圧力を求め、翼の周りで積分すると、きちんと、浮力+揚力
という形で求まる。むしろ、揚力=浮力+循環・速さ・密度
という捉え方のほうが正確。浮力は、ここでは揚力の特別な場合となり
揚力は浮力を非静止状態へ拡張したものと見ることが出来る。
528ご冗談でしょう?名無しさん:2001/08/05(日) 00:02 ID:???
>>526
ちょっと違う。
揚力があるんだから圧力差があるのは当りまえ。翼に働く力は大気
の圧力のみだからね。

27派の説明では敢えて圧力差に言及しなくても、空気に下向きの
運動量を与えた力積の反作用が揚力であることを容易に理解できる。

標準理論派は、翼周りの圧力分布の積分で揚力を求めるか、速度
分布から循環を計算してクッタ‐ジューコフスキーの定理で揚力
を求める。が、肝心の翼周りの圧力分布や速度分布に対しては、
方程式を解いてみないとわからないってことで、揚力の説明が
ちゃんとできてない状態。

標準理論派の「翼周りの速度分布はこーして生じる」って説明が
あるのなら、早く書いてね。
529ご冗談でしょう?名無しさん:2001/08/05(日) 00:10 ID:???
>>527
>重力を考慮した条件では、クッタージューコフスキー
>の方法で速度場からベルヌーイの定理(オイラーの方程式から導かれる)
>を使って圧力を求め、翼の周りで積分すると、きちんと、浮力+揚力
>という形で求まる。

循環はどこで使う?
揚力を運動量変化で説明するのがダメな理由もキボン。
530ご冗談でしょう?名無しさん:2001/08/05(日) 00:21 ID:7H7.w9Js
>>528
揚力があるんだから気体の運動量変化があるのもあたりまえなのでは
ないでしょうか?
運動量変化を気体分子の引力・斥力によるものだと説明されてますが
気体分子の引力・斥力とは圧力そのものなのでは。
その辺がどうも引っ掛かってるのですが。
531ご冗談でしょう?名無しさん:2001/08/05(日) 03:21 ID:ISw5QgnY
結局のところ、「何故飛行機は飛ぶのか」という疑問に対して
どういう解答をしてやるのが最も良心的かってことでしょう。
27派の説明は、初心者にはやや難解な圧力という概念(静圧との
区別とか)を持ち出さずに、運動量という比較的取っ付きやすい概念
だけで説明しているのがウリで、しかも定量も簡単。
だが、言葉や概念がわかりやすいのは必ずしも良心的とは言えないの
では?

例えば、他人を殴ったとして、相手が「何故殴るんだ!」という質問
に対し、「その反動で僕の手は、これだけ腫れ上がって、これから
貴方の顔に与えた力積及び、貴方の顔が得た運動量が計算出来る」
なんて解答、普通するだろうか?(喧嘩ならするってw)もっともだ。
しかし、質問者の「中学生」は、因縁を付ける目的で質問したのだ
ろうか?「私には、飛行機が揚力を得るトリックが解りません。
だれか種明かしをしてください。私でも確認できる、物理法則を
どのように利用しているのか、教えてください。」と聞いていたので
あろうに。

27派の回答は、つまるところ、「揚力があるでしょ。その反動を
空気は受けなければならないんだよ。そしてそれは、翼付近の空気が
集中的に下向きになることで見られるよ。その量から揚力が計算出来
るよ。もしくは飛行機の重さと速度からその下降成分も計算出来るよ」
と言ったわけ。こういう解答が期待されていたのだろうか?

クッタージューコフスキーの定理は、ベル派の説明の特殊な場合に
おける精密化と捉えられる。本質的に圧力差の理論。しかし、
この定理は飛行機の揚力の計算を目的に作られたものではなく、
ダランベールのパラドックスの数学的証明に絡んだ議論から派生した
奴で、飛行機の揚力の説明をする場合、必ずしも通らなければならない
ってわけでも無さそう。
結局、27派は、質問者の解答にはまともに答えてないし、ベル派の
説明の一部をあたかも新しい理論であるがごとく紹介してくれただけ。

>気体分子の運動量変化のマクロな視点が圧力
不正確な表現じゃないかな?圧力は本質的に面積力だから、常に
有限の大きさ(表面積)の領域を考えて議論しなければならない。
「ある領域内の分子の運動量変化の合計は、その境界に作用する圧
力の合計に一致する。」ってのは、統計力学であろうが古典力学だろう
が、成立すること。統計力学では領域を無限に小さく出来ない代わりに
ある有限時間の平均を取る。
532ご冗談でしょう?名無しさん:2001/08/05(日) 06:37 ID:???
>>530
>気体分子の引力・斥力とは圧力そのものなのでは。
>その辺がどうも引っ掛かってるのですが。

気体分子の運動を考えているのに「連続体」の概念を持ち出すから
引っかかる。
気体の中に置かれた物体の表面には絶えず分子が衝突している。
そのときの分子の運動量変化(=力積)の総和の平均が圧力。

>>531
わざわざ、そーゆーヒネクレタ考え方をすることもないでしょ。
ベル派は、揚力の原因としている翼周りの速度分布をちゃんと
説明できるの?「流体の運動の法則でそーなるんだよ」ってのが
回答になっていないのはわかってるんでしょ。

>結局、27派は、質問者の解答にはまともに答えてないし、

前スレの27番を読み直してみたら。

>ベル派の説明の一部をあたかも新しい理論であるがごとく紹介
>してくれただけ。

流れの場から循環を計算して、クッタ‐ジューコフスキーの定理で
求めた揚力は、ぢつは空気の下向きの運動量変化になってるって
ゆーのは、前スレ27のオリジナルでしょ。テキストに書いてある?
533ご冗談でしょう?名無しさん:2001/08/05(日) 19:27 ID:mrbP.AO2
>気体の中に置かれた物体の表面には絶えず分子が衝突している。
>そのときの分子の運動量変化(=力積)の総和の平均が圧力。
圧力ではなく、物体にかかる応力でしょ。物体が充分に小さい時
その応力を基準に圧力が正規化される。
圧力は、本来流体各点の微少時間の運動量変化から、その勾配が定められるが
電磁気学の電位のように、全体では定数の差の不定性があるが、上の応力の仮定
から境界条件を定めることが出来て関数としての圧力が定まる。
534ご冗談でしょう?名無しさん:2001/08/05(日) 23:12 ID:???
>>533
>圧力ではなく、物体にかかる応力でしょ。
表面の法線応力が圧力だけど、接線応力も忘れちゃいけないって
ことかな。つまり粘性を無視してはいけないとゆーことなら、その通り。

でも気体分子の運動を考えているときに、連続体(流体)の概念を
持ち込むのはダメでしょ。「分子≠流体の微小部分」なんだけど、
これがアヤフヤだからベル派は27派の説明に納得できないのかな。
535流れ曲げ派:2001/08/05(日) 23:24 ID:9A3LdeGE
やっぱ、空気が静止してる座標系での翼の受ける力を考えた方が
良くない?
536ご冗談でしょう?名無しさん:2001/08/06(月) 01:52 ID:r8v8TxfA
>>532

>流れの場から循環を計算して、クッタ‐ジューコフスキーの定理で
>求めた揚力は、ぢつは空気の下向きの運動量変化になってるって
>ゆーのは、前スレ27のオリジナルでしょ。テキストに書いてある?

これってブラウジウスの第一公式が言ってる事じゃないかな?
537ご冗談でしょう?名無しさん:2001/08/06(月) 03:33 ID:???
>>536
ブラジウスの公式は、渦無しの非圧縮性完全流体から物体が受ける
力を与える式。クッタ‐ジューコフスキーの定理はブラジウスの公
式からも導けるけど、こっちは圧縮性の粘性流体に拡張して適用で
きることが示されている。で、ブラジウスでもクッタ‐ジューコフ
スキーでも、その物理的な解釈が「揚力=流体の単位時間当たりの
運動量変化」になるってことなんだが、流体力学のテキストでは
あまりそのようには説明されていないと思う。

翼周りの速度分布を流体の運動の法則から計算して、それで求めた
揚力も、結局、流体の下向きの運動量変化になっていると解釈でき
るわけで、やはり27派の主張が揚力の本質でしょう。
538ご冗談でしょう?名無しさん:2001/08/06(月) 14:37 ID:jdsLI2IQ
運動量保存則は力を及ぼし合う物体同士の力が勝手な値を取ることが
出来ずに、一定の制限があるということを示す法則。物体の数が増える
と決して単純な法則であるとは言えない。3つ以上の物体からなる系
では、一つの運動量変化が分かったからと言って、他の2つがわかる
わけでもない。つまり力がどのようにかかるかについては何も言わない
結果論的な法則。
ここで問題になっているのは、どのようにして、力が働いているのか
を議論しているのであって、それが当然運動量保存を満たしていなけ
ればならないことから、その大きさが推定出来ること自体は高い価値は
持つが議論の本筋からは離れたことじゃないかな?
539536:2001/08/06(月) 17:35 ID:WsN8k4pc
>>537
なるほど、圧縮・非圧縮の違いがあるんですね。
>ブラジウスでもクッタ‐ジューコフ
>スキーでも、その物理的な解釈が「揚力=流体の単位時間当たりの
>運動量変化」になるってことなんだが、流体力学のテキストでは
>あまりそのようには説明されていないと思う。

って、「揚力=流体の単位時間当たりの運動量変化」になる事を用いずに
ブラウジウスの公式を求めることも可能なんでしょうか?自分はこの求め
方しか知りませんでした。

まあここでのベル派と27派のどちらが揚力の本質であると考えるかは
個人の好みじゃないですかねえ。
翼にかかっている力に注目するならベル派の圧力差のほうがわかりやすいし、
運動量に注目するなら27派って事じゃないでしょうか。
540ご冗談でしょう?名無しさん:2001/08/06(月) 21:36 ID:???
>>538
問題をスリ替えてはいけない。
「何故、力が働くのか」が問題なのであって、「どのように力が
働くのか」ではない。ベル派は後者に対しては、かなり良い説明
を与えるが、前者に対する回答にはなっていない。

>>539 
個人の好みかねぇ。
ベル派の「運動の法則があるから揚力が発生する」って「説明」で
満足できるっとこと?
「転がっている車輪は何故倒れないか」とか「逆立ちゴマは何故
逆立ちするのか」とかだと「それって運動の法則じゃん」では
ダメでしょ。翼の揚力だけ特別なのは変だと思うが。

>「揚力=流体の単位時間当たりの運動量変化」になる事を用いずに
>ブラウジウスの公式を求めることも可能なんでしょうか?

「圧力」でやるんじゃないの? 流体の運動量を使って導くのが
主流であれば、27派はもっとスンナリ受け入れられると思うよ。
541536:2001/08/06(月) 23:19 ID:xDZgwTlc
>>540
まあ自分としては出発渦や翼後縁での剥離あたりを含めて説明してあれば
いわゆるベル派で結構だと感じます。
あと、
>「圧力」でやるんじゃないの? 流体の運動量を使って導くのが
>主流であれば、27派はもっとスンナリ受け入れられると思うよ。
ってことですが、自分の場合は運動量を使って計算するやり方でした。
542538:2001/08/06(月) 23:39 ID:jdsLI2IQ
>>540
その通り。何故力が働くのか?という問いに関しては運動量保存則
は余り良い答を出してくれないわけ。ある瞬間に於ける関係しか
言わないからね。相互に働く力がどのように変化するかについての
情報を消去した法則だからね。
「相互作用がある」という程度の情報しかくれないでしょ。運動量
保存則だけじゃ。それだけなら、0の相互作用も含んでしまうわけ
よ。
543ご冗談でしょう?名無しさん:2001/08/07(火) 00:17 ID:???
>>541
では、いわゆるベル派の揚力の説明を書いてみて。

>>542
ベル派と27派を間違えていないか?
「運動量保存則」じゃなくて「運動量の変化=力積」だが、
そんな定性的な関係式ではない。
544536:2001/08/07(火) 00:42 ID:FNpJwT9g
>>543
過去スレから言われてるやつで別に目新しい点はないですよ。
後縁を曲がりこめずに剥離して、この状態を見ると循環が存在しており、
ってやつです。
この説明だと粘性の影響が入ってる点が優れてると感じます。
545ご冗談でしょう?名無しさん:2001/08/07(火) 01:49 ID:iqZFsSpY
>544
そんなに一枚板な説明じゃないようだよ。
圧力と流速の間の制限関係(要するにベルヌーイの定理ね)と
翼上面で実測される流速とを用いて、圧力低下を導いて、下面の
圧力との差を応力として揚力を説明することだと思ったんだけど
前スレ27がクッタージューコフスキーとか持ち出して来たから
色々と意見が分裂している状況。

逆に前スレ27派の説明は、基本的には飛行機の荷重を空気が受ける
時に、それがすべて運動量に転換されて下向きの流れが出来るという
指摘。上面の下降気流が本質とか、クッタージューコフスキーの話の
関係はどうやら、付属オプションみたいなもので、良く調べると
ベル派の説明の特殊な場合における精密化みたい。
前スレ27の批判の中心は、翼上面の気流の速度分布(ベルヌーイ
の定理を使うのなら、圧力分布と同義か?)が巧く説明できず万人
を納得させる説明は結局、流体の基礎方程式が完全に解けなければ
ダメという点のよう。確かにそうだが、前スレ27の運動量説は
飛躍している点が多そうな気もする。

クッタージューコフスキーの話は、ダランベールのパラドックスの
数学的証明に絡んだ話だから、地面の影響とか、圧力の正値性とか
の物理問題としての必須条件を無視しており、飛行機の翼の話と無理に関連付けないほうが
良いと思うが。
546Ledの教え子:2001/08/07(火) 02:45 ID:???
単純な話
単位時間当たりに通過する気体面積の差と気体粘性によって揚力が発生しまふ。(笑
547ご冗談でしょう?名無しさん:2001/08/07(火) 08:25 ID:???
>>544
だから、ただ剥離するだけじゃダメで、下降気流にならないと循環
は生じないよ。

>>545
ベル派の説明がダメなところはその通り。

クッタ‐ジューコフスキーの定理は、単位時間当たりの流体の運動
量変化と同等であるとゆーことなんだが。だから、考えられたとき
は非圧縮性完全流体だったけど、その後、圧縮性粘性流体にも拡張
して使えることが示された。

循環の定義を考えれば、それは翼の進行方向に対して直角な速度
成分が、周回積分のパスの内部でどれだけ生じたのかを求めてい
ることになっている。翼の周りで発生した下降気流そのものだな。
これをストークスの定理を使って rot v の面積積分に置き換えて
「渦と関係がある」なんてことを強調したりすると、混乱して
「発進渦があるから飛行機は飛ぶ」なんて思い込むヤツが出てくる。

>前スレ27の運動量説は飛躍している点が多そうな気もする。

気体分子の運動を考えて、それを統計力学的に扱ってマクロな物理
量を求めるってのが「飛躍」か?

日常の流体の現象を考えるのに、気体分子の運動から扱っていたの
では大変だから、空気を連続体とみなす「飛躍」をしてマクロな物
理量だけで計算できるように体系化されたのが流体力学でしょ。
ってゆーか、100年前には「空気の分子」なんて概念は一般的では
なかったから、連続体として扱うしか手段はなかったんだな。
548ご冗談でしょう?名無しさん:2001/08/07(火) 15:02 ID:/dSbcrZE
>気体分子の運動を考えて、それを統計力学的に扱ってマクロな物理
>量を求めるってのが「飛躍」か?
実際には、統計力学的に扱って無い。
「翼の周りの空気」という巨大な単一の「モノ」と、翼しか見ていない。
これって翼と空気の2つの関係しか見ていない単細胞的発想。
どこが統計力学なの?地面という存在や、重力の存在、気圧の非負性と気圧が
空気の荷重を支えているという仮定を全く取り入れていない。
クッター・ジューコフスキーの定理でも、恐らくそうでしょう。あれって
飛行機の揚力の説明が目的の理論じゃないからね。非粘性圧縮流体に拡張された
と言っても、少なくとも2次元の話だろうし、極端に理想化された条件の元でしょ。

運動量変化や、循環云々を一生懸命覚えたところで、
「何故」翼と空気の間に力が働くか?という問いの答えは得られないと思うぜ。

飛行機(超音速は除く)の後ろには、確かに下降成分の運動量変化が見られ
それは、渦に成長する。(渦面は進行方向に垂直で、左舷・右舷に互いに
逆向きのヤツが2つ出来る。)
「渦」全体の運動量や、運動エネルギーの増減と、飛行機の揚力を結びつけて
定量する方法は有効だが、それと揚力発生理由(より単純な原因への還元)
はまた別の話だと思うんだが。
549ご冗談でしょう?名無しさん:2001/08/07(火) 19:57 ID:???
>>548
「空気は連続体で、分子なんて存在しない」ってハズカシイ思い込み
は捨てた方が良いよ。

>「何故」翼と空気の間に力が働くか?という問いの答えは得られないと思うぜ。

思うのは勝手だが、「自分の理解できないことは間違い」って態度
は良くないな。

口をコップにピッタリつけて中の空気を吸うと、コップは口に
吸い付くよね。これは当然、内側の圧力が低くなっているから
なんだが、これもやはり「地面という存在」とかを考えないと
ダメなのかい? コップと「空気の分子」の間にどのような相
互作用があるのかを考えて、統計力学的に扱えばそれで解決す
る問題だよ。

最後の部分は、翼端渦と「循環の渦」を混同してるね。
>>547とかを読んでね。
ってゆーか、スレ全体をもっとちゃんと読んでからレスして欲しい。
550536:2001/08/07(火) 23:32 ID:ELH5WWI.
>>547
下降気流が起こってることに関しては否定するつもりはありません。
循環があるのは単純によどみ点の移動から分かるしそれで良いんではないかと。

やはりどちらの説明でもなぜ下降気流が起こるのか、循環が起こるのかという部分
が難所みたいですね。
551548:2001/08/08(水) 02:09 ID:unYGbpfM
>>549
そりゃ、コップと人間の肺の関係は、真空のコップと空気の入った
コップを2つ合わせて間にガラス板でも噛ませたのと同じ関係で、
結局のところ、人間がコップの中の空気を吸い込んだ状態ってのは
ガラス板を噛ませた2つのコップ(一方は真空)の上下をくるっと
引っくり返したのと同じで、真空の方にガラス板が押し付けられる
関係は変わらないでしょーに。縦のモノを横にしたな話。
飛行機と翼の関係とは違うでしょう。(原理は似ているが)
その原理を飛行機の翼に延長すると、結局は圧力差のことを言って
いるように思えるのだが。
圧力差ではなく、運動量変化(特に循環)が本質であり、それで
全て説明出来るみたいなこと言っていたのは誰だったっけ?

分子等の概念をなるべく排除した上で分子や原子などの統計力学
的概念(超対象性の力学)を導入したほうが、両者の為になるから
意図的に統計力学的手法を排除して考えるという態度は、統計力学
から量子力学(場の理論)へ移行した歴史から学ぶ賢明な態度じゃな
いかな?
552548:2001/08/08(水) 02:19 ID:unYGbpfM
あと、翼端渦と循環を混同しているのは、私ではない。
空気全体はそれが得る応力(重力・地面や翼から受ける力の和)が
内部の圧力分布を変えることによって刻一刻とその速度分布を
変えながら運動している。翼から受ける力は、100%空気の運動
に変換されるわけではなく、一部は地球表面にも作用する。
雪を踏みながら歩くとき、雪面の雪がつぶれる(=下に落ちる)が
人間の荷重は雪にのみ支えられるとでも思うのは普通では無い。

クッタージューコフスキーの定理:密度・速さ・循環=揚力に
ついてだが、
密度・速さを、翼周りの空気の質量、循環を一様流からのズレと
解釈すると、確かに運動量変化を表す式のように感じられるが、
本当にその考え方で良いのだろうか?
553ご冗談でしょう?名無しさん:2001/08/08(水) 08:48 ID:???
>>550
なぜ下降気流が生じるのか説明してね。

27派は解決済みだよ。難所になっているのはベル派でしょ。

>>551
だから、「圧力」が生じるのはどーしてだって聞いてるんだが…
「非常に多い数」の気体の分子が壁面に衝突したときの運動量
変化の「総和」だろ。学校で習わなかったのかい?
分子の平均速度(=温度)が同じであれば、圧力差が生じるのは
単位時間に衝突する分子の数が違うからだな。

>>552
「空気は連続体」って思い込んでいるところがイタイぜ。
なんで流体力学から離れてモノを考えられない?
そーか、「統計力学的手法を排除して考える態度」だからか(藁

「クッタ‐ジューコフスキーの定理=単位時間の運動量変化」
はオッケーでしょ。
循環の定義を「渦」で考えずにスナオに解釈すれば出てくる。
これから流行る説明かもな。そしたら2chの貢献度は大きいぞ(藁
554ご冗談でしょう?名無しさん:2001/08/08(水) 09:41 ID:GlA3XftY
循環は、電波(藁)で言えば周波数みたいなもので、ある特定の
「周波数」の振幅の分布によって、値が決まる。
渦無し障害物無し有限速度、粘性無し、非圧縮性、2次元平面全体
が領域、という条件の塊みたいな世界では、循環は常に0です。

循環は直接は運動量変化とは結びつきません。(それに密度等を
かけたとしても)実際次元を考えてごらんなされ。
次元は速度かける長さでしょう。
密度かける速さかける速度かける長さ=kg/m^2 m^3/t^2=kg m/t^2
これで力と同じ次元になった。
循環を運動量変化(特に下降成分)と結びつけないほうが良い
ということがこれでわかったね。似ていて異なるものです。
555ご冗談でしょう?名無しさん:2001/08/08(水) 10:05 ID:???
>>554
運動量変化と同等と言ってるのはクッタ‐ジューコフスキーの定理で、
循環そのものではない。よって、このイチャモンは却下。

周回積分のパスの中で流れが曲がっていれば循環はゼロじゃない。
流れが曲がるってのは、つまり、直交する速度成分(=下降気流)
があるってこと。

循環が「周波数」みたいなものってのは、何を言いたいのかわからんぞ。
556ご冗談でしょう?名無しさん:2001/08/08(水) 17:34 ID:GlA3XftY
>周回積分のパスの中で流れが曲がっていれば循環はゼロじゃない。
>流れが曲がるってのは、つまり、直交する速度成分(=下降気流)
>があるってこと。
これは、2次元の世界で、充分遠方で一様流、かつ翼表面以外境界
が無い特別な場合について言えるが、それ以外の場合は果たして言
えるかどうか。単純に流れが曲がるだけじゃ循環は発生しない。
流れを複数(無限個であることも否定しない)の組み合わせで表し
その特定の流れのみが、揚力を発生させるってのが、ジューコフス
キーの教え。
特定の成分の大きさが0でなければ揚力が発生し、そうでなければ
発生しない。
揚力を発生させるとは、その特定の成分が0でない流れを発生させる
ということ。(何故発生するかははっきり言ってわからない。ただ
コツというかノウハウはあるのでしょう)


>
>
557ご冗談でしょう?名無しさん:2001/08/08(水) 19:19 ID:???
>>556
「流れを複数の組み合わせで表し」って、流れの場を2次元のベク
トルで考えてるんだから、x成分とy成分で表せば十分でしょ。
x軸を翼の進む向きにとれば、周回積分のパスから出て行くときに
y成分を持っていれば循環はあるよ。

>何故発生するかははっきり言ってわからない。

なんだ。結局「ベル派には、何故飛行機が飛ぶのか説明できない」
ってことを言いたかったのか。

やはり、ベル派にしろ標準理論派にしろ、流体力学のみでは翼の
揚力は説明できないってのが、このスレ的な結論で良いのかな。
「こんな説明はどーだ」ってのはある?

翼の揚力の本質は、気体(分子)の運動量を変化させた力積の反作用だよ。
558ご冗談でしょう?名無しさん:2001/08/08(水) 22:07 ID:yCQ6CNPA
↑分子の直径は約8000kmで、中に地球という巨大分子を含むもの
と思われ。
地球と空気「分子」は中が悪く相互に蟲し合う仲なので、空気「分子」
だけが運動を変化させるのです。
559ほしいも:2001/08/08(水) 23:56 ID:xOp3xBd2
ここの議論で、飛行機がなぜ飛ぶか分かりました。
結局、翼のために翼の後方で空気の下方流が生じるために、「クッタジーコフスキーの定理により、揚力が生じる」、または「空気の運動量変化により翼に力積が与えられる」でよろしいのですね。
ベルヌイの定理を用いた場合の問題点は、翼の上面の流速が下面よりも速い場合でも、以下の場合はクッタジーコフスキーの定理と矛盾する事です。
即ち、前縁で分かれた流れが翼の後縁で丁度出会って、かつ後方へ何事もなかった様に翼の前方と全く同じ様に一様に流れていく場合は、循環はゼロになり、揚力もゼロとなります(翼から充分遠方で線積分すれば分かりやすい)。しかし、ベルヌイの定理からは揚力が生じます。
560536:2001/08/09(木) 00:42 ID:fsImDKOo
>>553
27派は解決済みとの事ですがこれのことですか?

>衝突(斥力)か引力で分子の運動の方向が下向きに変化すれば、
>揚力が発生するってことなんだけどな。

これだとなぜ下向きに変化するのかが分かりません。
561536:2001/08/09(木) 00:44 ID:fsImDKOo
>>559

>即ち、前縁で分かれた流れが翼の後縁で丁度出会って、かつ後方へ何事も
>なかった様に翼の前方と全く同じ様に一様に流れていく場合は、循環はゼロに
>なり、揚力もゼロとなります(翼から充分遠方で線積分すれば分かりやすい)。
>しかし、ベルヌイの定理からは揚力が生じます。

ってどういう状態ですか?詳しく知りたいのですが。
562ご冗談でしょう?名無しさん:2001/08/09(木) 05:57 ID:6ZKr07T2
線積分は翼の表面で行うんだヨ!
563ご冗談でしょう?名無しさん:2001/08/09(木) 07:14 ID:FUhl4rUY
ベルヌーイの定理を揚力の説明に使うこと自体は(その前提となる、
完全流体で渦無し等の条件を見落とさなければ別に問題無いし、
そもそもクッタージューコフスキーの定理自体、途中でベルヌーイ
の定理を使っている。
だから、ベルヌーイの定理とクッタージューコフスキーの定理が矛盾
する筈が無い。

「気流が曲がるから、循環が発生するんだ」みたいな論法が見受けら
れるが、これは余りにも不正確。気流の曲がり(流線の曲がり)と循
環は直接の関係は無い。
循環を0にしない流れの分布の流線は曲がっているが、0にする
流れの分布の流線で曲がっているものが存在する。
流れは揚力を与える分布と、そうでない分布の適当な和で表される。

完全流体で渦無しという条件の元で無限遠で一様流、領域は翼形状
部分以外の2次元平面全体という数学的簡単の為に極度に理想化さ
れたモデルで、翼の回りを流れが沿うという条件で、流れの分布の
成分で揚力が0で無いものがあるという主張がクッタージューコフ
スキーの定理。
気流が表面をなめらかに流れるという境界条件で、流れの分布が
100%決まり、揚力が発生するかしないかが決まるのは確かだが、
これは翼の形状に依存する。つまり翼の形状によっては揚力が0で
ないことがあり得るということ。また滑らかに流れるという条件だ
けでは揚力の発生は示せない。

このモデルの場合、密度は一定であるが、流速は2次元平面全体
に分布しており、そもそも運動量というものが巧く定義出来ない
が、循環では無く、オイラー微分を周回積分した量のほうが
構成する流体成分のある時刻の瞬間運動量変化を良く反映する筈
である。だから循環と運動量は直接は結びつかない。
564ご冗談でしょう?名無しさん:2001/08/09(木) 09:29 ID:???
>>559
細かい表現はチョト違うが、揚力が発生する理由はその通り。

>>560
ボールを壁にぶつけたら跳ね返るし、ひもをつけて引張れば、
引張った方向に動くでしょ。

>>561
翼に入る流線と翼から出る流線が全く同じって場合のことでしょ。
この時、循環を計算するとゼロになるけど、途中で流れが曲がって
いるので速度差があり、その「圧力差」で揚力が生じるから矛盾
ってことね。559のパラドクスだな。答えは簡単でしょ。
ベル派に解決してもらおうか(藁

>>562
循環の周回積分を翼の表面で行う、とは定義されていないよ。
565ご冗談でしょう?名無しさん:2001/08/09(木) 09:52 ID:???
>>563
>だから、ベルヌーイの定理とクッタージューコフスキーの定理が矛盾
>する筈が無い。

だったら、>>559のパラドクスを解決してみてね。

>気流の曲がり(流線の曲がり)と循環は直接の関係は無い。

速度の周回積分が循環なんだから、大いに関係しているよ。
途中で気流が曲がっても、出て行くときに入ってきたのと同じ
になっていたら循環はゼロだけどな。

>また滑らかに流れるという条件だけでは揚力の発生は示せない。

クッタの条件だけではダメってことだな。あと何があれば良い?

>このモデルの場合、密度は一定であるが、流速は2次元平面全体
>に分布しており、そもそも運動量というものが巧く定義出来ない

飛行機は概ね静止した空気中を飛んでるんだぜ。風洞実験しか頭に
ないから「流速が平面全体に分布する」なんて思うのかな?
で、なんでそれだと「運動量がうまく定義できない」の?

>だから循環と運動量は直接は結びつかない。

運動量変化と関係するのは 空気の密度×翼の速さ×循環
クッタ‐ジューコフスキーの定理だよ。循環は簡単に書くと
 (流速の下向き成分)×(下向き成分がある部分の長さ)
だから、密度×翼の速さ×(下向き成分がある部分の長さ)が、
単位時間に下向きに流れた空気の質量になってるでしょ。
それに流速の下向き成分をかければ、運動量の下降成分になるよ。
566ご冗談でしょう?名無しさん:2001/08/09(木) 15:24 ID:FUhl4rUY
↑の主張では、循環はy軸のみに関係する量で、x軸には無関係
であることを主張している様(藁)
ある閉曲線の回りの循環は、速度分布v(x,y)をその曲線の回りに
戦績分したもの。スカラー量。
流速の下向き成分(0,-Vy)と曲線の接線ベクトル(dx,dy)のy軸成分
(0,dy)との内積-Vy dyを曲線に沿った戦績分で総和した量が、
↑が「欲しい」物理量の筈。こっちのほうが、「下降成分の大きさ」
を如実に反映する。

速度分布が(0,e^(-x^2)sin(y))とかになっている分布があるとする。
この時、平面全体の運動量は巧く定義出来ない。
x成分は勿論0になるが、y成分のほうは、積分を計算する時の
極限の取り方によって負の値にも0にも正の値にもなり得る。
無限の領域での話だからなね。

クッタージューコフスキーの定理は、流速からベルヌーイの定理より
圧力を求めて、翼表面で積分した量を循環で表す公式。
速度差理論は、それを簡単に説明したもの。
ベルヌーイの定理を使っている。

特殊な条件の元とはいえども、クッタージューコフスキーの理論の
ほうがそりゃ精密に計算しているんだから、違いは出て当然でしょ。
近似じゃ0でない揚力が、実は0だったなんてことは大いにあり得る。
精密な計算を(特殊な状況で)出来る理論が、近似理論に不正確だ
なんてイチャモン付けたって意味ないぜ。
567ご冗談でしょう?名無しさん:2001/08/09(木) 15:36 ID:???
「研究者(2001年決定版)」
長所:自分のやりたいことが仕事になるのでうれしい。
短所:会社員よりもお金が儲からない。非常に難しい。研究者
   になるまでの リスクがあまりにも多い。失敗すると今ま
   で積み上げて きた努力や地位や学歴がすべて消滅して、
   ただ年を喰っただけの丸裸の自分が残る。
   社交性が身に付かず、また結婚の可能性も薄いし、相手
   の選択の幅もかなり狭い。ただ暗い。運の占める割合が
   大きい。苦しい。将来の見通しがたたない。ここまで来て
   才能のなさがわかっても引き返せず鬱な人生をおくる。
   こんなに難しい職業なのに医者や弁護士に比べてたいして
   尊敬されない。etc..。

   こうなりました。これが日本における研究者の実体なのです!
568536:2001/08/09(木) 18:32 ID:0FoP1422
>>564
>ボールを壁にぶつけたら跳ね返るし、ひもをつけて引張れば、
>引張った方向に動くでしょ。

いや、だからこの説明だと別に下向きに曲がらなくてもいいわけで。
たとえば円柱周り(回転なし)だと流れが曲がったりしないですよね。
なぜ流れが曲がるのかという点がまったく無視されています。

>この時、循環を計算するとゼロになるけど、途中で流れが曲がって
>いるので速度差があり、その「圧力差」で揚力が生じる
ってベル派でも流れの速度差は循環によって説明しているのでは?
循環が0の場合はベル派の計算でも揚力は0です。
「翼の上面の流速が下面よりも速い場合でも循環はゼロ」ってのがそもそも
間違っています。というかわざと?
それと、「559のパラドクス」とやらが起こる場合の翼の断面形が具体的に
知りたいのですが。もしそういう現象が起こるのであれば興味があります。
569ご冗談でしょう?名無しさん:2001/08/09(木) 19:15 ID:???
>>566
>↑の主張では、循環はy軸のみに関係する量で、x軸には無関係
>であることを主張している様(藁)

x軸方向の一様流の循環はいくらでしょう? 答えはゼロ。わかる?
x軸方向は一周すると打消し合うんだよ。パスの内部でy成分が
発生していると、y軸方向では打消しあわないので循環はゼロ
じゃなくなる。
もちろん翼の上下で流速のx成分が異なれば循環に寄与するけど、
静止している大気中を飛行する翼には、クェットの流れみたいな
x軸方向の速さの勾配は無いでしょ。

>速度分布が(0,e^(-x^2)sin(y))とかになっている分布があるとする。
>この時、平面全体の運動量は巧く定義出来ない。

sin(y)は奇関数だぜ。全空間で積分したらゼロ。書きたかったのはcos(y)だろ(藁
で、揚力を求めるのに「平面全体の運動量」を考える必要はあるのか?
まだ「風洞実験」から抜け出せないのかい?

>近似じゃ0でない揚力が、実は0だったなんてことは大いにあり得る。

これじゃ「>>559のパラドクス」の解決にはなってないぜ。
もっとガンバレ。
570ご冗談でしょう?名無しさん:2001/08/09(木) 19:36 ID:7xu7xepo
ヘンなスレが上にきていたのでageてみた。

>>568
>たとえば円柱周り(回転なし)だと流れが曲がったりしないですよね。

えーと、これは形状が対称だからじゃないの?
ゴメン、何が疑問なのかよくわからないよ。

>「翼の上面の流速が下面よりも速い場合でも循環はゼロ」ってのがそもそも
>間違っています。というかわざと?

う〜ん、これは条件が違うよ。x軸方向の流れを考えると、例えば
平板の上下で流速のx成分が異なれば循環はゼロじゃないよ。

でも、>>559の流れの条件だと、翼の上面と下面で流速のx成分は
同じで、上面だけ+y成分と−y成分が同量生じているって場合
でしょ。このときの循環はゼロ。
y成分があるから流れの「速さ」は上面の方が速いから、ベルヌー
イの定理で静圧が低くなり揚力が発生する。

さぁ、どーする?
571曲げ派:2001/08/09(木) 22:19 ID:MDogSYXA
循環ってポテンシャルで計算する為に無理やり作った概念じゃないの?
572536:2001/08/09(木) 23:24 ID:27KFiGE.
>>570
要するに翼の形状が非対称だから流れが曲がるって事ですか?
それじゃ適当すぎるでしょ。

んでもって「>>559のパラドクス」だけど、
>翼の上面と下面で流速のx成分は
>同じで、上面だけ+y成分と−y成分が同量生じているって場合
>でしょ。このときの循環はゼロ。
って明らかに0じゃない気が。
一体どんな翼断面周りの流れを考えてるんですか?
なんか具体的な例をあげてくれると分かりやすいんですが。
573曲げ派:2001/08/09(木) 23:38 ID:MDogSYXA
仰角がついてるからですが何か?
574ご冗談でしょう?名無しさん:2001/08/10(金) 02:14 ID:???
>>572
>要するに翼の形状が非対称だから流れが曲がるって事ですか?

空気の分子ってのは沢山あるんだ。それが対称な形状の物体に対称的に衝突
したって、流れが曲がったようには見えないだろってことなんだが。
で、何がわからない?

>なんか具体的な例をあげてくれると分かりやすいんですが。

翼のまわりに横長の長方形のパス(経路)を考える。
流れは左の辺から入って右の辺から出て行く。
>>559の条件から、流れは左右の辺に直交していて、下の辺では平行。
上の辺では、流れが山なりに出てまた入ってくるが、辺と平行な速度成分は
下の辺と同じ。
このパスで速度の周回積分を行うと、直交しているので左右の辺からの
寄与はゼロ。上の辺では速度の平行成分だけが寄与するが、パスの向きが
下の辺とは反対になるので相殺してゼロ。
よって循環はゼロ。わかる?
575ご冗談でしょう?名無しさん:2001/08/10(金) 02:46 ID:qWXikxb2
問題はそれが、渦なし完全流体の流れとして、実現されるか
どうかですね。
576536:2001/08/10(金) 02:51 ID:Knd1JjXU
>>574
対称だったら流れが曲がらないのはいいんですけど、非対称だったら
曲がるのはなぜ?

あと、周回積分ですが、パスを翼に沿って設定すると循環≠0になりますよね。
揚力を計算する場合はこの翼間近の循環を使います。
これは翼周りに循環があってその外に逆の循環があるって事でしょうか。
実際にはこんな流れは無いような…
577ご冗談でしょう?名無しさん:2001/08/10(金) 09:44 ID:???
>>576
>対称だったら流れが曲がらないのはいいんですけど、非対称だったら
>曲がるのはなぜ?

ホントにわからないの? 流れの中に仰角をつけて平板を置くと、気体分子は
板に対して「非対称に衝突」してるでしょ。このときの流れは曲がってる、
つまり板に流入する方向と板から流出する方向が違うってことなんだが...

>揚力を計算する場合はこの翼間近の循環を使います。

「翼間近」ってどのくらいの距離ってテキストに出てた?
ベルヌーイの定理で計算した揚力とちょうど同じになるところか?

クッタ‐ジューコフスキーの定理では一様流の密度と流速を循環に乗じている
のだから、パスも一様流部分を含まないと変だぜ。
あと、クッタ‐ジューコフスキーの定理を圧縮性粘性流体に適用するときは、
一様流部分の密度と十分遠方の循環を用いれば良いってのも示されてるんだな。

>実際にはこんな流れは無いような…

ベルヌーイの定理もクッタ‐ジューコフスキーの定理も流れの生じる原因に
ついては何も言っていないでしょ。だから、>>559の流れの場が与えられた
ときもちゃんと揚力を計算できなければダメだよ。
578ほしいも:2001/08/10(金) 11:10 ID:6BnOfSqY
>559の件ですが、ここで想定されている流れは、よく揚力の説明で出てくるものだと思います。この流れを仮定した場合ベルヌイの法則での結果と矛盾するのは、以下の理由によると思います。
即ち、この流れは回転方向が逆の二つの渦と定常的な流れの重ね合わせで、翼の後縁当たりに、もう一つの渦が隠されているためと思われます。
 問題はクッタ‐ジューコフスキーの定理の積分はどの経路を取るのが正しいかとのことではないでしょうか?
ここで、分かりやすい様に、磁場中の電線の受ける力にたとえてみます。なぜなら、クッタ‐ジューコフスキーの定理は、電磁気学のアンペールの法則とローレンツ力の式を組み合わせた様な格好になっているからです。(ただし受ける力は逆向きです。クッタ‐ジューコフスキーの定理の導出の仕方を知らないのでよく分かりませんが、ベルヌイの法則を使っているのでつじつまが合う?)

即ち、左から右に向いた定常的な一様な磁場があり、翼のところに電線があって紙面の向こう側から手前方向へ電流が流れている場合です。もう一つの渦が翼の後縁の後ろ当たりにありますが、ここにも電線があり丁度逆方向に電流が流れていると考えてください。
二つの電線が独立ならば翼の方の電線は上向きの力を受けます。そしてもう一つの電線は逆向きの力を受け互いに離れていきます。何らかの方法(ボンドでくっつける)で二つの電線を互いに固定すると力はうち消されて、動きません。
結局、積分経路をどの様に取るかは渦が翼と一緒に動くか否かで決まり、>559の流速分布が安定的に存在する場合は互いに固定されている場合で、揚力は発生しません。
互いに固定されていない場合は翼に揚力が発生しますが、もう一つの渦が遠くへ行ってしまうので、翼の後縁付近に下向きの流れができます。
579536:2001/08/10(金) 13:19 ID:C2s3Ng/A
>>577
>「翼間近」ってどのくらいの距離ってテキストに出てた?
>ベルヌーイの定理で計算した揚力とちょうど同じになるところか?
極端な話パスを広げすぎると出発渦まで含んじゃいますよね。
あと、回転円柱が2つあっても全体で循環は0だし。
だから適度に近い所を選ぶ必要があるのでは?

>>578
やはりこのような流れは存在しないって事でいいんでしょうか。
というか出発渦そのもの?
580536:2001/08/10(金) 14:14 ID:023YuLkI
>>577
非対称だったら曲がるってのは>>559の例もありますし
少し乱暴すぎるのでは。
循環ができる理由も>>578さんの説明で為されてますよね。

と言っといて申し分けないんですがちょっと飲み込めない点が。
要するにこの翼は循環が無い状態でもいわゆるクッタ条件を
満たしてる(よどみ点が後縁と一致)ということになりますよね。
すると循環があるとクッタ条件を満たさなくなる(よどみ点が後縁
からはずれる)気がするんですがこの考えはどこが間違ってるんで
しょうか?循環があるとよどみ点がずれると単純に考えている点?
581ご冗談でしょう?名無しさん:2001/08/10(金) 15:09 ID:???
>>578
>問題はクッタ‐ジューコフスキーの定理の積分はどの経路を取るのが正しいかとのことではないでしょうか?

翼によって流れが影響される範囲でしょ。

磁場中の2本の電流の周りの磁力線と>>559の流れの流線は対応し
てないと思うが...それに2本の電線を固定すると回転のモーメ
ントが発生するよ。

>>579
>極端な話パスを広げすぎると出発渦まで含んじゃいますよね。
>あと、回転円柱が2つあっても全体で循環は0だし。

勝手に新たな条件を持ち込んで反論しても無意味だぜ。
>>559の流れだけで考えるべきだろ。

>>580
>非対称だったら曲がるってのは>>559の例もありますし
>少し乱暴すぎるのでは。

もう一度聞くけど、何が疑問なの? コンアダ効果のことか?

>循環があるとよどみ点がずれると単純に考えている点?

そう。これがいけない。
582ご冗談でしょう?名無しさん:2001/08/10(金) 18:34 ID:zLnh7htw
>翼によって流れが影響される範囲でしょ
基本的に全平面がそうなんですが...
583ご冗談でしょう?名無しさん:2001/08/10(金) 20:01 ID:???
>>582
その通りだよ。
特に粘性流体の揚力をクッタージューコフスキーの定理で計算する
ときには「無限遠での循環」を用いるってことになってるみたいだ
けど、実用的にはパスを広げていって、値の変化が無視できるくら
いの範囲なんでしょう。
でも、あまり遠くだと粘性の影響で流れが無くなったりしてチョト
困るので、流体力学で揚力を「計算」するときには圧力分布の積分
で求めるんでしょ。近似的だけど精度は悪くないとされているからね。

揚力を説明するときは27派、計算するときは標準理論派ってのが
正解だと思わない?
584ご冗談でしょう?名無しさん:2001/08/10(金) 23:17 ID:A7FCKoM2
>粘性流体の揚力をクッタージューコフスキーの定理で計算
揚力と粘性係数の依存関係を教えて下さい。
どんな関係になるか、非常に興味があります。
585ご冗談でしょう?名無しさん:2001/08/10(金) 23:23 ID:awtVtCSQ
>クッタ‐ジューコフスキーの定理を圧縮性粘性流体に適用するときは、
>一様流部分の密度と十分遠方の循環を用いれば良い

ソース、是非奇ボンヌ
586ご冗談でしょう?名無しさん:2001/08/11(土) 01:01 ID:???
>>584
オレも知りたいぞ。

>>585
例えば、物理学辞典(培風館)から引用
クッタ‐ジューコフスキーの定理
「….流体が縮む場合には,ρの代わりに一様流の密度ρ_∞を
 とればよいし(H.Glauert),粘性流体についてはΓとして
 無限遠での循環をとればよい(L.N.G.Filon).…」

しかし「粘性」にはビンカンに反応するなぁ(藁
587ご冗談でしょう?名無しさん:2001/08/11(土) 03:03 ID:5VW6QSlA
>>586
どうもありがとうございます。
粘性がある場合、一様流を考えているのですから、無限遠は
実際には無限では無く、「ある半径の円でも考えて、その上の
循環が、ある値に収束するので、その値を代用せよ」
という解釈だと思います。

で、ちょっと話の筋が良く解らなくなってしまったのですが、
要するに粘性流体や圧縮性流体でも、循環という量が解れば
揚力が「計算」出来るという話のようですね。

しかし、粘性がある例でもわかるように、十分翼から離れた
流れは殆ど一様流ですが、厳密にはごく僅かズレてます。
循環を取って集積すればそれが積もって0にならないという
ことでしょう。

で、問題は循環が発生し、翼が力積を受け続け、かつ翼が
外力としてそれと釣合う逆向きの力を同時に受けた場合、
流体だけが、翼とは反対向きの力積を受け続けることになります。
しかし、流体の質量は∞ですから、運動量変化は0ということ
になりますね。(定常状態だから当たり前ですか。)
実際、流体全体の運動量は
ρΣv(x,y)で与えられ、v(x,y)が時間に依らないから当然です。
ここでΣは積分記号のことで、平面全体に渡る和です。
これはどのように解釈すればよろしいのでしょうか?
588587:2001/08/11(土) 03:16 ID:xQ.oZ6cw
「ある半径の円でも考えて、その上の
循環が、ある値に収束するので、その値を代用せよ」



「ある半径Rの円でも考えて、その上の循環が、R->∞の時
....(以下同文)」

と直させて頂きます。
589ご冗談でしょう?名無しさん:2001/08/11(土) 04:25 ID:???
>>587
前半はグッドだよ。

>しかし、粘性がある例でもわかるように、十分翼から離れた
>流れは殆ど一様流ですが、厳密にはごく僅かズレてます。

実際には「静止」した空気中を翼が進んでいるいるわけで、翼の周りに
流れ(下降気流)が生じていても、十分離れた場所では空気の粘性の
ために流れは無くなってしまう。

>で、問題は循環が発生し、翼が力積を受け続け、かつ翼が
>外力としてそれと釣合う逆向きの力を同時に受けた場合、
>流体だけが、翼とは反対向きの力積を受け続けることになります。

ゴメン。何を言いたいのかよくわからないよ。

空気中を進んでいる翼の周りに循環が発生している状態は、すな
わち、翼の周りで空気が下向きの運動量を得ているということ。
つまり翼は空気に下向きの力を作用させているわけで、その反作用
として上向きの力(揚力)が翼に働く。
下降気流の運動エネルギーは空気の粘性のために散逸し、十分離れ
た場所では流れはなくなる。

これで納得できる?
590587:2001/08/11(土) 06:33 ID:/e..tx9w
でも、翼の周りの任意の領域の運動量変化は0なんじゃないですか?
同じ論法で。
591ご冗談でしょう?名無しさん:2001/08/11(土) 08:55 ID:???
>>590
空気の粘性はそんなに大きくないから、急激にエネルギーが散逸して
流れが消滅することはないよ。実際に下降気流も観測されてるしね。
592ご冗談でしょう?名無しさん:2001/08/11(土) 10:22 ID:EIYP2fK6
>>591
適当な翼を内側に含む領域の運動量の総和は、時間に対し
不変となりますが、これはどう解釈すればよろしい
のでしょうか?簡単の為に非粘性、非圧縮(完全流体)
で結構ですから、御説明下さい。
593ご冗談でしょう?名無しさん:2001/08/11(土) 11:26 ID:???
>>592
「運動量変化」って時間変化のことか。

もともと下向きの運動量がゼロの空気の中に翼が入ってくると、
翼の周りで空気は下向きの運動量を得る。
で、その大きさが時間に依らず一定であるってことは、翼は一定の
大きさの揚力を受けてるってことでしょ。

翼は空気に対して仕事をしながら(=空気に運動エネルギーを与えながら)
飛んでるんだよ。
594592:2001/08/11(土) 18:28 ID:8Dmj12Ho
>593
遅レスすみません。
私は翼の回りの有界領域で、流体の運動量の単純な総和を
取ると、各点で速度が変わらないのですから、運動量は常
に変わらず、つまり、運動量変化=翼が空気に与えた力積の結果
は常に0じゃないか?つまり、翼に近付く空気は、運動量の和が
変化せず流れているのでは無いか?少なくとも、一斉に下向きの
運動量を得て...ということはあり得ないのではないか?と疑問
を思ったのです。
これはおかしいですよね。揚力があるのですから。
一体どこが間違っているのか、見当がつかず、困っている次第です。
翼と空気の関係に相当お詳しいようですから、質問させて頂きます。
595ご冗談でしょう?名無しさん:2001/08/11(土) 22:44 ID:???
>>594
>一体どこが間違っているのか、見当がつかず、困っている次第です。

間違っているのは「運動量変化」を考えるときに、翼周りの空気の
速度「分布」と実際の空気の速度を混同しているから。
空気は流れているんだな。
一様に流れ込んできた空気の速度は、翼の周りでその分布に応じた
速度に変化している。つまり空気の運動量が変化してるってこと。

ベル派の新たな見解が無いのであれば、飛行機スレは終了かな。
596夏だから言わせてくれ:2001/08/12(日) 00:52 ID:???
飛ばなかったら飛行機じゃネェだろボケ
597ご冗談でしょう?名無しさん:2001/08/12(日) 04:52 ID:0iTSfH4g
>>596
はげしくガイシュツ

http://cheese.2ch.net/sci/kako/988/988866699.html
のレス76参照
598592:2001/08/12(日) 06:07 ID:d1ihL84o
>>595
またまた亀レスですが、ご説明により、何となく自分のミスに気が付きました。
私は、今まで、平面に固定された領域で速度ベクトルv(x,y)を積分したもの
つまりその領域の総運動量の時間変化は、それを単純に微分すれば良いもの
だと思っていましたが,領域が流れにつれて動くという条件を見逃していました。
領域が動くことも考慮しなければならないんですね。
でも、そういう場合の計算はどのようにやれば良いのでしょうか?
もしやり方をご存知なら、教えていただけませんか?
私は是非とも、翼付近の下向きの運動量変化=揚力であることを確認したいのです。
599ご冗談でしょう?名無しさん:2001/08/12(日) 11:31 ID:???
>>598
こんなんで良いかな。

速度ベクトル(U,0)のx軸方向の一様流に対して、長さ√2 Lの
平板を仰角45度で置いたら流れは曲げられて速度が(0,-U)になっ
たとする。使う式は「運動量変化=力積」mv' - mv = Ft。
流れに対する翼の断面積(とゆーか間口)が L なので、空気の密度をρ
とすると一秒間に翼に流れ込んで速度が変化する部分の質量はρLU。
これより、一秒間の運動量変化は
 x軸方向: 0 - ρLU U = Fx 1
 y軸方向: ρLU (-U) - 0 = Fy 1
翼が空気に対して作用する力は (Fx,Fy)=(-ρLU^2, -ρLU^2) なので、
その反作用として翼は (ρLU^2, ρLU^2) の力を受ける。
この力のy成分が揚力。x成分は抗力(の一部)。

高校の物理の期末テストに出てきそうな問題だな。
空気の分子の運動量変化で考えれば大学か。
600曲げ派:2001/08/12(日) 12:04 ID:YoEcSrlU
 y軸方向: ρLU (-U) - 0 = Fy 1
(-U)って係数省いてますね。
ρLU(-U2)
U2=sinθxU
θは仰角
になったりしない?
全然足りなかったり・・・。
601ご冗談でしょう?名無しさん:2001/08/12(日) 12:39 ID:???
>>600
キミはもっとちゃんと文章を読まなければダメだ。

>>599は流入した空気が全部下向きになるとゆー単純化した条件で、
運動量変化が揚力になるってことを示しただけ。間違っていない。
実際の空気の流れはもっと複雑だろ。
602曲げ派:2001/08/12(日) 12:52 ID:YoEcSrlU
>>601
そりゃ係数省くよりDQNじゃんかw
603ご冗談でしょう?名無しさん:2001/08/12(日) 13:47 ID:???
>602
あんたいい加減にしろよな。
ろくな事言わないくせにコテハンでチャチャだけ入れる。
だから、いつもコピペで煽られるんだよ。

「うわっ、またコイツわけわかんない事を書きに
来やがった!嵐でやってんのかよ。ボコボコにされて
退場きぼんぬ。」
604曲げ派:2001/08/12(日) 13:49 ID:YoEcSrlU
>>603
批判なら論理的、かつ物理的に願います。
605ご冗談でしょう?名無しさん:2001/08/12(日) 13:56 ID:???
>>604
そりゃ係数省くよりDQNじゃんかw
(論理的、かつ物理的に回答)
606ご冗談でしょう?名無しさん:2001/08/12(日) 16:12 ID:???
力学がちゃんとわかってないみたいだから「物理的にお願いします」って、
この場合は「物理的に接触する=直接会って下さい」ってことだろーな(藁

>>599は、空気の運動量変化の前に、傾いた平板にボールをぶつけたら
どーなるかを考えた方が良かったと思うぞ。
607ご冗談でしょう?名無しさん:2001/08/12(日) 17:20 ID:Q00UpBHY
>>598
考えている領域の各点の速度の総和を単純に時間微分したら
確かに0(定常状態だから当たり前)
オイラー微分を考えなければダメ。

時間、空間を変数とするベクトル値関数に対し、オイラー微分は
D/Dt V=(d/dt)V+(V,grad V)と定義される。
(,)はベクトルの内積
V=(V1,V2,V3)の時
(V,grad V)=V1 (δ/δx)V1+V2(δ/δx)V2+V3(δ/δx)V3
となる。ここで(δ/δx)等は偏微分記号

オイラー微分は、流れによって動いていることも考慮した微分
のことで、正にオイラー方程式の左辺そのもののこと。
オイラー微分は記号でD/Dtと書かれる。

ρD/Dt V=-grad P,div V=0が完全流体の方程式で、オイ
ラー方程式と呼ばれてます。Pは圧力をあらわすスカラー関数
で、定数差の不定性は、適当な点で正規化することによって
決定されます。

定常状態の場合はρD/DtV=(V,grad V)=-grad Pとなります。
物理的意味は、微小体積にかかる力は、圧力勾配(ベクトル量
になる!)の面積分に等しいという意味です。要するに微小部分
は、その位置における圧力勾配によって運動量を変化させる
ということですね。
必ずしも微小で無い領域の単位時間の運動量変化は、要するに
オイラー微分を積分してやれば良いのです。そしてその値は正に
その領域での各点での圧力勾配(grad P)の積分です。
領域を曲線に取り、積分を符号を考慮した線積分にしても
同じで、循環に似ています。
オイラー方定式と結びつかない限りは意味のある量ではありませ
んが、結びつくとそれは、圧力勾配をその曲線で積分した量、
圧力が連続なら、値は端点の圧力値の差となります。
これは実はオイラー方程式に限らず、ナビエーストークス方程式
等でも同様に考えることが出来ます。

特に閉曲線に積分路を取った時、興味深いことが起こります。
もし考えている領域で圧力場が連続なら、線積分の値は0という
ことになりますね。つまり、その閉曲線はその上の圧力場が連続
である限り、トータルの運動量の変化を起こさずに流れている
ことになります。逆に言えば、下降気流等で、運動量が変化して
いる場合は、圧力が不連続になる必要があるのです。
翼と空気の関係では、圧力が不連続になる可能性を秘めています。
(クッターの条件等が関係しているでしょう。)
逆に言えば、運動量が変化している場合、翼の上下に圧力の差
が存在しなければならないことを意味しています。
ただ、下降気流がある場合は、翼の上のほうが、下よりも圧力が
高くなることをまた意味しており、揚力の符号が逆になります。
領域(2次元なり3次元)全体の運動量は、ニュートンの作用
反作用の法則で下向きの運動量変化は起こっているでしょう。
(領域が非有界の場合)しかし、翼近くでは、上昇成分が観測
されても、矛盾しません。上の考察がそれを強く暗示しています。
608ご冗談でしょう?名無しさん:2001/08/12(日) 18:42 ID:???
>>607
いやぁ、なかなか良いね。最後の段落以外はオッケーでしょ。テキストに
書かれている通りだ。
でも「運動量変化と揚力の関係」を知りたい>>598に「圧力勾配」を持ち出し
て説明するものどーかな? オイラーにしろナビエ‐ストークスにしろ、
圧力分布が与えられた時に速度分布がどーなるかを表す式でしょ。肝心の
揚力は圧力分布の積分で求まるのだから、ちゃんと圧力分布を決める式を
出さないとダメじゃない。

で、
>ただ、下降気流がある場合は、翼の上のほうが、下よりも圧力が
>高くなることをまた意味しており、揚力の符号が逆になります。

そう、普通はこう考えるよね。でも実際には下降気流は存在していて、かつ
翼の上面の方が圧力が低い。やはり圧力と流れの関係は一筋縄では説明でき
ないってことかな。最終的には方程式を解かないとわからないんでしょう。
第2スレではこの辺のコンシステントな説明をずっと待っているんだが、
未だに書き込みは無し。何か良いアイデアはある?

>領域(2次元なり3次元)全体の運動量は、ニュートンの作用
>反作用の法則で下向きの運動量変化は起こっているでしょう。

結局、揚力は運動量変化でしか「説明」できのかな。

>しかし、翼近くでは、上昇成分が観測されても、矛盾しません。

翼の前縁では、確かに上昇気流になっているね。
609ご冗談でしょう?名無しさん:2001/08/12(日) 20:05 ID:hU0o04UQ
>>608
どうも。
ここで飛行機の翼と空気を議論する際(特に運動量との絡み)
には、この程度の数学的制限が無いと、空気が翼に光のように
反射するから揚力が起こるんダ!みたいな、短絡的な意見が起こり
ギャラリーの皆さんが変な勘違いを起こして、それが醸し出す
妙な電波で、現実に存在しているパイロットの皆さんの精神波動
まで乱されるのは問題だと思いまして。

付け加えたいこととして、>>607で挙げましたオイラー方定式等は
渦無しという前提条件が付いております。渦がありますと、方程式
の形自体変わります。
この場合圧力場は、速度場によって支配されます。
つまり、渦無しという気が付きにくい条件から、圧力項は実は消え
速度場だけの方程式となります。圧力は速度場から計算され、
定数の差の不定性は、領域のある一点の値を決めることによって
完全に決定されます。
その意味で、速度場の決定→圧力分布の決定→揚力の決定
→空気全体の運動量変化の推定というループに矛盾があるか
無いかを議論する方向が良いと思います。(∞の質量という壁
がありますが)
粘性/圧縮性がある無いにかかわらず、上のことは一般的に
成立します。(渦無しの場合)ですから、この枠組みで決定
される揚力に不満がある場合、渦がある場合や、半圧縮性
(要するに流体は連続領域内では、完全流体だが、翼と流体
が必ずしも接している必要が無いという条件に緩和する)
を考慮することではないでしょうか?
後者は、翼の上に空洞が発生することを認める、実は古くか
らある考え方です。
610592:2001/08/13(月) 02:40 ID:euB60.y2
帰省移動の為、超亀レスになってしまいました。
>>599=?>>608さん、>>607=?>>609さんもご丁寧に
ありがとうございます。特に608さんの説明は懇切丁寧ですね。
オイラー微分ってのは初耳でした。流体の本を買って勉強します。

で、結局翼付近をミクロに見た場合、運動量は下向きになるんでしょうか?
それとも上向きになるんでしょうか?
マクロな視点で見たら、下向きになるのは作用反作用の法則から明らか
ですけど。
下向きになると、揚力の向きが重力の向きと同じになるんですか?
それじゃ飛行機、落ちてしまいますね。
上向になるしかないでしょうけど、じゃそれは何故?ということになりますね。
本当の所はどうなるんでしょうか?
私も数学をもっと勉強して、607さんみたいに、ボクが丸一日悩んでしまった
問題をあっさり解決できるような人間になりたいです。

599さんの説明は、非常にわかり易く、太陽の光が反射して差し込
んで来るような明快さを持っていますね。私も599さんのように、自然を
無邪気に解釈できるような大らかな頭脳を持った人間になりたいと思います。
611曲げ派:2001/08/13(月) 05:11 ID:NQu0PA/6
もー、速度場が出来た時には流束が翼に対して仕事をしてしまった
後なので無意味っつってんでしょうが。
612ご冗談でしょう?名無しさん:2001/08/13(月) 11:25 ID:???
>>609
>ここで飛行機の翼と空気を議論する際(特に運動量との絡み)
>には、この程度の数学的制限が無いと、空気が翼に光のように
>反射するから揚力が起こるんダ!みたいな、短絡的な意見が起こり

これは空気を連続体と見なす流体力学的立場では「短絡的」かも
しれないけど、気体の分子運動論的な立場からは「分子の反射」と
考えても問題はないよ。

このスレ的には
 「流体力学だけで考えていると揚力の説明がうまくできない」
ってことをちゃんと否定できるかが議論の焦点。

以下は「流体力学をちょっと勘違いしているのでは?」って話。

>この場合圧力場は、速度場によって支配されます。

「支配」は調子に乗りすぎ。単にベルヌーイの定理で速度から圧力を
計算するだけ。あと、渦無しであればベルヌーイの定理の右辺の
定数は流線に依らずに全領域で同じになるってことでしょ。

>速度場の決定→圧力分布の決定→揚力の決定
>→空気全体の運動量変化の推定というループ

この手続き自体に矛盾は無いけど、残念ながらループにはなっていないよ。
「始めにある流れを与えて計算したら圧力分布がわかります」ってことで、
「どーしてそのような流れになるの?」ってことはわからない。

うず無しの仮定をすれば、速度は速度ポテンシャルから決定され、
オイラーの方程式とかの運動方程式を使う必要は無いってことで、
「方程式の形自体が変わる」わけではない。
さらに非圧縮だとすれば、速度ポテンシャルは簡単な調和関数に
なってるから、流れをニランでその関数を適当に決めてやれば良
いってだけの話。
613ご冗談でしょう?名無しさん:2001/08/13(月) 20:22 ID:nTv0iT7Q
>気体の分子運動論的な立場からは「分子の反射」と
>考えても問題はないよ。

気体分子運動論から、最も流体力学的なマグナス効果(=クッタージューコフスキー
の定理に関連)を説明するのは最も迂遠なやり方。巧くない。
むしろ、空洞理論のほうが、気体分子運動論と相性が良いし、かかる揚力も
静圧と真空の圧力差で、3%とか5%の差とは桁違いに大きい。
ただ、これでは失速という流体力学的現象の説明が難しいってところか。
614ご冗談でしょう?名無しさん:2001/08/13(月) 21:34 ID:???
>>613
分子運動論で揚力を説明すると、どの辺が「迂遠」で「巧くない」のかな?
統計力学を使うところ? でもちゃんと説明できるよね。

>空洞理論のほうが、気体分子運動論と相性が良いし、

相性が良いってのはホントかい? 気体分子はパチンコ玉とは違うよ。
空洞理論って粘着の条件とか粘性とかを無視してるんじゃないの?
615ご冗談でしょう?名無しさん:2001/08/13(月) 22:34 ID:WzDxfhzk
>>614=612?
前のスレッドの先頭のほうでの議論は、揚力がマグナス効果
と原理的に同じものであるかどうかが、議論になっていたが
うやむやになってしまった。
揚力はマグナス効果による圧力差だけでは説明出来ないし、
ずっと弱いという指摘がされていた。
ピンポン球や、水流にスプーンが接する場合には、粘着性
(表面張力等に類するもので、物理化学的な性質)も考慮
しなければならないのとは別の理由で、飛行機の翼にかかる
揚力がマグナス効果だけでは不満という声には同感する。
616613:2001/08/14(火) 04:03 ID:QtRh2xVU
>>614
空洞理論が、分子運動論と相性が良いというのは、翼の上の流体
要素の持つ対称性が高いから。翼の上だけを考えて、「真空」の
存在が示せれば、殆どOKだからね。
翼の上下を同時に考えなければならない一般の理論(クッタージ
ューコフスキーの定理、ベルヌーイの定理応用のマグナス効果の類)
に比べてラクに議論できる。
617ご冗談でしょう?名無しさん:2001/08/14(火) 11:04 ID:???
>>615>>616の話は前スレで既に終了してるんだが…

マグナス効果=クッタ‐ジューコフスキーの定理だよ。揚力が計算できる。
で、それが単位時間に空気が得た下向きの運動量になっていることも
示されてる。だから「流体力学」で計算した圧力による揚力も「一般力学」で
運動量変化で求めた揚力も同じってこと。

>揚力はマグナス効果による圧力差だけでは説明出来ないし、
>ずっと弱いという指摘がされていた。

これは勘違いじゃないかな。
100年前の人たちが圧力差で揚力を計算したら、とても飛べるだけの力には
なっていなかったってことでしょ。

「空洞理論」で翼の上の「真空」を示せても、その「真空」に向けて、
いろいろな方向から空気が流れ込むからダメってことだったよ。
圧力と流れの両方を同時に説明しなければいけないので、「ベル派」や
「標準理論派」は結局「方程式を解かないとわからない」んでしょ。
618ご冗談でしょう?名無しさん:2001/08/14(火) 11:30 ID:Tvu57Ru2
終わった話が次々出てきてまとまらないので、ちょっと質問。

「揚力は翼の上下に圧力差があるからです。翼の上側では流速が速いので
 ベルヌーイの定理から圧力が小さくなることが示されます」

これが揚力のありがちな「説明」でしょ。これに対して純粋に

「なぜ上側で流れが速くなるんですか?」

って疑問が生じるわけだけど、これに明快に答えてください。
ちゃんとした回答が無ければ、上のは「説明」にはなっていなくて
単に現象を記述しただけってことでしょ。
619おっさん:2001/08/14(火) 11:30 ID:/AWDfTJY
今日、又、猫を拾って来ました。
でも、少し汚らしいので猫を洗ってあげることにしました。
(う〜ん、angelっていい人)

ところが、この馬鹿猫、洗っている最中ぎゃ−ぎゃー喚いて逃げ回り、
家の中が水浸しになってしまいました。
ふざけるなっつ〜の!!!
人の親切を仇で返すとんでもないバカ猫です。

でも、人のいいangelは怒ったりしません。
水に濡れたままでは可哀想なので、
620ご冗談でしょう?名無しさん:2001/08/14(火) 14:21 ID:eZKYZUzY
>「空洞理論」で翼の上の「真空」を示せても、その「真空」に向けて、
いろいろな方向から空気が流れ込むからダメ

じゃ「真空」が示せたわけじゃないんじゃん!日本語になってない
よ!
621おっさん:2001/08/14(火) 14:27 ID:vCyEpYZk
愛誤達の協力で超人気スレッドも第三弾に突入いたしました。

ではスタート

「猫の手を借りたくて一本もぎました」

「た」
622ご冗談でしょう?名無しさん:2001/08/14(火) 16:58 ID:P0bXTGIM
ともかく何で曲がるのか何で加速するのかはっきりしてくれ。
623おっさん:2001/08/14(火) 17:03 ID:???
バッタの足と触覚抜いて芋虫にしたことあるよ
624ご冗談でしょう?名無しさん:2001/08/14(火) 17:31 ID:???
>>620
「真空」を示せても、何でまわりの空気が吸い寄せられずに、
翼だけが選択的に吸い寄せられるのかも言わないとダメってこと。

>>622
何で曲がるのか <= 空気の分子の斥力と引力
何で加速するのか <= これまでのところ説明は無し

引き続き>>618の質問の回答をお待ちしております。
でも、アガってるとキモイ書き込みがあるので、
しばらくはsageてね。
625ご冗談でしょう?名無しさん:2001/08/14(火) 20:57 ID:???
引力斥力って翼が止まってても働いてんじゃん
626ご冗談でしょう?名無しさん:2001/08/14(火) 22:45 ID:???
>>625
そーだよ。だから「大気圧」があるんだろ。
レベル低すぎのコドモはいらない...
>>618の回答をミンナで考えようぜ。
627ご冗談でしょう?名無しさん:2001/08/15(水) 16:38 ID:???
プロペラがいっぱいついた太陽電池で飛ぶ細長い飛行機がニュースで出てましたね。
火星で飛ばすとか...

ところでこの飛行機、尾翼とかもないし
感覚的に前転・バック転方向に安定性がないような気がするんですが、
どうなってるのでしょうか?
飛行機のこと知らないので教えて下さい。
628ご冗談でしょう?名無しさん:2001/08/15(水) 18:27 ID:6tACXBR.
age
629曲げ派:2001/08/15(水) 20:52 ID:AMMvB9yA
>>627
見たこと無いけど無尾翼機でしょ?
主翼のトルクを打ち消せる安定した構造があるので水平尾翼は
いりません。
水平尾翼が無いのは制御でやってるんだと思います。
630627:2001/08/15(水) 21:14 ID:???
>>629
写真探してきました。
英語が読めないので何かいてるのかわかりませんが...
http://www.dfrc.nasa.gov/Projects/Erast/helios.html
631曲げ派:2001/08/15(水) 21:51 ID:AMMvB9yA
後退角もなけりゃエルロンもない・・・。
プロペラの前方配置と、重心と上半角だけでバランス取ってるのか・・。

この下に突き出た垂直翼でバランス取ってるんだろうね。

ひでー飛行機。
632チクリだが:2001/08/15(水) 21:52 ID:???
http://salad.2ch.net/test/read.cgi?bbs=ymag&key=997850864
今日11時少女漫画版に攻撃を仕掛けるとかほざくヘタレがいます
どうぞ叩いてください
633ご冗談でしょう?名無しさん:2001/08/15(水) 23:27 ID:???
>>631
やはり君は何も判っていないのに文句だけたれるアホウだな。
ヘリオスのミッションと、材料、動力等の制約条件を考えて
み給え。

>>627
そういう話し俺も大好きなんだが、ここではちょっと場違いだよ。
航空・船舶板でやると良いと思うよ。既に過去ログ倉庫に入って
今だhtml化さてれないが、下記7/16辺りの記述から見てみると
よろしい。向こうの質問スレでやるか、盛り上げるつもりなら新
スレ立てれば宜しい。
http://mentai.2ch.net/test/read.cgi?bbs=space&key=995078800&ls=50
634おっさん:2001/08/16(木) 02:24 ID:???
イランでハッテンなんかしたら即効射殺されるってば。

ちなみに日本の銭湯とかでイラン人見たことある人いる?
あいつらって絶対チンコ隠すらしいけど、実際のところどうよ?
635627:2001/08/16(木) 03:26 ID:TpEstOa2
>>633
確かにこのスレの題目からずれる話題かもしれませんね。
でも、新しくスレ作っても高度な話になると相手できなくなるしなぁ...
まぁ、とりあえず基本から学んで行こうと思います。
636ご冗談でしょう?名無しさん:2001/08/16(木) 14:53 ID:???
筋が読めん

27派(分子間引力が原因の下降気流反動派)=曲げ派?
ベル派=標準派(翼上空気水平成分速度不変により、平均速度が大きいから
ベルヌーイより下面より上面の圧力が下がる)
折衷派、空洞派って何?これが謎。中間当たりから出てきていて、前スレ

でも、良く読めば中間当たりから出てきてる。
クッタージューコフスキーの定理って、ベル派(標準派)なの?
それとも27派なの?(下降気流云々とは、無関係だというレスがこの
スレで最近出たみたいだが)
637ご冗談でしょう?名無しさん:2001/08/16(木) 17:43 ID:???
>>636

27派は揚力の原因を一般力学と統計力学で説明できている。
ベル派は>>618の疑問にちゃんと答えられないでいる。つまり揚力の原因を
説明できていない。

クッタ‐ジューコフスキーの定理は、閉領域内で生じた空気の下向の運動量
変化を表す。流れの運動の法則でクッタ‐ジューコフスキーの定理を導いた
のなら、27派とベル派の「計算」は同等。
ベル派が「説明」できないのは「翼のまわりの流れが何でそーなるの?」ってこと。

もちろん27派だって、流れの詳細を説明するのは大変。だから揚力の説明には
「流れ」を持ち出したりしない。だけどベル派は、そーなる原因がよくわかって
いないにもかかわらず、「流れ」で揚力を「説明したつもり」になっているのが
ダメなんでしょ。
638ご冗談でしょう?名無しさん:2001/08/16(木) 19:28 ID:???
>>637
>クッタ‐ジューコフスキーの定理は、閉領域内で生じた

「閉領域内」はダメだな。「ある領域内」のことね。
639曲げ派:2001/08/16(木) 21:59 ID:???
流体力学派は、難しい式並べてばかりで答えを出さないのです。
揚力≒間口面積x動圧xcosθ
こんな単純な式なんだから簡単な説明が有りそう。
640ご冗談でしょう?名無しさん:2001/08/16(木) 23:10 ID:???
>>639
それは単純すぎ。
仰角をθとすれば開口面積=翼面積×sinθだから、速度が一定であれば
 揚力∝sin2θ
だな。倍角公式は知ってるよな。つまり仰角が45度のとき揚力が最大ってことだ。
普通の飛行機の最大揚力は仰角が十数度のときに得られるけど、45度よりカナリ
小さいぞ。この違いの理由を考えたことはあるのかい?
641ご冗談でしょう?名無しさん:2001/08/17(金) 00:16 ID:HdGEy4JM
翼面上の渦流による気流の剥離?
642曲げ派:2001/08/17(金) 00:43 ID:???
揚力≒翼面積×sinθxcosθ
つうのはずーっと前に書きましたがなにか?
(=翼面積x1/2sin2θ ね。)

曲げ派商店では翼面積×θでいけるくらいの角度しか扱っておりません。

失速は流体力学に上げます。
643曲げ派:2001/08/17(金) 00:45 ID:???
x動圧 抜けてるがみのがせやw
644ご冗談でしょう?名無しさん:2001/08/17(金) 02:17 ID:???
>>642
>曲げ派商店では翼面積×θでいけるくらいの角度しか扱っておりません。

仰角が大きくなると「得体の知れない力」が働いて、翼下面の「弾性衝突」
だけでは揚力を説明できなくなるってことだろ。

で、仰角が小さい場合には、その「得体の知れない力」の影響を無視しても良い
という保証はどこにあるんだい? 根拠を示してね。
645ご冗談でしょう?名無しさん:2001/08/17(金) 04:23 ID:???
>>637
領域は、無限の大きさを持たなければならない旨のことが前の
方で出ていたが、本当なの?
地面の存在や、有限の領域でも同じことが成り立つの?
646ご冗談でしょう?名無しさん:2001/08/17(金) 04:56 ID:???
>>645
「領域」って循環を計算する時の周回積分の積分経路のこと。
翼が空気に影響を及ぼしている範囲までとるべきだけど、遠くでは
変化が小さいので適当な大きさでオッケー。でも、この辺の問題が
あるので、実用的に揚力を計算するときには圧力分布を使うんでしょ。
あと、純粋な揚力を計算するときには「地面」は考えないよ。
「地面効果」とかは揚力と別の話。
647ご冗談でしょう?名無しさん:2001/08/17(金) 12:12 ID:???
>>637
そういうけど27派の説明も流れが"なぜ"曲がるか示してないぞ。
>>624の説明は"何によって"曲がるかってだけじゃないか?
648ご冗談でしょう?名無しさん:2001/08/17(金) 13:43 ID:???
>>647
まっすぐ飛んできた分子が翼の周りでぶつかったり、引張られたりすれば
その進路は曲がるってことだけど、そんなことがわからないのかい?
649Part 1:2001/08/18(土) 06:39 ID:???
クッタージューコフスキーの定理が、そのまま有限の領域の話で使えるかどうかは
よく知らないが、循環という量が何か関係あるのは間違いないと思われるが、純粋
数学の話にも直結するし、別にこれと無理に飛行機の揚力を関連付ける必要も無い
と感じるが。

圧縮性を仮定(すなわち音速が流れの速度に比べて相対的に遅いと仮定出来る。)
場合、翼が境界から十分離れていれば、翼の近くの気流と翼だけの相互作用と考えて
も良いと思われるので、作用反作用の法則から、気流の下向き運動量増加=翼が与えた
力積=揚力という等式が成り立つのは恐らく正しいんじゃないでしょうか。

しかし粘性は、一部の流れの性質が周囲に伝わる速度を「早く」する効果があるので、
(通常は圧力の波=音波がこの役割を果たす)
粘性が高い場合や、地面効果を無視出来ない程境界と翼の間が近ければ上のことか
らずれてくるのは仕方が無いでしょう。
650Part 2:2001/08/18(土) 06:40 ID:???
ところで、この運動量による揚力の理解、「飛行機が何故飛ぶか」という解答に対する
ベストな解答であるのか?

運動量変化は、明確に翼が空気に与えた力学的仕事であり、翼の運動エネルギーの
一部が転用されたことによってなされたもの。
これは、摩擦抵抗等の抗力ロスに加えて、高度維持に必要不可欠なエネルギーロスに
含まれる。しかし面白いことに、このロスは気流が速ければ速い程小さくなる。
単位時間増加の気流の(下向き速度変化による)運動エネルギー増分=1/2×翼気流の質量X(下向き速さ増分)^2で、質量は気流の速度Uに比例、速さ増分は運動量保存則
より、1/Uに比例だから。エネルギーロスは気流の速さUに反比例。

要するに、下向き運動量変化によって揚力を得る場合、速度が速ければ速いほど
高度維持に必要なエネルギーロスが小さくなる。
これがもし事実だとすれば、運動量派の説明のベル派等に無い価値が見出せることに
な利マス。
651Par3:2001/08/18(土) 06:41 ID:???
しかし、このことをもって「何故揚力が発生するか?」という問題のベストな解答とする
のは、ちょっとね。

第一、翼が気流の下向き運動量を増やす反動で揚力を得る機能を持ったオブジェク
トであるとして、では何故速度が遅い場合は、それが出来ないのかという問いに十分
答えられないことと、更に失速を説明出来ないという点。

超音速で飛んでいる場合、衝撃波失速と呼ばれる現象があるが、それを説明出来な
ければ良い説明と呼ぶのは少し...という気がするが。

そして何よりも、そんじょそこらのモノでは、どうして翼と同じような機能を持たない
のかという素朴な疑問に対して何も指針となる答えを出してくれない点。

だから、私は個人的にはこの説明を支持しません。
652Part 4:2001/08/18(土) 06:43 ID:???
「飛行機は何故飛ぶか」という疑問は、「飛行機で無いものは、一般にどうして落ちるのか」
という問題と同じ。勿論重力があるからじゃ解答じゃないよ。

どうせ、2chだし書くんだったら、思い切って次のように表現すれば、斬新かつ本質的と、
結構すんなり皆に理解してもらえたかも知れない。それなら、私はある程度支持するよ。
--------------------------------------------------
運動量派の本質(某○中風:断じて私ではありません。)
--------------------------------------------------
一般に空気中の物体は、空気との相対速度によって、上向き、下向きの力が発生する。
これを揚力と呼ぶ。
ある相対速度では、その力が0になる。その速度は物体の形状や質量に関係する。
地面に静止している場合、一般に重力と呼ばれる下向きの力になる。
一般にこの時空気との相対速度が0なので、形状には無関係で質量のみに比例する。
つまり重力は揚力の特別な場合である。但し慣習上、揚力はーgなる加速度を差し引い
て表現する。(gは重力加速度と呼ばれていた量で9.8m/sec^2)
翼は、地面と空気の相対速度が比較的小さい場合でも、揚力を0にすることが出来る
もの。
:-空気が重力を発生させる!
:-万有引力の法則は間違っている(藁)
---------------------------------------------------
万有引力の法則を否定すれば、運動量派の揚力の説明は本質的な説明として俄然輝いて
来る。
ちなみに私は万有引力の法則は間違っているとは思わないが、正しいとか真理である
とも思わない。
653曲げ派:2001/08/18(土) 10:56 ID:bAYhhZhg
>>649-652 は粘性を考えすぎて逝ってしまった人デスナ。
654曲げ派:2001/08/18(土) 18:00 ID:bAYhhZhg
>>647

曲がるわけは簡単。
@翼下面にぶつかった動圧が行き場を求めて反射する。
A反射方向は翼面に対して垂直方向。
B上面は符号が違うだけで似たような現象である。
655曲げ派 :2001/08/19(日) 12:49 ID:yToU9dCk
曲がるというか、私は運動量じゃなくて翼の受ける力で考えてるけどね。
656ご冗談でしょう?名無しさん:2001/08/19(日) 19:02 ID:U1S31qpc
>>654
たとえば翼がカマボコ型のときには?
流れに対して下面(直線部)を水平にしとくとその説明じゃ辻褄が合わない。
657ご冗談でしょう?名無しさん:2001/08/19(日) 19:20 ID:NDo9/.1.
「分子」は翼表面のごく近くでは斥力が働き、それからやや離れる
と引力が働くという、複雑な機構を仮定しなければ、上面の
引っぱり力は説明出来ないとオモワレ。
気体の種類に強く依存するの?揚力って。
658曲げ派:2001/08/19(日) 19:34 ID:yToU9dCk
>>656
そんなんで揚力発生するのかねえ?
ベル派は発生すると思ってるみたいだけど。

>>657
「自然は真空を嫌う」で説明出来ますが。
659ご冗談でしょう?名無しさん:2001/08/19(日) 22:17 ID:???
>>658
つまり空洞派と本質的に同じ原理なのですね。
660ご冗談でしょう?名無しさん:2001/08/19(日) 23:43 ID:???
>>658
別にベル派じゃなくてもその場合揚力が発生するだろ。
実際の翼型でも下面が平べったいのは存在する。どう説明すんだ?
あと、円弧翼についても説明希望。
661曲げ派:2001/08/20(月) 00:55 ID:gviD.Sdo
>>659
空洞派ってなんだろう?

>>660
両者とも大部分は仰角のおかげだと思います。
662ご冗談でしょう?名無しさん:2001/08/20(月) 02:11 ID:Ulc9pIDA
>>661
迎角のおかげ?
この場合迎角0でも揚力は存在するよ。
663曲げ派:2001/08/20(月) 21:20 ID:o6nW7Rhg
>>662
ナンセンス。
揚力の主たる物は迎え角によるもの。
そのような微小な効果を論じるのは、
車が前に進むのは排気ガスの反動が原因だと言ってるようなもの。
664ご冗談でしょう?名無しさん:2001/08/20(月) 22:20 ID:TOSrM0a2
>>663
微小な効果ってどこから出てきたでまかせだ?
キャンバー効果を知らんのか?
665曲げ派:2001/08/20(月) 22:23 ID:o6nW7Rhg
>>664
じゃあなんで三角翼は平面翼を採用してるんでしょうか?
NACA翼形はあくまでも効率を上げる補助的な役目しかないでしょう。
666ご冗談でしょう?名無しさん:2001/08/20(月) 23:21 ID:???
>>665
キミはいつも「三角翼は平面翼」に逃げるねぇ。
「微小な効果」と主張するなら>>644の質問にビシッと答えてみたら。
テキストのコピペでも構わんぜ。

>>657
>「分子」は翼表面のごく近くでは斥力が働き、それからやや離れる
>と引力が働くという、複雑な機構を仮定しなければ、

それってゴク普通の分子間力。物性のテキストは読んだこと無いのか?

>>654
>B上面は符号が違うだけで似たような現象である。

あのなぁ。翼の上面で「動圧が反射する」ってのを、詳しく説明してごらん。

>>649-652
う〜ん、「ト」んでるぞ。
part2の「ロス」は勘違いかもしれないが、part4「万有引力」は真性か。
667ご冗談でしょう?名無しさん:2001/08/20(月) 23:51 ID:3rrdJI1Q
>>665
効率を上げるったってキャンバによって迎角数度分違ってくるぞ。
微小とは言いがたいでしょ。三角翼はまた別の話じゃないか?
668ベル派:2001/08/21(火) 02:39 ID:6QI7exes
>>618の、なぜ翼の上面では下面よりも流速が速いかについてですが、
これは、単に質量の保存、または連続の式からきていると思います。
一般に翼の周りの速度分布は翼から翼厚の2−3倍程度離れると
あまり乱されず一様と考えれます。
そこで翼の前縁と後縁とを結んだ線に平行に上と下に翼厚の2倍程度の平板を置きます。
この板は上記の理由から流速分布にはほとんど影響を与えないと考えられます。
次に翼の前縁で分けられた上部と下部の流れについて、
ベルヌイの定理の導出で出てくる連続の関係を適用します。
すなわち翼の上面と上部に置いた板との間の流路のどの断面でも流量が一定であるため、
前縁に比べ流路の真ん中付近はくびれているため流速が速くなります。
このため翼の上面では圧力が低下します。
一方、翼の下面と下に置いた板との間の流路はくびれが無いため
流速は一定で、圧力も低下しません。
実際の流れは、翼に近いほど翼のカーブにより流れが影響を受けるので
翼に近いほど流速は上記で考えている平均的な流れよりも速くなるはずです。
以上の様に、定性的ですが、揚力が生じることを示すことができます。
曲げ派の説明も定性的ですので、こんなもんで良いのではないでしょうか。
669ご冗談でしょう?名無しさん:2001/08/21(火) 02:44 ID:zUZg6fz6
 空洞理論(Cavity理論)とは、流体が慣性を持つ為に、一様流では、境界のに沿って調和
して流れることが出来ない場合、領域に空洞(Cavity)を作らないように、流体が
膨張するなり、周囲から流体要素を吸引するなりして流れることを指し、特に流体が
膨張して流れる場合が重要となり、この場合圧力が低下することになる。
圧力の低下は、周辺の流体との繋がりが、境界に邪魔されて良好では無い場合、必ずしも周囲の流体を引き寄せて消滅することを意味しない。また流体の慣性も影響を与えて、流入する質量が十分でなければやはり、圧力は低下することはあっても、上昇することは
無い。
 圧力が巧く低下しない場合、つまり周囲の流体が引き寄せられて、十分に密度が低下し
ない場合、これが正に失速である。大抵はこれが起こるが、特定の条件の元では
膨張・圧力低下だけが起こる。飛行機はこれを利用する。完全流体に近い場合は、剥離
すら起こり、本当に真空になるかも知れない。(キャビテーションの発生)

分子運動論から見れば、翼の断面で後方が緩いスロープとなっており、分子運動は
地表から見て水平成分は平均して0であるから、高速に飛んでいる翼の上面に
当たる場合、垂直方向の衝突速度が低下することが、本質的な原因と考えるのであろうが
これは、見方を変えただけで、空洞理論とそれほど変わらないと思われる。
 ただ、直感的に解りやすいことは認められる。
失速も、要するに「分子」の翼から見て水平方向の速度分布が平均して、翼後方へ
偏っているから、翼両端、或いは上方から高速な「空気分子」のみが引き寄せられ
上方向からの撃力を増大させてしまうことから説明出来なくも無い。
「分子」で考えると、流体で考える時には見えなかった仮説も、予想出来るという点
で興味深い。
 ただ、流体特有の非可逆現象(時間を反転させると、方程式の形が変わってしまい
現象が時間反転に対して対称では無い)までも、分子運動で説明出来るかはわからない。
とりわけダランベールのパラドックスとの絡みでは、分子運動論はシビアな問題を孕んで
いる。「分子」に相互の速度差を認識出来るような複雑な機能を仮定すれば、時間軸に
対して非対称になり、説明出来る可能性も残されるが、普通のニュートン力学で良く取
り扱われる弾性体モデルの方程式は、時間反転に対して可逆だから色々なパラドックス
を産み出す温床となる。
670ご冗談でしょう?名無しさん:2001/08/21(火) 08:03 ID:???
>>668
やっと出てきた、ベル派の説明。少し検証してみよう。

>一般に翼の周りの速度分布は翼から翼厚の2−3倍程度離れると
>あまり乱されず一様と考えれます。

この部分の根拠は? キミの説明の中では一番大事なところだ。
それにしても「翼厚の2〜3倍」は小さくないかい。
セスナ機の翼厚って数十センチのオーダーだろ。

>前縁に比べ流路の真ん中付近はくびれているため流速が速くなります。

だから、知りたいのは「なぜ流れがくびれるのか」だよ。
くびれた部分の流速が速くなるのは当りまえ。
翼の上面と「内径の絞られた管」では境界条件が全然違うでしょ。

翼の前縁で持ち上げられた空気の流れが、後縁から出るときには下向きの
運動量成分を持つことを説明できないとダメでしょう。

>曲げ派の説明も定性的ですので、

「27派」と理論の無い「曲げ派(コテハン)」は違うぜ。
671ご冗談でしょう?名無しさん:2001/08/21(火) 08:42 ID:???
>>669
空洞理論ねぇ。

仰角のある平板翼の上面だけを考えるなら良いけど、それだけで揚力を
考えるのはダメでしょ。下面の「弾性衝突」の影響も考えないとね。

平板でない翼の場合は、さらにダメ。
「気体が膨張するから圧力が下がる」なら、翼の前縁で気流を上昇させる
時には圧縮されて圧力が上がっているでしょ。揚力はどーやって説明する?

分子運動論の「解説」は止めておいたほうが良さそうだぜ。分子間には
相互作用があるのを忘れてるよ。
あと、運動法則は可逆的だが、熱力学的なマクロな系は不可逆的。
相反定理とかエントロピーを勉強するのが先だな。
672ご冗談でしょう?名無しさん:2001/08/21(火) 13:32 ID:???
>>670
27派は円弧翼をどう説明するの?
673ご冗談でしょう?名無しさん:2001/08/21(火) 14:58 ID:???
>>672
どう説明するのって、下降気流があるんだったら揚力はあるよ。
674ご冗談でしょう?名無しさん:2001/08/21(火) 15:47 ID:nj10Z4eY
>>673
ではなぜ円弧翼で下降気流がおこるんだ?その根拠は?
675ご冗談でしょう?名無しさん:2001/08/21(火) 15:58 ID:???
>>671
正に機体の前縁で圧縮され過ぎて、空洞になるべき部分での膨張
をしても、相対負圧になれなかった 場合が失速(衝撃派失速)
です。超音速領域では、時にはそういうことが起こるそうです。
しかし、速度の遅い気流の場合、前縁での圧縮はそれほど起こらず
運動エネルギーに転換され、早い速度の上昇気流になるようです。
676ご冗談でしょう?名無しさん:2001/08/21(火) 16:08 ID:???
>>674
仰角があれば良いんじゃないの。

>>675
>しかし、速度の遅い気流の場合、前縁での圧縮はそれほど起こらず
>運動エネルギーに転換され、早い速度の上昇気流になるようです。

圧縮はそれほど起こらず、膨張は十分に起こって揚力になるって
ことかい?
その根拠を示して欲しいぞ。
677ご冗談でしょう?名無しさん:2001/08/21(火) 18:41 ID:0xTUdzO.
>>676
迎角0でも起こるよ。
678ご冗談でしょう?名無しさん:2001/08/21(火) 19:46 ID:???
>>676
ホントに? これって「>>559のパラドクス」じゃないの。

下降気流が生じていなくても、翼の上面と下面で速度差があるので揚力がゼロじゃ
なくなるってヤツでしょ。
まだ未解決だからちゃんと説明してね > ベル派
679ご冗談でしょう?名無しさん:2001/08/21(火) 21:54 ID:102RO.Qg
>>678
迎角0で揚力もありますし下降気流もありますよ。
円弧翼ならテキストにも載ってるんじゃないの?
ジューコフスキーの定理で揚力も簡単に求まるはず。
680曲げ派:2001/08/21(火) 22:14 ID:WuWr4rCw
>>679
簡単ならここに書いてね。
681ご冗談でしょう?名無しさん:2001/08/21(火) 23:11 ID:???
>>679
どんな流れになるの? 何か参考文献を紹介してちょ。

>>680
キミは、まず、>>666に答えるべき。
シッタカブリをしてるわけじゃないだろ。
682ご冗談でしょう?名無しさん:2001/08/21(火) 23:14 ID:LD7J3n9k
>>680
確かLはこんな感じじゃなかったっけ。
 L=4πURsin(α+β)
ただし、
 R:ζ平面での円の半径
 β:  〃    の中心と、円とx軸の交点を結んだ線とx軸の為す角
求め方はテキスト見てくれ。たぶん1ページくらいに収まるはず。
間違ってたら誰か訂正よろしく。  
683ご冗談でしょう?名無しさん:2001/08/21(火) 23:28 ID:???
>>682
>β:  〃    の中心と、円とx軸の交点を結んだ線とx軸の為す角

円の中心はζ平面の原点に無いってこと? αは何?
よくわからんぞ。
684ご冗談でしょう?名無しさん:2001/08/21(火) 23:37 ID:LD7J3n9k
>>681
なんて説明すりゃいいんだろう。
円弧翼についてどのくらいご存知なの?
685ご冗談でしょう?名無しさん:2001/08/21(火) 23:46 ID:???
>>684
複素速度ポテンシャルでしょ。
z平面の円弧をζ平面の円に写像してるの?
686ご冗談でしょう?名無しさん:2001/08/22(水) 00:03 ID:vCgRFUOU
>>685
>z平面の円弧をζ平面の円に写像してるの?
そうです。そんでもってζ平面の円の中心は虚軸上にあります。
この円の半径がRで、実軸と±aで交わるとすると、
 F=U(Zexp(-iα)+R^2*exp(-iα)/Z)-iΓ/(2π)logZ
 ただし、
  Z=ζ-ζ0、ζ0=i*a*tanβ
 αは通常どうり迎角です。
687ご冗談でしょう?名無しさん:2001/08/22(水) 01:36 ID:tmTyymN6
ちなみにβはz平面上で円弧翼が虚軸と交わる点と翼の前縁を結ぶ線
が実軸となす角でもあります。このβ分だけ迎角が実質増えたことに
なるわけで、曲げ派が言っているような微小な効果ではありません。
688曲げ派:2001/08/22(水) 02:01 ID:5/xtkifA
ρとvはどこいった?
689ご冗談でしょう?名無しさん:2001/08/22(水) 02:09 ID:sGTt7c6M
統計力学は必ずしも対象に離散性を仮定しない。基本的に内部自由度が高い複雑な対象の時間的統計を
論じるものであって対象を離散モデルで表すことは、専ら数学的簡単化の利益が高い時に行われる。
統計力学では、質量分布が中心となり、弾性衝突モデルを採用する場合は、とりわけ密度分布が集中的
で形を崩さない波動となるような、力学を考える。ラグランジュ座標を取って考えるのが普通だが、別に
オイラー座標を採用しても問題は無い。流体と違うのは、相互作用として圧力を取らず、複雑なポテンシ
ャルを仮定する。但しこれらは、ある時刻の質量分布で決まり、その時間変化は直接関係しない。
それを仮定することが、粘性を考えるということであるが、これは流体なんか目じゃないほど複雑な
システムとなり、この時間発展を統計して、揚力を説明しようというのは現時点では無謀である。
問題は統計力学的手法か、流体的手法かではないと思われ。。

このスレの最初から今まで問題になっているのは、何故、翼の前面に衝突した空気が圧縮されずに
むしろ圧力を減らす代わりに速度を上昇させて、翼の上を流れるかである。
翼への「衝突」の前後で、非対称的になっている。
何故圧力を上昇させて、速度を減じないのか。こういう場合もあっても問題ないのでは無いか?
というのが、ベル派、空洞理論派の主張に対する共通の疑問といったところであろう。
ベルヌーイの定理は、完全流体にしか扱えないというのが、基本ではあるが、完全流体では無くとも
圧力と速度の関係は、一方が増えれば、他方が減るという関係程度はある程度仮定出来るであろう。
もし、そうだとしても、その仮定だけでは,確かに圧力だけが減るという結論は導けないかも知れない。

ただ、空気の速度が速い場合、つまり超音速領域では、実際に圧力が前縁で急上昇し、それが回復せずに
上面を流れ失速の原因となる場合があるという。(超音速失速)
低速では、これは起こりにくいらしい。

良く考えると、何となくこれは空気の慣性で説明出来なくもなさそうだが。
690ご冗談でしょう?名無しさん:2001/08/22(水) 02:28 ID:55mJYaC6
>>689
その通りだ
691689:2001/08/22(水) 09:04 ID:cmryhdIY
>>690
何が?
692ご冗談でしょう?名無しさん:2001/08/22(水) 12:11 ID:???
>>686
仰角?
693ご冗談でしょう?名無しさん:2001/08/22(水) 12:38 ID:???
>>686>>682
計算したよ。仰角がゼロの時も確かに循環Γ=4πURsinβになるね。
で、Rsinβって、ζ平面上の円の虚軸上の中心の位置でしょ。中心が原点に
あると平板で実軸上にズレると対称翼、虚軸上にズレると円弧翼になるって
ムカシ習ったのを思い出したよ。
それで、結局は回転円柱周りの流れで考えるんだから、流れを曲げた反作用が
揚力ってことには変わりはないね。
694ご冗談でしょう?名無しさん:2001/08/22(水) 12:47 ID:iqQDrqB.
>>692
俺はきちんと迎角って書いてるぞ?
>>693
だから流れが曲がることは否定してないって。
何で曲がるかってのを聞いてる訳。
この形状だと曲げ派の理屈はおかしいでしょ。
695ご冗談でしょう?名無しさん:2001/08/22(水) 12:53 ID:???
>>689
いやぁ、良く勉強したね。いいカンジだ。けど、ちょっと惜しいな。
まず、分子間のポテンシャルはそんなに「複雑」じゃないよ。
よく使われるのはレナード=ジョーンズ型のポテンシャルだけど、
分子間距離のべき乗の項が2つあるだけ。
で、これを分子動力学的に扱うとなると現実の分子数を扱うのは絶望的。
でも統計力学的に速度分布の平均とか平均場近似で相互作用を扱う
とかすればできそーでしょ。

流体力学で「簡単に」説明できないのはわかったみたいだな。
でも「空気の慣性」だけで説明しようとするのはダメだよ。
空気の分子には斥力や引力が働くって書いたんでしょ。
696ご冗談でしょう?名無しさん:2001/08/22(水) 12:57 ID:???
>>694
回転円柱の表面に流体がくっついているから、じゃダメ?
「曲げ派(コテハン)」には「理屈」はなさそうだぜ。
697ご冗談でしょう?名無しさん:2001/08/22(水) 17:26 ID:NWwt5EYc
>>696
その説明で良いんならベル派の流れが加速する理由も問題ないんじゃないの?
698ご冗談でしょう?名無しさん:2001/08/22(水) 17:51 ID:???
>>697
回転円筒が直に速度差を生じさせるのでベル派もハッピーだな。
粘性が大事ってことでね。
でも、飛行機の翼は回転してないよね。
27派なら翼の表面に沿った流れがそのまま下降気流になるってことで
大スジはオッケーでしょう。ベル派は...やっぱり困るねぇ。
699692:2001/08/22(水) 21:25 ID:???
>>694
調べてみたらちゃんと迎角って言葉があるんだね.
大変失礼つかまつった.
700曲げ派:2001/08/22(水) 23:03 ID:Ny4yzNrY
えーと、ガイシュツだけど、揚力の大部分が迎え角によるものでは無い
という主張は、背面飛行を説明できないんでボツです。
701ご冗談でしょう?名無しさん:2001/08/22(水) 23:24 ID:CTIpPQos
>>698
求められてるのは流れが下降気流になる理由ですよ。
702ご冗談でしょう?名無しさん:2001/08/22(水) 23:29 ID:???
>分子間のポテンシャルはそんなに「複雑」じゃないよ。
流体に作用する圧力というポテンシャル量に較べればお世辞にも
単純とは言えません。:
703Ledの教え子:2001/08/22(水) 23:31 ID:???
>>700
説明出来ると思うぞ!
背面で飛んでる姿勢を考慮してね。

君達の争いって姿勢の違いで揉めてるだけね♪(不毛だと思うぞ)
704ご冗談でしょう?名無しさん:2001/08/22(水) 23:35 ID:???
>>692
ちゃんと迎角って言葉があるって言うかそう呼ぶのが普通じゃない?
仰角とは言わないと思うよ。

>>700
わざとやってる?確信犯のような気がしてきた。
705ご冗談でしょう?名無しさん:2001/08/22(水) 23:38 ID:???
>>700
キミの主張は翼の上面で生じる下降気流を説明できないのでボツ。
背面飛行だって、翼の上面の空気の流れはあるだろ。
で、>>666の質問にはちゃんと答えられないのかい?
自分の理解できることだけが正しいと思い込むのはダメだぞ。
706ご冗談でしょう?名無しさん:2001/08/22(水) 23:48 ID:???
>>701
ええと、>>698に理由が書いてあるが、何がわからない?

>>702
「圧力というポテンシャル量」ねぇ。
う〜ん、単純なのはキミの頭だな。
707ご冗談でしょう?名無しさん:2001/08/22(水) 23:58 ID:CTIpPQos
>>706
理由ってこれ↓ですか?
>翼の表面に沿った流れがそのまま下降気流になる

そもそも翼の周りに一定(クッタ条件を満たすような)の循環を仮定しないと
上のようなことは言えないですよね。ではなぜそのような循環が発生するので
しょうか?
708曲げ派:2001/08/23(木) 00:05 ID:vZy6ANgU
自己循環理論
709ご冗談でしょう?名無しさん:2001/08/23(木) 00:36 ID:???
>>707
「コアンダ効果」でいいかな。
要するに翼と空気の分子の間に働く引力だ。
ところでスレを全部読んで質問してるのかい?
激しくガイシュツってヤツだぜ。

>>708
どーした「曲げ派」。>>666の答えはまだかい?
710曲げ派:2001/08/23(木) 00:50 ID:vZy6ANgU
ん?どんな質問?
711ご冗談でしょう?名無しさん:2001/08/23(木) 01:41 ID:???
>>710
揚力のメインが翼下面の空気の弾性衝突なら、迎角が45度のときに
最大揚力になるが、実際は十数度で最大。つまり弾性衝突以外の要因
が寄与していることになる。迎角が小さいときにこの要因は無視できる
くらいに小さいとゆー主張だったでしょ。その根拠を示してね。

あと、>>654のB。翼の上面の「符合の違う動圧の反射」ってのが
意味不明だ。詳しく説明してくれ。
712曲げ派:2001/08/23(木) 01:45 ID:vZy6ANgU
B上面は符号が違うだけで似たような現象である。
713ご冗談でしょう?名無しさん:2001/08/23(木) 01:56 ID:dLZjOdcE
>>709
コアンダ効果なんだけども過去ログに目を通した限りでは諸説入り乱れてて
るんだけど、この辺でそのメカニズムについて説明しといてもらえないかな。
曲げ派はほっとけば?
714曲げ派:2001/08/23(木) 02:02 ID:vZy6ANgU
>>713
そう言うけど流体力学派は式を導けないんだぜ。
715ご冗談でしょう?名無しさん:2001/08/23(木) 02:32 ID:o6SSuPwk
つまるところ、下降気流反力派(=旧27派+曲げ派)の疑問は
標準派(ベル派=圧力と速度の間の束縛関係を中心として、粘性や
圧縮性・慣性を加味して、翼の周囲の空気の流れを説明し、圧力が
応力として翼に作用するとい前提で議論する方法)の最大の弱点
である、翼上の空気が、何故高速になるかを説明出来ないから、
運動量変化で説明する方法に転換しろということなのね。
(低速になって、圧力が増加することは、少なくともベルヌーイ
の等式だけ見る限りでは起こりえる。というか、起こらないという
ことを否定できない。位置エネルギーと運動エネルギーの和が解っている
からといって、ある位置の速度を決定できるわけでは無いことは
物理では初歩的な事実)
しかし運動量に視点を置く考え方の有効性の根拠としてコアンダ効
果や、クッタージューコフスキーの定理を挙げているが、果たしてこ
れらは適切なのだろーか。
コアンダ効果は余りにも現実的な経験則といった感じがするし、
前提条件とかよくわからない。失速という現象が存在する以上
無条件に成立するわけでもないだろうに。
クッタージューコフスキーの定理は、完全流体の2次元の無限領
域に基礎を置く話だから、飛行機の揚力の議論を運動量でしよう
という立場の人は、あくまでも参考程度に留めておくべきじゃない
か?となると、実は根拠が標準派以上に曖昧だね。
でも、標準派の人の良くある説明の曖昧さ(ベルヌーイの定理に
まつわる部分)の指摘自体は正しい。
ちょっとした言い換えだが、飛行機は何故飛ぶのか?という疑問
よりも、飛んでいる飛行機は何故突然落ちないのか?という疑問に
変換してみたら。ベルヌーイの定理だけの説明だったら、突然
落ちることも否定は出来ない。しかし、一般的に飛行機は安定
して今も飛んでいる。
716ご冗談でしょう?名無しさん:2001/08/23(木) 02:36 ID:o6SSuPwk
>位置エネルギーと運動エネルギーの和が解っている
>からといって、ある位置の速度を決定できるわけでは無いことは
>物理では初歩的な事実)
ドキュソなカキコでした。

等エネルギー面の上に軌道があることは
解ったとしても、速度の範囲については、
それ以上のことを主張できるわけではない
ということです。
717ご冗談でしょう?名無しさん:2001/08/23(木) 07:32 ID:???
>>712
それは詳しい説明じゃないだろ。
それと>>711の前半の質問に対する回答はどーした?
降参かい?

キミの書き込みは、正しい理解のジャマになるだけ。
オームの法則スレも混乱させてるしな(藁
718ご冗談でしょう?名無しさん:2001/08/23(木) 08:04 ID:???
>>713
コアンダ効果は「粘着の条件」で説明できるよ。
物体の表面に接している流体の流速=物体の速さになるってこで、
流体が表面にくっついていること。
一様な流れの中にカーブした物体を置けば、物体の表面の流速は
ゼロだから流れはカーブに沿って曲がるでしょ。

確かに「粘着の条件」は経験則だけど、気体分子が物質の表面に
吸着しているのは事実。なぜ吸着するのかは、量子力学的に扱う
必要があるので、ちょっと複雑だな。
719ご冗談でしょう?名無しさん:2001/08/23(木) 08:12 ID:???
>>715
う〜ん、一番イケナイのは、27派と「曲げ派」を一緒にしてるところ。(藁
その次は、27派の根拠がコアンダ効果やクッタ‐ジューコフスキーの定理だと
思っているところ。気体分子の斥力と引力だって書いただろ。だから
「根拠が標準派以上に曖昧」ってイチャモンは却下。

流体力学ではちゃんと説明できないという認識に至った点は
評価するよ。ちゃんと考えればわかるじゃん。
720曲げ派:2001/08/23(木) 09:42 ID:ZekokfoI
>>717
俺に言ってるの?
多体だから。
721ご冗談でしょう?名無しさん:2001/08/23(木) 12:31 ID:???
>>720
説明になってないぜ。やり直しだ。 > 曲げ派

ちゃんと説明できれば揚力の理解が深まると思うんだが、
曲げ派に期待しても無理かな。
722ご冗談でしょう?名無しさん:2001/08/23(木) 14:44 ID:dtpPExfw
一度揚力が発生したのなら、何故揚力が発生し続けなければならない
のかというのが問題。
揚力を起こさない、つまり失速を起こす解が、流体力学の基礎方程式
は勿論、統計力学で得られる方程式でもおそらく、あるのにもかかわ
らず、一度、揚力を起こす解が実現されると、他の解は現れない。
失速を起こす解に徐々に変化するだろうというのが、ダランベール
の背理(予想)でしょ。その変化が充分遅ければ、適当にパラメータ
(迎え角、速度)を変えることによって、揚力の増減を調整出来る
筈。早かったら、人間がコントロール出来なくなる。
突き詰めると、人間の精神能力が十分に高かったからということに
なるか。でも、人間より遙かに下等とされている生物ですら、飛ぶ
のは珍しくも何とも無いからなー。
723ご冗談でしょう?名無しさん:2001/08/23(木) 18:29 ID:LWyeSuGQ
なんかよく分からなくなってきたぞ。
27派の流れが曲がる根拠はコアンダ効果じゃなかったのか?
>>719では根拠は気体分子の斥力と引力って言ってるぞ。一体どっちだ。

>>718の説明も疑問が。流れが表面に沿うのは単にそうしないと
真空ができちゃうからってことだと思うんだけど。
そもそもコアンダ効果って結局経験則なの?
724ご冗談でしょう?名無しさん:2001/08/23(木) 20:48 ID:???
>>722
>一度、揚力を起こす解が実現されると、他の解は現れない。

定常解とか知ってる?
あとダランベールのパラドクスは粘性を考えれば解決。

>>723
コアンダ効果ってホントに「真空」で説明できるの?
どーして流れだけが選択的に「真空」に引き寄せられて、
周りの空気は引き寄せられないの?
やはり>>718のように「粘着の条件」と「粘性」で説明すべき。
で、それは結局、分子間の相互作用で説明できるよ。

>>719は、27派の根拠はそんな経験則じゃないってことでしょ。
725ご冗談でしょう?名無しさん:2001/08/23(木) 21:13 ID:???
>>722
>一度、揚力を起こす解が実現されると、他の解は現れない。

定常解とか知ってる?
あと、ダランベールのパラドクスは、粘性を考えれば解決。

>>723
コアンダ効果はホントに「真空」で説明できるの?
周りの空気が「真空」に引き寄せられないで、流れだけが選択的に
引き寄せられるのはどーして?
やはり>>718のように「粘着の条件」と粘性で考えるべき。
で、それは結局、分子間の相互作用で説明できるよ。

>>719は、27派の根拠は「経験則」じゃないよってことでしょ。
726バランス理論派:2001/08/23(木) 23:05 ID:WnmFWpLE
曲げ派から名前を変えます。

やり直しだそうなんでもう一度説明します。
1)揚力が得られる状態とは、流速の動圧と翼面の静圧が
バランスした状態である。よって
動圧の圧力=翼面の上下の静圧の圧力差。

2)翼面の圧力方向は当然面に垂直。
727ご冗談でしょう?名無しさん:2001/08/23(木) 23:39 ID:???
>>726
あらぁ、説明できなくなったんでコテハン変えて降参ってことだな。

>動圧の圧力=翼面の上下の静圧の圧力差。

これって、ベルヌーイの定理そのものじゃないのかい(藁藁
翼の上下の静圧の圧力差ってどーして生じるのかな?
728ご冗談でしょう?名無しさん:2001/08/23(木) 23:40 ID:q3HD4IQs
>>726
余計分からんぞ?
翼面の上下の静圧の圧力差=揚力
じゃないのか?
729ご冗談でしょう?名無しさん:2001/08/23(木) 23:50 ID:dtpPExfw
>周りの空気が「真空」に引き寄せられないで、流れだけが選択的に
>引き寄せられるのはどーして?
前面で上向き運動量を付けられていること。
翼が充分長く、周囲の空気が引き寄せられる間に、翼が移動して
新たな真空が作られる。(だから翼は左右に長い)
後は逆流防止の為に、下面の空気もやや運動量を付けて、翼の移動
方向の逆に流れていることも、重要か。
730728:2001/08/24(金) 00:05 ID:VRDRwTU2
>>727
>>動圧の圧力=翼面の上下の静圧の圧力差。

>これって、ベルヌーイの定理そのものじゃないのかい
ってそうなの?大体"動圧の圧力"ってのもわからんが。
731ご冗談でしょう?名無しさん:2001/08/24(金) 01:09 ID:???
>>729
コアンダ効果は翼じゃなくても起きるんだから、
そんなのは説明になってないよ。

>>730
ベルヌーイの定理は 動圧+静圧=一定 ってことだから、
翼の上面と下面で等式を作って移行しただけ。
旧曲げ派は今ごろ頭痛が痛いだろーな(藁
732ご冗談でしょう?名無しさん:2001/08/24(金) 01:23 ID:???
>>731
それだと
「動圧の圧力"差"=翼面の上下の静圧の圧力差。」
ってならないかな?
733732:2001/08/24(金) 01:27 ID:???
>>731
書き込んで気付いたよ
>頭痛が痛い
ワラタ
734ご冗談でしょう?名無しさん:2001/08/24(金) 01:59 ID:???
>>732
彼のアタマには「翼上面の動圧の反射(ただし符号は反対)」ってのが
あるから、「動圧の圧力差」って書きたかったんでしょ。
「動圧の圧力」じゃ変すぎるぜ。
まっ、どっちにしろ何を主張したいのかわからんけどね。
735バランス理論派 :2001/08/24(金) 02:11 ID:qp76I./E
動圧x間口面積=揚力って前書いたよ。
736ご冗談でしょう?名無しさん:2001/08/24(金) 06:47 ID:???
>>735
ええと、>>726では翼上下の静圧差と動圧がバランスして等しいと書いて
あるが、これだと 揚力=翼上下の静圧差×間口面積 ってことになって
明らかに間違いだぜ。鉛直方向の断面積をかけなきゃダメだろ。
どー説明するんだよ > バラ派 (藁
737ご冗談でしょう?名無しさん:2001/08/24(金) 09:11 ID:???
738ご冗談でしょう?名無しさん:2001/08/24(金) 12:46 ID:???
>>737
あのね、そのページの説明じゃ揚力を理解できないって話が続いてるんだぜ。

これは流体力学のジレンマかも。
翼の揚力はバッチリ計算できるのに、なぜ揚力が生じるのかって質問に対しては
マトモに答えることができない。で、どこがイケナイのかってゆーと、空気を
連続体で近似している点かな。計算する上では近似することに関しては問題無い
けど、「空気の分子の運動」という本質を覆い隠してしまっているので、翼に
作用する力をうまく説明できないんだな。まぁ、流体力学ができた頃には、
分子なんて概念は一般的でなかったら仕方ないけどね。

だから
 揚力を説明するときは「空気の分子の運動量変化」で
 揚力を計算するときは「ベルヌーイの定理」その他で
ってのが良いんじゃないの。
739ご冗談でしょう?名無しさん:2001/08/24(金) 15:39 ID:awLJoZqg
離散的な性質を元に無いと説明に説得力が無いというのでしょう。
しかし、どこに離散的な性質が使われている?
それを連続化しても、同様に説明出来るんじゃないの?
つまり、空気の微少粒子は、翼上面を通る時、何故か解らないが、
コアンダパワーが働き、翼上面に引き寄せられる。空気微少粒子同士は
粘性があるので、翼へ接している分子にコアンダパワーが働いて、表面に
引き寄せられた時、釣られて接していない粒子も、同じ方向に動こうとする。
つまり、コアンダパワーは、翼上面のかなり大量の空気を下に流す。
勿論コアンダパワーは、翼に対しては、空気とは逆の向きの力を与える。
これが揚力の本質。空気と物質の間に働く、新しい力、コアンダ。
状況によっては、翼上面後部では、コアンダパワーと慣性の為に、空気が
圧縮されて、高圧になることも考えられるね。それが下降気流のエナジー
になるのです。

つーのが、>>738の主張のようですな。「全ての現象の要因は、個々の微少要素に
対称的に分配されて、その集合で説明されなければならない」「部分は全体の
近似」でなければ、説明では無い。というのも、738の主張。

粘性で思い出したけど、それこそ翼と空気の摩擦力が重要なら、>>729の説明で
「サイドから流れ込む空気と翼の間に摩擦が働いて、流入を阻害する」ので、
低圧部喪失(揚力失効)が遅れる。というのを付け加えることが出来そう。
揚力が少し失われてくると機首が下がり、抗力が小さくなり、速度が増し...
再び機首を上げて、揚力を回復。こういうループを繰り返しながら飛んでいる。
常に、一定の速度・迎えでは、一定の揚力が得られる、という固定観念は捨てた
方が良いだろうな。時間平均で得られれば良い。ダランベールのパラドックスで
いずれ揚力なんて無くなると予想されている位だから。
740ご冗談でしょう?名無しさん:2001/08/24(金) 18:23 ID:???
??
単に気圧差で考えたらダメなんですか?





---------------------

極端な話、直角三角形を考えて高速に水平移動すれば対角線の面上が低気圧となるから押し上げられる。
勿論、他の成分も重なるので非効率だけどわかり易いと思います。
741ご冗談でしょう?名無しさん:2001/08/24(金) 18:42 ID:???
>>739
わかってんだか、わかってないんだかってカンジだけど、
最後のダランベールのパラドクスは確実に間違っているな。
「完全流体中を物体が移動しても抗力は働かない」
ってパラドクスなんだけど。知ってるのかい?

>>740
だから、どーして低気圧になるのかって話だ。
742ご冗談でしょう?名無しさん:2001/08/24(金) 19:10 ID:???
741
空気に一定の粘性あるからですよ。(瞬時に流入とかしないでしょう)
低速だと揚力でないから、力業で上昇しないといけないけどね。
743742:2001/08/24(金) 19:13 ID:???
粘性=慣性力=質量
744ご冗談でしょう?名無しさん:2001/08/24(金) 23:28 ID:5yPcqHFk
>>740,742
あんたのが一番判り易い。
745バランス理論派:2001/08/24(金) 23:28 ID:b85/XANk
>>373
ヴァカ八軒!
746ご冗談でしょう?名無しさん:2001/08/25(土) 00:12 ID:???
>>742
下降気流が発生する仕組みを説明してちょ。
翼の上の低気圧に向けて、下側から空気が流れ込んで上昇気流にならないのは何故?

あと「粘性=慣性力=質量」って、どの等号も無意味。
慣性力って座標変換で生じるみかけの力だけど、知ってるのかい?

>>745
やっと自分がヴァカであることに気付いた、とゆーザンゲだな。
747ご冗談でしょう?名無しさん:2001/08/25(土) 00:37 ID:???
> 下降気流が発生する仕組みを説明してちょ。
> 翼の上の低気圧に向けて、下側から空気が流れ込んで上昇気流にならないのは何故?


       D

  |
  |
A |    X    C
  |
  −−−−−−−−−−
      B


素人だから上手く伝えられないかもだけど...
低気圧になると当然四方から空気が流入してくるよね。
そこに何も無ければ上昇気流も発生するはず。
だけど実際は翌があるよね。
AからはXへと回り込もうとする。
DからはX方向へ
CからはX方向へ
Bからの気流移動は殆ど発生しない。(水平航行時)
で、翼が通過した後は全成分を考えると下降気流(xから見てBC方向)になるよね。

最も、下降気流は結果であって、揚力自体は気圧差によって翼がBからXへ引っ張られるって感じです。

> あと「粘性=慣性力=質量」って、どの等号も無意味。
> 慣性力って座標変換で生じるみかけの力だけど、知ってるのかい?

そうなんだ、知らなかった。
748ご冗談でしょう?名無しさん:2001/08/25(土) 02:15 ID:???
一応結論出たみたいね。

要するに移動することによる翼の幾何学的条件から空洞が出来るので、それを阻止する
べく流れる必要があり、結果として上面は低圧にならなければならない。当然、そこに
周囲の空気が流れ込まなければならず、通常は実際に流れ込み、低圧は起こらない。
(起こったとしても、有限時間でそれは回復されてしまう。)
しかし周囲の空気は、翼の摩擦(粘着条件)や、圧縮性や状況によっては粘性も作用し、
必要な分量が流入せず、圧力回復には至らず、低圧が維持されるのね。

でも、流体力学の理論では、粘性は速度場の状況を全体に伝える性質なので、もしか
すると、翼前方の空気が翼に当たる遥か以前から翼中心方向に集中するような流れ
になり、低圧が喪失してしまう可能性を否定はしていないね。つまり、摩擦力はともか
く粘性に関しては、「容疑者」ではあっても、「犯人」と断定出来るには至ってはいない。
(個人的には「クロ」と思うが)

ダランベールのパラドックスは、「一般に流れは障害物に力を及ぼさない流れになる
だろう」という遠大な予想。それにまで至る時間や、途中の状況については何も言っ
ていない。完全流体の定常解でそれを満たすようなものが存在することは
それを裏付ける話。それこそクッタージューコフスキーの流れのように、そうでないも
のも*数学的には*存在することからもそれがわかる。

揚力が失われる状態に近づくこと、つまり失速は標準派(ベル派)は否定していないみ
たいね。*自分たちの理論に都合の悪い結果*も否定しない。奥ゆかしいね。
とはいえ、それがどういう状況で揚力が発生するか、失速するかを、決定する方法論
はまだ無いみたいね。そこは経験論が支配する世界。

このスレで徹底的に叩かれている「粘性下降気流分子弾性運動理論派」は、揚力が
あることに関しては説明力もそして標準派並みの予言力もあるようだけど、他との繋
がりを考えるとかなり疑問みたいね。他との繋がりを考えなきゃ、Self-Containedな
話になって、自らの理論を自らの結論から証明するみたいな*寂しい話*になっちゃ
いそうだよね。寂しくない物理の専門家がここに来るわきゃないことからそれは裏付け
られてるね。

そういう話をする人の数学の取り扱い方は、自分の理論に都合の悪い結果から目を
背ける為に、好き勝手な条件をどんどん加えちゃって、最後は、自然の多様性に逃げる
場合が多いからね。ダマされないように注意しなきゃ。(普段は、自然を個性の無い
ヴァカな分子の集まりと見なしているくせしてね)

まぁ、唯一の取り得は、離散的と見なさせるモデルを取るなどして、確率に逃げる
ことが出来る点か。「分子の一様性は、空間分布の一様性を意味し、分子はそれ自身
確率を計算するオブジェクトである」か。だから、揚力を与える分布は普通の分布で
あり圧倒的に確率が大きいから...とね。この方面からのアプローチは新しい視点かも
知れないね。(でも無いか。)

物理理論の基本は、1に説明力、2に他分野への接続力、そして最後に予言力。
説明力・予言力を誇示せんばかりに、数学理論を捻じ曲げた形で振り回して、結
果として都合の悪い状況になって皮肉にも予言力も説明力も失ってしまうという愚だけ
は犯さないで欲しいと思うな。数学はむしろ物理の理論の説明力や予言力を否定する
為に発達した歴史であるという人すら居る位だからね。数学に負けちゃだめだyo!
749ご冗談でしょう?名無しさん:2001/08/25(土) 02:17 ID:???
>>747
この話も第1スレでガイシュツだな。

翼の後から斜め上に流れ込まないのは何故?
翼の端から空気が流れ込んで低気圧が無くならないのは何故?
翼端渦は知ってるかい?

揚力を「真空」で考えるのは止めようぜ。
750748:2001/08/25(土) 02:20 ID:???
それこそクッタージューコフスキーの流れのように、そうでないも
のも*数学的には*存在することからもそれがわかる

次に訂正。意味不明より

それこそクッタージューコフスキーの流れのように、揚力を発生させる
も のも*数学的には*存在することから,定常状態の話とダランベール
のパラドックスは関連はしている話ではあるが、同義では無いことが
わかる。
むしろジューコフスキーはパラドックスの否定の暗示といったところ。
751747:2001/08/25(土) 02:56 ID:???
> 翼の後から斜め上に流れ込まないのは何故?
> 翼の端から空気が流れ込んで低気圧が無くならないのは何故?
> 翼端渦は知ってるかい?

高速で飛ぶ時のお話ですね。(低速だと余り関係ない)
翼の両端を後方へ下げることに意識的に低気圧で発生した上昇気流を
翼の先端に回しているだけのことですね。

それでも、更に速度を出すと翼の両端までの距離は有限なので有効範囲を超えてしまう。
つまり、翼が生み出した低気圧の影響を受けない内に通過したことになる。

整理
低速の時、翼の幅を使って飛んでるのに対して
高速時は翼の長さを使って飛んでるだけのお話ね。

結局、747のメカニズムの応用でしかないと思います。
752ご冗談でしょう?名無しさん:2001/08/25(土) 02:58 ID:???
>>748
また、わけのわからないことを書いちゃって。

>しかし周囲の空気は、翼の摩擦(粘着条件)や、圧縮性や状況によっては粘性も作用し、
>必要な分量が流入せず、圧力回復には至らず、低圧が維持されるのね。

これ、良さげなカンジがするけど、よく考えてみると
 「空洞」ができるのは瞬間だけど、それが回復するのには時間がかかる
って、インチキが隠されてるんだな。「空洞」のでき方を考えていない。

たとえ時間がかかったとしても、「空洞」に空気が流入してきて低気圧は
無くなるよ。「空洞」の周りは「高気圧」の空気で囲まれてるんだぜ。
で、翼は新たな「空洞」を作らなければいけないけど、それには低気圧が
無くなるのと同じくらいの時間がかかるでしょ。瞬間的には生じない。
つまり「空洞」はできないってことだ。
だから、翼の上面の低気圧は、他の理由で説明しないとダメ。

あと、このスレで叩かれているのは「流体力学」だぜ。
分子間の相互作用とか統計力学とかを理解できないコドモが
変なことを書いているだけ。

ダランベールのパラドクスは、まだ間違ってるしな。(藁
753ご冗談でしょう?名無しさん:2001/08/25(土) 03:09 ID:???
>>751
>高速で飛ぶ時のお話ですね。(低速だと余り関係ない)
>翼の両端を後方へ下げることに意識的に低気圧で発生した上昇気流を
>翼の先端に回しているだけのことですね。

う〜ん、意味不明だな。

まぁ、この「空洞」の話は、
 どーして翼の上面の低気圧を維持することができるのか
を説明できないとダメでしょ。
「空洞」は容器で囲まれているいるわけではないんだぜ。
まわりには「高気圧」の空気があるんだよ。
754ご冗談でしょう?名無しさん:2001/08/25(土) 03:10 ID:WahGy1aw
>>752
748が正しいかどうかは別として、
>翼は新たな「空洞」を作らなければいけないけど、それには低気圧が
>無くなるのと同じくらいの時間がかかる
まったく同じ時間がかかるわけではないなら低圧部もできるんじゃない。
755ご冗談でしょう?名無しさん:2001/08/25(土) 03:24 ID:???
>>747
これって、ホントにXが低気圧になるの?

「風よけ」の中にいると、気圧が下がって息苦しくなる?
756ご冗談でしょう?名無しさん:2001/08/25(土) 03:32 ID:WahGy1aw
>>755
ああいう形だと少なくとも剥離が起こって低圧になるんじゃない?
757747:2001/08/25(土) 03:33 ID:???
うわー、勢いだけで書いたらガタガタだぁ。

ってことで、出直してきます。
758ご冗談でしょう?名無しさん:2001/08/25(土) 03:35 ID:???
>>754
「空洞」を作るためには空気を「排気」しなくちゃいけないけど、
それはどーやるんだろ。
結局、低圧部はベルヌーイの定理で説明するしかないんじゃないかな。
759ご冗談でしょう?名無しさん:2001/08/25(土) 03:42 ID:WahGy1aw
>>758
748の説だと翼が移動したことに対して空気がとっさについてこれずに
空洞ができるってことじゃない。
760ご冗談でしょう?名無しさん:2001/08/25(土) 03:45 ID:???
>>756
剥離が起こっていない翼周りの流れでも揚力があるので、
「空洞理論」はダメってことかな。

風よけの風下側はほぼ大気圧じゃない?
あと、キャビテイションって水で起こる現象でしょ。
761空洞理論派:2001/08/25(土) 03:47 ID:X/ADJC.6
なんだ、空洞理論ってそういうことか。
なら同じ意見だ。ハンドル変えよう。

>>753
静止状態じゃなくて移動している上で作られた定常状態だから
容器など無くても低圧は維持できるんだよ。

>>758
排気は翼下面がやってる。
762空洞理論派:2001/08/25(土) 03:48 ID:X/ADJC.6
>>760
(゚Д゚)ハァ!?
763ご冗談でしょう?名無しさん:2001/08/25(土) 03:51 ID:WahGy1aw
>>760
まあ翼周りで剥離が起こるっていったら失速時か翼の後縁ぐらいだしねえ。
ところで風除けの風下って大気圧なの?ダランベールのパラドックスじゃないけど
物体背後に回りきれずに低圧化してそうな気が。
764ご冗談でしょう?名無しさん:2001/08/25(土) 03:55 ID:???
>>759
水くらいの密度と粘性ならば「とっさ」について来れない場合もあるね。
スクリューとか。でも、空気ならほぼついて来れるでしょ。これで
飛行機の重さを支えるのに必要な「空洞」を作るのは大変だよ。
765ご冗談でしょう?名無しさん:2001/08/25(土) 03:58 ID:???
>>762
そんなカキコしてると、またいじめられちゃうよ。
766ご冗談でしょう?名無しさん:2001/08/25(土) 04:00 ID:WahGy1aw
>>764
俺もほぼついてこれると思うよ。
たしかこういった空気の圧縮性を考慮しなければならないのって
M=0.3付近からじゃなかったっけ。
767ご冗談でしょう?名無しさん:2001/08/25(土) 04:10 ID:???
>水くらいの密度と粘性ならば「とっさ」について来れない場合もあるね。
>スクリューとか。でも、空気ならほぼついて来れるでしょ
水のほうが空気よりもレイノズル数が高いってこと知らない人の発想
むしろその逆。
粘性と密度で説明するなら、空気は密度の割に粘性は高いからだと一掃
ついてこないと言える。但し空気は膨張収縮する性質があるから、空洞=真空
とはならないだろうがね。
768ご冗談でしょう?名無しさん:2001/08/25(土) 04:11 ID:???
>>766
う〜ん、マッハ数まで覚えてないなぁ。
まぁ、「空洞」派は定量的な根拠を示さないと説得力はないね。
769ご冗談でしょう?名無しさん:2001/08/25(土) 04:28 ID:WahGy1aw
ちなみに手元にたまたまあったNACA0012 at α=9(deg)の圧力分布図みると
最も圧力が低くなるのは最大翼厚部よりもだいぶ手前だよ。翼の前縁から
数%のとこでピークを迎えそこから急激に回復してる。
この急激な回復が剥離現象を招いちゃうことになる原因だったよね。
770ご冗談でしょう?名無しさん:2001/08/25(土) 09:17 ID:???
>>767
また、わけのわからないことを書いてるなぁ。

あのねぇ、空気の動粘性率(=粘性率/密度)は水の10倍以上でしょ。
だから空気のほうが移動している翼(板か?)についてきやすい。
粘性がゼロであれば、板についてこないからすぐに「空洞」ができるよ。
で、それが回復するのは分子の拡散だから有限の時間がかかるね。
まぁ、粘性を考えたら「空洞派」は止めたほうがよさそーだぜ。
771ご冗談でしょう?名無しさん:2001/08/25(土) 09:20 ID:???
>>769
>最も圧力が低くなるのは最大翼厚部よりもだいぶ手前だよ。

これは迎角が9度だからかな。失速は10数度だから結構大きいよね。
772ご冗談でしょう?名無しさん:2001/08/25(土) 10:39 ID:nok.n3q.
>>770
空洞は出来ないけど、圧力が低下する理由はありそうということね。
773空洞理論派:2001/08/25(土) 10:58 ID:M7ivN4T2
>>770
>拡散だから有限の時間がかかるね。

よく判らん表現だが、煙みたいな速度を想像してるんだろうか?

あと空気がついてこない方が揚力上がって空洞派に有利なんですが
混乱してませんか?
774ご冗談でしょう?名無しさん:2001/08/25(土) 11:01 ID:???
>>772
実際の(計算の?)流れを見るかぎり、「空洞」による低圧部は無いってのが、
>>769の圧力分布。仰角が大きくなると「空洞」ができやすくなるハズでしょ。
だから、無意味な「空洞」を考えるのは止めようぜ。
775ご冗談でしょう?名無しさん:2001/08/25(土) 11:10 ID:???
>>773
空気には粘性があるんだから、「空洞」はできないって話だ。
実際の翼断面でも「空洞」は生じないんだから、空洞派も廃業
した方が良いんじゃないの。

あと、分子の運動と煙の拡散は違う。空気分子の平均速度って
知ってるかい?
776空洞理論派:2001/08/25(土) 12:17 ID:M7ivN4T2
>>775
空洞が生じようとするのを埋め合わせるために空気が集まって来て
結果、空洞は生じず、翼上面に沿った流れができるということ。
777空洞理論派:2001/08/25(土) 12:23 ID:M7ivN4T2
粘性がそんなに強い力をもたらすかねえ?
飛行機を持ち上げる程の力を分子間引力が持ってるなら
空気中で回転する物体は大きな抵抗ですぐ止まりそうだけど。
778空洞理論派:2001/08/25(土) 12:24 ID:M7ivN4T2
独楽とかね。
779769:2001/08/25(土) 16:40 ID:9cikoyQI
>>771
迎角の影響もあるだろうけど、とても翼の背後とはいえない
場所だよ。NACA0012を傾けてみればすぐに分かると思う。
翼型データならWeb上にもあるから探してみたら。
いずれにせよ空洞だけで説明するのは無理があると思う。
780ご冗談でしょう?名無しさん:2001/08/25(土) 18:31 ID:nok.n3q.
いつの間にか空洞が出来るという話になっているが、空洞派理論
のエッセンスは、翼背面で、進行方向のベクトルと平行で無い場所
や、翼表面の空気の流れで、翼と垂直な成分が0で無いことも
あり得てそこでは表面空気が膨張して圧力が低くなる可能性もある
という点。
前者は、空気をそこに流入させる可能性があり、単位時間に流入
する量は、仮想的な空洞の体積に等しい。
要するに、翼という障害物は、空気をその中に侵入させないように
はするが、別に引っ張ったりするわけでは無い。結果、空気自身
の慣性によって、表面形状によっては、翼から離れる場合も起こり
そこでは膨張し圧力減少を招くということでしょう。最大翼厚後部
に起こる必然性は必ずしも無いと思うが。(それは翼形状の個性の
問題でしょ)
781ご冗談でしょう?名無しさん:2001/08/25(土) 19:03 ID:???
>>776
「空洞」を作らない空洞派か (藁
翼の上面での気流もちゃんと説明できないし、
コアンダ効果も「空洞」では説明できない。
なんでそんなに「空洞」にこだわる?

>>777
回転面の接線方向に強い力が働けばすぐに止まるだろーね。
揚力は流れとほぼ直角の方向だぜ。回転面の法線方向に等方的に
力が作用するとどーなるのかな (藁

自分の理解できないところに真実があるかもしれない
って謙虚な態度が欲しいぞ > 空洞理論派
782ご冗談でしょう?名無しさん:2001/08/25(土) 19:10 ID:???
>>780
「進行方向のベクトルと平行で無い場所」とか意味不明なことを
書かれても、誰もわからないんじゃないのかな。

最大翼厚部より前側だと空気は圧縮されそうだが、>>769の圧力分布
では低圧になっている。これに対する言い訳はあるの?
783動圧反射+空洞理論派 :2001/08/25(土) 19:17 ID:M7ivN4T2
翼上面をどう説明すると聞かれるから空洞と答えたが
翼下面だけでも説明できるのを忘れてたのでまたしても名称変更だ。
変なツッコミ来るしな。

>翼の上面での気流もちゃんと説明できないし、
>コアンダ効果も「空洞」では説明できない。

なんでそんなもん説明しなきゃならんのだよ?

>回転面の接線方向に強い力が働けばすぐに止まるだろーね。
>揚力は流れとほぼ直角の方向だぜ。回転面の法線方向に等方的に
>力が作用するとどーなるのかな (藁

お前ベクトル解析やり直しだ。
784ご冗談でしょう?名無しさん:2001/08/25(土) 19:18 ID:nok.n3q.
>>782
実際多少は圧縮されるでしょ。速度が大きくなればそれは顕著
になる筈。全ての部分で圧縮されると言う意味では無いが。
速度によって圧力分布は変わる筈。温度、高度、速度、迎角等の条件によって変化する
ので、単純に圧縮されないと決めてかかってはいけない。
圧縮される為には、入ってくる空気よりも、出て行く空気が少ない
場合に起こる。
785ご冗談でしょう?名無しさん:2001/08/25(土) 20:47 ID:???
>>783
プププッ。「ボクにはわかりません」ってことかな。
もうツッコミしないから、あとは一人で頑張ってね。
786 :2001/08/25(土) 20:51 ID:M7ivN4T2
>>785
サイナラー
787ご冗談でしょう?名無しさん:2001/08/25(土) 20:52 ID:???
>>784
なんか説明が逆かな。
単純に考えると最大翼厚より前側では高圧になると思われるが、
ある条件では低圧になっている。これは何故でしょう?
ってことなんだけど。
あと、定常状態だと「入ってくる空気よりも、出て行く空気が少ない 」
とゆーことはないよ。
788ベル派:2001/08/25(土) 23:53 ID:B2PcIuYY
翼の表面の圧力は、圧力計を翼に組み込むことで、実測可能です。
実用的な翼の圧力分布では、翼の上面のほとんどの部分で無限遠に比べ低圧になっています。
また、上面と下面における流速も実験的に測定することが可能です。
ブリタニカ百科事典の航空力学の項目の記述によれば、前縁で上面と下面に分かれた流れは
後縁でほぼ同時に合流するそうです。
揚力の原因は、上面と下面の圧力差で説明できますが、以上の実測結果はベルヌイの法則とも
整合しています。
789ご冗談でしょう?名無しさん:2001/08/26(日) 00:09 ID:/i7lueBw
790789:2001/08/26(日) 00:15 ID:???
>>788
ブリタニカ間違ってんじゃないの?
上面側のほうが早く到着するはずだよ。これは実験データでも理論でも
そうだったと思うけど。
>前縁で上面と下面に分かれた流れは
>後縁でほぼ同時に合流する
ってのは同時到着理論っていうありがちな間違いとして教科書なんかにも
のってるよ。
791ご冗談でしょう?名無しさん:2001/08/26(日) 03:29 ID:???
>>788
「ベル派は間違っている」ってことじゃないよ。正しく計算できる。
「どーして翼の上面の流速が速いの?」ってゆー素朴な疑問に簡単に
答えられないのが問題になってる。

ブリタニカの航空力学の文責は誰?

>>789
この圧力分布と流線ををみれば、「空洞派」とか「低圧部に引かれて流れが曲がる」
とかの主張が、実際の現象を正しく表していないってのがよくわかるな。

>>790
「同時到着理論」を専門家が正しいと信じていた時代があったのかな。
792ご冗談でしょう?名無しさん:2001/08/26(日) 05:07 ID:???
良く解ってないんだけど、翼上面の空気のカタマリが翼によって
一斉に引っ張り下げられるので、その際に翼と空気との間に力が
働いてそれが揚力ってわけでしょ。非ベル派の立場からすれば。
空気って推す場合ならともかく、引っ張る時にそんなに一斉に
引っ張られるものなのかな。周辺の色々な方向から空気を取り込んで
しまうので、翼が重力と釣り合う位の力積を空気から貰おうとするなら
相当に巨大な体積の空気が、翼によって一斉に下向きにならなくては
ならない気がする。特に低速の時。前スレの途中で、ジャンボ機なら
時速900kmで翼の上10m位が一斉に下ろされると概算されて
いたようだが、300km位だと速度が減る分、翼上通過空気の量が
減るから、高さにして9倍も必要になる。100m近くの空気が
吹き降ろされるものかな?粘性によって。
793ご冗談でしょう?名無しさん:01/08/26 09:53 ID:???
>>792
翼がどのくらいの範囲の空気に影響を及ぼすかは、>>789とかの
リンク先にある流線の図を見ればわかるよ。少なくとも翼厚の
2〜3倍ってことはないよ。

ジャンボの時速が300キロの時は上昇中で、機首が上がっているでしょ。
この時はエンジンの推力の下向き成分も大きくなってるよ。どーして
「下降気流だけで機体の重さを支える」って考えちゃうのかな?

この話、前スレでガイシュツ。

巡航しているときにもエンジンは少し下を向くように設計されて
いたりするのかな? 教えて > 航空機マニア
794ご冗談でしょう?名無しさん:01/08/26 11:57 ID:hsOCbDos
久しぶりに来たけど、なんか、またガイシュツの話しに
なって堂々めぐりしてるね。どこかで収拾つけないと...。

>>782
そんなに不思議かな? 迎角があるから(定性的に言うと)空気は
翼下面の前縁に近いところに当たる。その時の翼上面における流れ
を考えれば、いくら前縁側とはいえ翼上面は負圧になるんじゃない?

>>792
>翼上面の空気のカタマリが翼によって 一斉に引っ張り下げら
>れるので、その際に翼と空気との間に力が働いてそれが揚力
>ってわけでしょ。
非常に大雑把言えば、空気に粘性があるから(せん断応力が働い
て?)翼上面に沿って流れてるだけだと思うけど。翼前縁で上向
き流れになること+翼後縁部分の傾斜+迎角を考えれば、翼上面
で空気は最初の方向から曲げられて最終的には下向きに流れてい
くでしょ。ちなみに空気に粘性が生ずる原因が分子間力によると
すると、乱暴な言い方をすれば分子間力により下向き流れが作り
出されるという事になるのかもしれない。

>>793
話しが難しくなるから、あくまで定常飛行での事を考えようよ。
エンジンは揚力とは全く関係無いぞ。紙飛行機やグライダーだっ
て自重と釣り合うだけの揚力を発生させている。「下降気流だけ
で機体の重さを支える」のは(力と運動量が混同している気も
するが)概念的にはそれで正しいんじゃない。
 もちろん、この下降気流の反作用として生じる揚力は、翼上下
面の圧力差(正確には翼面にかかる圧力の総和)と等しいけどね。
795ご冗談でしょう?名無しさん:01/08/26 16:47 ID:???
>>794
話を速く収束させることには賛成だが...

>そんなに不思議かな?

翼の上面で流れが直接当たるところの圧力が下がってるので不思議。
http://www.hanleyinnovations.com/nacaresults.html
これの迎角5度の図を見てみそ。空洞理論は信じられないよ。

>非常に大雑把言えば、空気に粘性があるから(せん断応力が働い
>て?)翼上面に沿って流れてるだけだと思うけど。

いくら粘性があっても、翼から100メートル離れた空気を積極的に引張るとは
考え難いよ。

>話しが難しくなるから、あくまで定常飛行での事を考えようよ。
>エンジンは揚力とは全く関係無いぞ。

そりゃ、巡航状態だけを考えれば何も問題ないよ。>>792の疑問は、
高度3000フィートを時速600マイルで巡航するように設計された飛行機が、
時速200マイルで飛んだときは、単純に考えると9倍の量を空気を下降気流に
しなくちゃいけないけど、そんなことは可能なのか?
ってことでしょ。で、その答えは「不可能」
だから、飛行機の重さを支えるためにエンジンの推力を使うんでしょ。
迎角を大きくしてかつフラップを下げた状態でも、速度の低下による揚力の
不足は補えないと思うぞ。

紙飛行機だって非常に遅い速度では飛ばないでしょ。
796ご冗談でしょう?名無しさん:01/08/26 17:54 ID:???
上面で流れが加速する理由としてこういうのはどう?
本来なら(翼後縁で剥離が起こらなければ)翼下面から上面側に
回り込む流れがあるよね。これが剥離によって阻まれる事によって
翼上面後縁付近に出発時に一時的に低圧部ができて、これによって
上面で流れが加速すると。どうだろ?
797ご冗談でしょう?名無しさん:01/08/26 18:01 ID:???
>>795
低即時は翼の幅を使い、高速時は翼の長さを使うって良く聞きますよ。
798ご冗談でしょう?名無しさん:01/08/26 18:45 ID:wStle8ag
>>796
ゴメン、意味不明だよ。

>本来なら(翼後縁で剥離が起こらなければ)翼下面から上面側に
>回り込む流れがあるよね。

この時点で、すでに翼上面が低圧になっていることを仮定しているけど...

>>797
どーゆーことかもう少し詳しく書いてみて。
迎角を大きくするために飛行機の機首を上げると、エンジンの推力の
下向き成分が増えるんだから、スナオに考えれば良いんじゃないかな。
799796:01/08/26 23:05 ID:Yggp.JV2
>>798
説明が悪かったかな。たとえば非粘性で正の迎角を持った対称翼(平板でもいいや)周りの
流れを考えると後方よどみ点は翼の上面側にあるよね。低圧を仮定しなくても。
つまり翼下面から上面側に回り込む流れがある。
800ご冗談でしょう?名無しさん:01/08/26 23:59 ID:2aVCr7is
あのー
翼実験で,Cp-x/C,Cp-y/Cのグラフからどうやって面積を求めていますか?
801ご冗談でしょう?名無しさん:01/08/27 00:02 ID:2NFCZnYc
794だが、(ちなみに前スレ27派、もちろんベル否定でない)

>>795
圧力分布について、俺は空洞理論派でないし、そもそも空洞理論
が何なのか未だに判らない。空気の流れの方向と翼表面の接平面
を考えれば、翼上面前縁部付近は負圧で不思議じゃないと思うが。
まあ、個人的な受け取り方の違いかもしれないけど。

下向きに運動量を与えられる空気の量や速度については、前スレ
412で27氏が見積もっていたが、あの見積もりが正しいのか俺に
は判らない。前スレ908で逆ギレした某氏が訳の判らない事を書い
ているが、27氏の見積もりに対する反論部分はまともだと思う。

つまり、翼から100メートル離れた空気が下向きに与えられる運動
量や、速度により空気がされる仕事がどう変わるかは、もう一度
議論しなおす必要があると思う。数字が一人歩きしてるよ。

その後ろの推力うんぬん、紙飛行機うんぬんの話しはちょっと
難しくて、本論と直接関係ない方向に話しが行くからパス。

空気が受ける下向き運動量の見積もりがきちんとできると良い
んだけど難しいよね。境界層の問題や3次元の問題があるから。
この見積もりについては、俺はできないや。誰かできる?
802 :01/08/27 01:12 ID:qeolEtH2
>>795
その圧力分布図、間違ってる。
ヴァカですか?
803ご冗談でしょう?名無しさん:01/08/27 01:29 ID:NCQDrxqI
>>799
クッタの条件で流速をゼロにするのは平板の後端だが...
よどみ点が翼の上面にあるとしても、>>796の「これが剥離によって阻まれる…」
以後はよくわからないよ。

>>801
空洞理論派って、翼で空気を押しやるから後側に「空洞」ができるって
ことじゃないのかな。オレもよくわからん (藁

>27氏の見積もりに対する反論部分はまともだと思う。

これは

>翼の上数cmでは、時速5000km/hを超える猛烈な下降気流、しかし
>1m地点では、僅か時速0.1mmの超微風、しかし下降流のトータルな運動量は
>機体を支えるのに十分という結論

このこと?
飛行機は静止している大気中を飛んでるんだから、こんな結論を導き出せる
わけないでしょ。常識的に考えれば前スレ412の見積もりで良いんじゃないかな。
上空での空気の密度がチョト違うけどな (藁

>その後ろの推力うんぬん、紙飛行機うんぬんの話しはちょっと
>難しくて、本論と直接関係ない方向に話しが行くからパス。

速度が遅くて十分な揚力が得られない時は、飛行機の重さを支えるために
別の力が必要ってことだよ。
804796:01/08/27 01:53 ID:X8mz3TB6
>>803
非粘性流れでクッタ条件を考慮せず(要するに循環0で)流れを考えると
よどみ点は翼の上面にありますよね。
でもって実際の流れでは粘性があることからそのような流れは実現せず
翼の後縁で剥離が起こって翼の上面に回り込めなくなってるわけです。
この結果一時的に低圧部が生じてその結果上面で流れが加速すると。
805ご冗談でしょう?名無しさん:01/08/27 02:07 ID:NCQDrxqI
>>804
「粘性があるから後縁で剥離が起こり上面に回り込めない」って
ことかな。う〜ん、やっぱりよくわからないよ。なんで?
806796:01/08/27 02:39 ID:X8mz3TB6
>>805
後縁で剥離する理由が分からないって事ですか?
単に後縁が鋭いからじゃ不満ですか?
807ご冗談でしょう?名無しさん:01/08/27 02:50 ID:NCQDrxqI
>>806
そしたら、最初からよどみ点は翼上面に無くて後縁にあるわけだから、
「回り込むのを阻む」こともないよね。途中で非粘性→粘性と変わる
んじゃないでしょ。
808796:01/08/27 03:31 ID:X8mz3TB6
>>807
最初からよどみ点が後縁にあるかはおいといて…
たとえば流れにたいして垂直に板を置いたらその背後で圧力が低下
してるでしょ。そうしなけりゃダランベールのパラドックスだ。
これも板の端で剥離が起こって回り込みを阻んでるからじゃないの。
809ご冗談でしょう?名無しさん:01/08/27 10:34 ID:wj/f3Dm.
ジャンボ機のスラスト(推力)なんて、フルパワーでも0.5G(Gは重力加速度)程度しか
無い筈。45度というフラップ全開でも失速限界を遥かに超えた角度を付けていても、スラスト
で得られる垂直方向の加速度なんて0.25Gを超えることなんて無いことが解る。
(正確にはこの場合、0.5G/√2=約0.3G)
有り得ないが、45度の角度で上昇していても!!必要な揚力の70%は空気を引っ張ることに
よって揚力を得なくてはならないことになる。

つまり、300km程度の速度なら、それでも50m上まで、ほぼ一定の速度の下降気流が
存在しなければならないことになるね。
勿論、「一定の」という条件を外せば、翼近くでは猛烈に速く、それより上ではそれ程でも無く
という分布であれば、一応辻褄は通りそう。(翼のずっと後ろの方では、減衰はあれども、粘性
によって、下降気流の速度分布は平均化されるでしょうし、このことをFull developed down wa
shと言っていると思ワレ)

この下降気流は、翼が空気との界面で、空気を下に引っ張ることによって産み出され、
粘性や圧力勾配によって周辺に分配される。(粘性と圧力勾配は共に下降気流に寄与するが
粘性のほうが、効率的に下降させる。しかしどちらが大きいかは、飛行機の速度にも関係
する話になるだろうが、圧力の伝達速度がかなり遅いのに対し、粘性は非常に早く伝わる
という点は注意すべき。)下への引っ張り方は、*空気が翼に常に接するように*必要に
して十分な力で引っ張り、粘性が高ければ、上の空気が引っ張りに抵抗するので、それなりに
大きくなる必要がある。つまり、揚力は、*翼が空気を引っ張るから*発生する。
上面の空気の圧力勾配がある以上、翼上表面の圧力は一般に低下し、上下で圧力差が見出され
る。....というのが、非ベル派の主張。

ベル派(=標準派?=空洞派??)の主張である、上面で空気が*勝手に*低圧になって、翼の
上下に与える圧力差が揚力の原因であるという考え方と、大きく異なっている。
空気と翼が表面で接している場合、それを引き離そうとする場合に働く力で(実際引き離せないが)
それに対して必要にして十分な力が働く、というのが大きな相違点。表面以外の空気が、表面に
接している空気と「癒着」しているので、表面の空気だけを引き下ろすというわけにはいかず、
一緒にその上のかなり大量の空気を引き下ろさなければならなくなり、それが原因で大きな力とな
るという。

可能性は否定は出来ないでしょう。ベル派の弱点である「なぜ勝手に低圧になるのか?」という
問いに対する明確な解答が無いことから、有力な説とさえ言えるかもしれない。
しかし、ここに来て空洞派という面白い意見が出てきた。

空気が完全流体(密度が変わることが出来ない。粘性を持たない)だったら、翼から離れて流れる
ことが禁止されるわけじゃないので、慣性により、翼の表面に接して流れることが出来ない場合
も有り得る。翼の形状で、その断面を為す曲線が急変化する場所では、そういうことが起こる可能
性は否定出来ない。翼にかかる応力は流体の圧力であるとして、その場合、真空が発生している
わけだから、非常に強い力と成り得るという主張。
実際の空気は、圧縮性も粘性も持つが、完全流体なら翼から離れる場合でも、その点では膨張して
低圧になった空気が翼に接して流れることになる。(ベル派で言う低圧部の存在理由)
「剥離はしていないが、本来なら剥離する筈であるので、膨張して低圧になる」というのが、
空洞派?の主張なのでしょう。

最初に発生した低圧部の圧力が小さければ、その上の空気が下降し、以降の流れに影響を与える。
だから、翼背面に発生する筈の「真空」が予想以上に回復されて、最小圧力部分が、前の方に
出来るという理由もあるでしょうから、圧力分布による「疑惑」も、この主張を否定するとは
言えないと思います。

論点は、空気と翼の界面には、それを引き離す場合に力(非ポテンシャル力になるでしょう)が
必要かどうかになりました。(この力は空気と翼の相対速度に大きく関係することになります。)
810ご冗談でしょう?名無しさん:01/08/27 12:16 ID:jhyje.1c
>>803
801(=794)だけど、「前スレ27氏の412見積もりに対する反論部分」
で前スレ908を出したのはその部分じゃない。あまり良い引用でなく
申し訳なかった。とにかく、前スレ412見積もりがしっくりこないん
だ。パラメータをいじれば説明できるような説明できないような感じ
だから。例えば、下降気流の速度が対気速度の1/10ってのはどこから
出てきたんだっけ?
(いや、自分で代案モデルも出さないくせに申し訳無い、既存の研究
で下向きの流れの定量モデル or 実測値があるような気もするが...)

>速度が遅くて十分な揚力が得られない時は、飛行機の重さを
>支えるために別の力が必要ってことだよ。

これは違うんじゃないかい?
速度が遅くて十分な揚力が得られない=気流剥離=失速だよ。
失速しない範囲で考えれば、速度が遅かろうが速かろうが、
重力に見合うだけの揚力は発生している。(詳しくいうと、
速度が変わるときは揚力ベクトルの方向が変わるので、重力
と釣り合うのは揚力の分力となるが、ややこしいし、微小
な違いなので無視)

あと、とにかくエンジンは無視して構わない。エンジン無しの
グライダーでも紙飛行機でも低速時高速時に同じ事が起きる
からね。揚力は一定としてこういう事だと思う。
低速飛行=迎角大=揚力係数大 → 大量の空気に(低速)下向き運動量
高速飛行=迎角小=揚力係数小 → 少量の空気に(高速)下向き運動量


P.S.
そろそろ「前スレ27」のコテハンに戻したらどう?
今や大勢はあなたの意見を認めているから、前スレみたいに
叩かれないよ。最近は別の叩かれコテハンキャラも居るし。
まあ、あくまでも非常識な態度を取らなければの話しだけどね。
811796:01/08/27 17:06 ID:s2KlB5zA
>>807
それと確かに途中で非粘性→粘性と変わるわけじゃないけど
粘性があって後縁が鋭ければ即剥離が起こるってわけじゃないよね。
剥離が起こるためにはある程度の流速が必要なのは剥離の
メカニズムから考えても妥当じゃないかな。
身近なとこだと皿なんかに水を入れてその中でスプーンを
動かしてみるとこの辺は納得いかないかな?
ゆっくり動かせば流れはスムーズだよね。
812ご冗談でしょう?名無しさん:01/08/27 17:16 ID:BvMklDhY



各大学の実力派学生ポータルサイトたん*萌えなびナビ*

早稲田(わせ)―http://waseda-links.com
ICU(あしゅ)―http://icunavi.com
立教(りきょ)―http://ri-style.com
東邦(とほ)―http://www.h2.dion.ne.jp/~jn1
東洋(とよ)―http://toyowave.com
中央(ちゅお)―http://chuo.pos.to



↑( ・∀・)イイ!
813ご冗談でしょう?名無しさん:01/08/27 17:59 ID:J/EQ..jc
>>810
>例えば、下降気流の速度が対気速度の1/10ってのはどこから
>出てきたんだっけ?

また前スレが見れなくなってしまったんだが、確か、気流の下降成分を時速900キロの
1/10の時速90キロ=25m/sで計算していたハズ。で、「1/10」の根拠は翼上面の後縁と
水平のなす角度のtan(sinかな)。5.7度くらいだからリーズナブルでしょう。
前スレに出てなかったっけ?
下降成分が25m/sのときは押し下げる空気の量は、
 地表付近の空気の密度だと翼の上下6メートル
 巡航高度の上空で密度が1/3くらいだと18メートル
だったかな。
で、時速300キロ=83.3m/sの低速時は迎角が大きいから、翼上面の気流が水平となす
角度を15度とすると、下降成分は20m/sくらいだな。速度が25m/sの80%だから
揚力を得るのに必要な空気の量は1/(0.8)^2=1.56倍。地表だと6×1.56=9.36m。
つまり翼の上下10メートルくらいで良いのね。迎角が大きいとイケるじゃん。

>これは違うんじゃないかい?
>速度が遅くて十分な揚力が得られない=気流剥離=失速だよ。

ジャンボを時速100キロくらいで飛ばすことを考えていたけど、離陸速度が
時速300キロだから、確かに無意味でした。

エスティメーションで疑問なところは他にもある?
前スレ27に代わってお答えいたしましょう。(藁
814ご冗談でしょう?名無しさん:01/08/27 18:04 ID:J/EQ..jc
>>811,>>808
要するに「空洞理論」ってことかな。
815ご冗談でしょう?名無しさん:01/08/27 20:49 ID:jhyje.1c
>>813
810(=801=794)だけど、数字の絶対値はともかく、オーダー的
な値と、定性的傾向は納得。どうもありがとう。
816796:01/08/27 22:00 ID:/821j3tI
>>814
違う違う。
空洞理論は「揚力=空洞による低圧」て言う主張じゃない?
自分が言ってるのは(まるっきり自説というわけではないけど)

出発時に翼上面の後縁付近に「一時的」な低圧部発生
        ↓
これによって上面の流れが加速(下面側からは剥離によって回り込めず)
        ↓
したがって翼の上面と下面で流速差が生じる
        ↓
翼の上面と下面で圧力差が生じる
        ↓
      揚力発生

ってな仮説。ちなみに
「出発時に翼上面の後縁付近に「一時的」な低圧部発生」っていう部分は
実際に現象としては起こってるみたいだよ。

あと、自分は一応空洞理論に関しては否定派なんで。
817SさんからMさんへ:01/08/28 01:14 ID:6wE8Pj.I
前スレが見れない?本当?見れたけど。
関連部分の引用サービス
参考に。
前スレ27=412さん、勝手に引用してスマソ

................以下前スレ412の引用...........................

412 名前: 27 投稿日: 2001/05/17(木) 15:46

巡航(定常)状態を考えてエスティメートしてみる。
飛行機の質量をM、速度をU,翼によって下降気流にされる空気の
質量をm、下向きの速度をvとすると、t秒間での空気の運動量の
変化は下向きの初速はゼロだから
 mv=Ft
Fは翼が空気に及ぼす力。で、これが機体の重力Mgと釣り合うので
 mv=Mgt
翼は1秒間に
 m=Mg/v
の質量の空気を下降流にしている。

ジャンボジェットの質量は300トン、翼幅は60m、速度を毎時900Km
とする。下降気流の速度は機体速度の1/10程度なので
 M=300000Kg, U=250m/s, v=25m/s
これから
 m=300000*9.8/25=117600
となり、毎秒120トンの空気を下降気流にしていることになる。
翼の間口は60mで1秒間に250m前方の空気が翼に流入するので、
 H=117600/(60*250)=7.84
翼の上(下)8メートルの空気が下降気流になればよい。
ただし上空での空気の密度は1Kg/m^3とした。

これで納得するだろう。
818ご冗談でしょう?名無しさん:01/08/28 09:25 ID:lxaLp2uY
>>816
ええと、翼上面後縁付近の低圧部に前方の空気だけが選択的に引かれるのは何故?
翼上面の後方とか上方の空気は動かないの?
あと、「一時的」に低圧部が発生するなら、そこに空気が流入すれば低圧部は無くなるよ。
819ご冗談でしょう?名無しさん:01/08/28 09:43 ID:lxaLp2uY
>>817
>>1から前スレに行くと「まだhtml化されてません」って出るけど...

> H=117600/(60*250)=7.84
>翼の上(下)8メートルの空気が下降気流になればよい。
>ただし上空での空気の密度は1Kg/m^3とした。

地表での空気の密度は 1.293kg/m^3 だから 7.84/1.293=6.06m。>>813は合っていたな。

迎角を大きくするのは気流の速度の下向き成分を増やすのメインだな。
2乗で効くし。
820796:01/08/28 12:49 ID:6b8WUDqg
>>818
べつに前方の空気だけが引かれるわけではないんじゃない。
前方の空気”も”引かれて加速する。まずいかな?
でもって上面側で空気が加速して流入することで(局所的な)低圧部は
なくなっちゃうと。もちろん加速した結果として下面側よりは
低圧になっている。
で、仮に上面側の速度が低下してこの状態からずれたとすると、
再び翼の後縁で回りこみが発生=>剥離=>局所低圧部発生
ってな感じで復元される。

実際にはこういったメカニズムでクッタ条件を満たすように
循環ができてるのではないでしょうか。
821ご冗談でしょう?名無しさん:01/08/28 15:00 ID:ypcozjrI
>時速300キロ=83.3m/sの低速時は迎角が大きいから、翼上面の気流が水平となす
>角度を15度とすると、下降成分は20m/sくらいだな。速度が25m/sの80%だから
>揚力を得るのに必要な空気の量は1/(0.8)^2=1.56倍。地表だと6×1.56=9.36m。

論理的にメチャクチャじゃないの?水平速度は時速900kmの3分の1のままな
んだから高さにして900kmの時の3X(1/0.8)=3.75倍、必要。(本来は3X3倍だけど
迎えを増やした分、3X(1/0.8)倍で良くなった)
6.06mの3.75倍は22.7m
地表付近で300kmで飛ぶ時は、迎えを大きく取って、下降気流成分を増やした
としても、上下23mの範囲を一様に20m/secの風を起こしたことにしなければ
計算は合わない。
迎角が大きいとちょっと非現実的な値になるね、(計算間違いは許してあげよう)

未だに2次元完全流体(無限領域・地面無し)理論で、揚力を総じて理解
しようなんて、無謀なこと止められない人が居るね。
参考にするのは良いけど、それに拘ると大局を見失う気が...
運動量と絡むのだけは止めたほうが良い。無限領域だから無意味だが。
地面効果の影響は、流体を扱う時は必ず設定しなければ、理論として無意味
だろうに。
822ご冗談でしょう?名無しさん:01/08/28 15:21 ID:ypcozjrI
次の非ベル派の「反論」は、「地面効果の影響だ」に決まっているが、
3000m程度まで上昇しても、旋回の半径を小さくする理由でその程度の
速度で飛行する場合があるから、地面効果は理屈にはならないよ。
釘を刺して置くが。
823ご冗談でしょう?名無しさん:01/08/28 16:43 ID:QOV26knw
>>820
>で、仮に上面側の速度が低下してこの状態からずれたとすると、
>再び翼の後縁で回りこみが発生=>剥離=>局所低圧部発生

上面側の速度が低下すると、下から回り込んでくるのは何故?
粘性流体ならば最初から回り込めないんでしょ。
824ご冗談でしょう?名無しさん:01/08/28 16:50 ID:QOV26knw
>>821
>論理的にメチャクチャじゃないの?水平速度は時速900kmの3分の1のままな
>んだから高さにして900kmの時の3X(1/0.8)=3.75倍、必要。(本来は3X3倍だけど
>迎えを増やした分、3X(1/0.8)倍で良くなった)

これはその通り。計算を間違ってました。前スレ412は理解してるのね。
で、23mくらいならオッケーなんじゃないの。上空だと18mだし。
ちょっと大きいとゆー「気がする」だけで、不可能である根拠は無いでしょ。
翼周りの流線の図は見たことあると思うけど、翼からどのくらい離れたところ
まで流線は曲がってたかな。

>運動量と絡むのだけは止めたほうが良い。無限領域だから無意味だが。

上の方では「23mが非現実的に大きい」のでダメって書いてあったけど、
今度は運動量が変化する領域が「無限大」だからダメってことかい?
こーゆーのが「論理的にメチャクチャ」って気がするが(藁

運動量で考えるのは理解できないかもしれないが、間違っていないから
安心して良いぜ。
それより翼の上面で流速が速くなる理由を考えてくれ。
825796:01/08/28 18:00 ID:BAjhMe96
>>823
もう一度>>811を読んでくれい。
いったんよどみ点が後縁に移動するとその付近の流速は周りにくらべて
非常に低いよね。つまり剥離を起こさず回り込み可能。

整理しとくと、
 ・出発時は流速が低いため剥離せず回り込み可能
 ・後縁によどみ点が一致した後はその周辺で流速が低下しており
  回り込み可能
って事。
826ご冗談でしょう?名無しさん:01/08/28 19:21 ID:PUzlKxyA
>>825
ええと、翼上面の後縁付近が低圧になっているので、そこに引かれて
上面の気流の速度が増加するってことでしょ。
で、その低圧部から出てくるときは逆に流速が遅くなるけど、
実際の速度分布はそんな流れになっているのかい?
827ご冗談でしょう?名無しさん:01/08/28 20:31 ID:81qMVKCk
>>821-822
君達、何か地面効果の意味を勘違いしてない?
特に822は書いている内容が支離滅裂で理解不能だよ。

>>821,824
あんまり、その数字の絶対値に拘らない方が良いんじゃ
ない?あくまで、一様な下降風を起こすとした単純
モデルの話しだから。感覚で申し訳ないが、オーダー的
には無理のない値だと思う。
828ご冗談でしょう?名無しさん:01/08/28 21:58 ID:81qMVKCk
827だけど、何度もスマソ。

ひとつ思い出したけど、誘導抵抗の原因となる翼端渦
が実際の翼に生ずるけど、この時の空気の流れは、翼端
を中心に片翼分の長さの円内の空気が、それぞれ内側に
巻きこむようになっている。

つまり、翼の上下方向にそれぞれ片翼分の長さの領域は
翼により流れが顕著に生成されていう事。これは翼端渦
の話しだけど、乱暴な言い方かもしれないが、さすれば、
上下方向に片翼分長さ程度の領域が下降風に影響してい
ると考えてもオーダー的には良いと思う。
829ご冗談でしょう?名無しさんん:01/08/28 23:25 ID:1zO1B5Sw
ちょとすまん。

迎え角のついた翼が静止空気を切って進行すると翼上面には
「真空」ができるわけだが、そこに空気が入り込む速さは
空気分子の熱運動速度と大体同じオーダであると思って良いのだろうか?

つまり失速速度は大雑把には空気分子の熱運動速度で規定されるというか。
(空気の熱運動速度ってどのくらいだったけなあ音速よりはだいぶ小さいだろうけど)
830796:01/08/28 23:48 ID:5SNL28Mw
>>826
低圧部だけ考えるとそうなるはず。ただし出発時には翼後縁で剥離
したり出発渦が出たりで実際の流れはより複雑になってると思うよ。
定常状態に移れば低圧部もなくなってるから上面の流れがそのまま
流出するだけだよね。別に流速が遅くなることはないよ。
何度もいうけど低圧部は定常状態から上面の流れが減速したりした
時にのみ発生するわけです。この時になら低圧部後方の微小な減速も
観察できるかも。
831ご冗談でしょう?名無しさん:01/08/29 00:25 ID:mtp8CKxo
>>829
分子の並進運動のみを考えると
 (1/2)m<v^2> = (3/2)kT
mは分子の質量、<v^2>は速度の2乗の平均、kはボルツマン定数、Tは絶対温度。
300Kの窒素分子(分子量28)だと<v^2>=26.48×10^4になったよ。
室温での分子の速さは500m/sくらいだな。誰かチェックしてくれい。
音速以下の飛行機なら「真空」はできないんじゃないの。
832ご冗談でしょう?名無しさん:01/08/29 00:30 ID:mtp8CKxo
>>830
>定常状態に移れば低圧部もなくなってるから上面の流れがそのまま
>流出するだけだよね。別に流速が遅くなることはないよ。

つまり、
定常状態では「低圧部」がないので、上面の気流の速さが増加することも無い
ってことね。
833796:01/08/29 01:00 ID:QXaACYS2
>>832
ちなみに定常状態では
 上面の流速>下面の流速
ですよね。
これで翼の上下面の流速差(というか翼回りの循環)は説明できたとおもいます。
834ご冗談でしょう?名無しさん:01/08/29 01:12 ID:mtp8CKxo
>>833
違うよ。
「低圧部」に引かれなければ上面の気流の速さは増加しないんだから
 上面の流速=下面の流速
でしょ。
835796:01/08/29 01:33 ID:QXaACYS2
>>834
まず
(1)
出発時に翼上面の後縁付近に「一時的」な低圧部発生
        ↓
これによって上面の流れが加速(下面側からは剥離によって回り込めず)
        ↓
したがって翼の上面と下面で流速差が生じる
    
ってとこまではいいよね。この状態ではクッタ条件を満たす様になってて
これを「定常状態」と呼びました。
この状態では
 上面の流速>下面の流速
ってのは良いですよね?

(2)
でもってここから上面の流速が減ったとします。すると
再び剥離が発生(低圧部発生)
    ↓
これによって上面の流れが加速
    ↓
定常状態に戻る

といった現象が起こるわけです。
この結果
 上面の流速>下面の流速
が保たれるわけです。OK?
実際の流れでは厳密には粘性やなんかによって微小な循環減少が起こるから
つねに(2)のプロセスが繰り返されて、微小な低圧部が存在してる事に
なりますね。
836ご冗談でしょう?名無しさん:01/08/29 02:50 ID:LfRQdbuY
それにしても、空気と翼が接している界面に相互引力が発生して、その上の空気が粘性
の為に一団となって、界面の空気が翼を引っ張るのをサポートする為(これは空気の慣性
も手伝っているので慣性力+粘着力というのが適切)、飛行機を支えるのに十分な力を
産み出すという説は考えれば考える程、興味深い。これを考えた奴は天才(天災?)だと
つくづく思う。

空気は基本的に翼の傾きなど認識しない。つまり、下降気流を決めているのは結局、
翼の上の表面の下がり方である。つまり、迎え角がつけばつくほど、そして翼の速度が
上がれば上がるほど、翼に接する空気が、下に引っ張られる速さが増す。
誰かが持ってきてくれた、翼の応力分布が、翼の前縁の一部に集中しているのは、そこで下に強く空気を引っ張っていることの証拠と考えられるであろう。この説を正しいと考えるのならば。

空気は、その部分で急激に引っ張られる。この付近の傾きと速度によって、翼と空気の
界面の下がる速度が決まる。翼の速度が速ければ、小さな傾きで、遅ければ大きな傾き
がついていれば、単位時間に下がるスピードはほぼ同等になるので、下降気流速度を
ほぼ同じにすることが出来るというわけ。

ここまでは、凡人の我々でも何とかわかるが、ここから先の発想が常人では無い。
何と、空気は翼の「速度」を認識し、粘性を勝手に変えるらしい。想像以上にインテリジェンスなオブジェクトであろうようだ。空気は
翼がゆっくり動いていると、粘性を増し、速いと粘性を勝手に小さくする。

で、低空では、巡航高度の数倍の高さの空気まで、粘性によって下降気流にするという。
それは翼が、低速で飛んでいることを空気が認知して、親切に粘性を変えてくれるらしい。
この空気の物体の速度認知機能を発見した、前スレ27氏の偉大さは、物理学史に残る
業績となるかも知れない。(でもこういう天才は往々にして生前は幸福ではないのは
歴史が教えるところか)

それにしても、意地悪な空気で、低速だと粘性を低くしてくれることは無いのだろうか?

>>前スレ27
837ご冗談でしょう?名無しさん:01/08/29 03:23 ID:mtp8CKxo
>>835
わかってないのは多分キミだけだから、メールでやっても良いのだが...

> (1)
>出発時に翼上面の後縁付近に「一時的」な低圧部発生
>        ↓
>これによって上面の流れが加速(下面側からは剥離によって回り込めず)
>        ↓
>したがって翼の上面と下面で流速差が生じる

ここまでは定常じゃなくて「過渡状態」だよ。この先、

        ↓
 低圧部に「余分」な空気が流れ込み、圧力が「徐々」に上昇する
        ↓
 すると上面と下面の流速差は「徐々」に少なくなる
        ↓
 低圧部が無くなると流速差も無くなる

これが「定常状態」ね。

流速差を保つためには常に低圧部が必要になるが、この場合、低圧部に
流入する速さと流出する速さが異なる。しかし、そのような速度分布に
実際にはなっていない。つまり、流速差を生じさせるような低圧部は
存在しないってこと。

(2)の変なところは
>でもってここから上面の流速が減ったとします。すると
>再び剥離が発生(低圧部発生)

定常状態では常に剥離は発生しているんでしょ。上面の流速が減ったから
発生するわけではない。

まだ、これ続けるのかい?
別のアイデアを考えた方がずっと建設的だぜ。
838前スレ27ではないが:01/08/29 09:15 ID:QsXuPNOs
隊員:「隊長! >>836で電波系煽ラーをハケーンしました!」
隊長:「まあ言っていることも良くわかんないし、ほっときなさい」
隊員:「ブラジャー! いやいやラジャー!」
839796:01/08/29 10:54 ID:11Aci2do
>>837
なんかうまく伝わってないみたいなんでもう一回書きますが、
>>825で書いたように後縁によどみ点が一致した状態(これを
「定常状態」と書いた)では後縁付近では流速が極めて低く
剥離は起きないですよね。
>定常状態では常に剥離は発生しているんでしょ
との事ですがつまり常に剥離してるわけではありません。

ところがおっしゃる通り実際にはそこから流れは徐々に減速
していきます。粘性の効果で循環は減少していきますから。
すると(上面の流速が徐々に減少すると)流れは再び上面側に
回り込み始め(すぐに剥離を起こさない理由は上で述べた通り)、
そして再び剥離を生ずると考えられます。
ただし、速度の減少は徐々に起こりますから、このとき起こる
剥離も小規模なものであり、発生する低圧部の圧力の周囲との差も
微小なものとなります。
したがってこの部分で極端な速度差が存在しないことになります。
ってな感じなんだけど。

ところで手っ取り早く循環は不変って説明じゃ駄目かな?
これだと楽なんだけど。
840ご冗談でしょう?名無しさん:01/08/29 13:57 ID:2FJQhgeA
>>839
シツコイぞ。圧力差が無ければ速度差も無いってわかってるんだろ。

>発生する低圧部の圧力の周囲との差も
>微小なものとなります。
>したがってこの部分で極端な速度差が存在しないことになります。

「周囲の圧力」って翼の下面の圧力と同じなんでしょ。そーしたら、次々と
隣との速度差を考えれば、翼の上面と下面で速度差はないってことになる。
つまりキミの考えではダメってことだ。
もう、この考えは捨てなさい。
841796:01/08/29 19:18 ID:1/ur8X.I
>>840
いや、だから
上面側で微小な流速減少=>微小低圧部が発生して加速
ですってば。
循環の減少はそれほど急に起こるわけじゃないから微小な
気圧低下で対応できるのでは?と思うんですけど。
842Ledの教え子:01/08/29 19:31 ID:0VR65prU
>>841
加速エネルギーの元を考えれば良いんじゃないの?
微小微小って叫んでみても、無から発生する訳じゃないでしょう?

# もうそろそろ止めなよ > 不毛な議論
843ご冗談でしょう?名無しさん:01/08/29 19:48 ID:shFX89fw
>>836の賞賛の意味が解らない人も居るみたいだから、私なりの836
の解釈を。

300tの飛行機は一秒間に300Gなる運動量を揚力が無い場合
獲得する。しかし、代わりに同じ運動量の空気を下向きに流した
ら反動で獲得した運動量は全て空気に与えられ水平飛行すること
が出来ることになる。

飛行機の速度で、一秒間に下降させられる空気の奥行が決まる。
勿論、それは秒速X1に等しい。
幅は、大体翼程度(2倍まではいかないだろうし、それ以上と
いうならば、その理由も大問題だが)。
となると厚さ(上下の幅)が問題。これが単位時間に発生
する下降気流の質量を決定する。

時速900kmの場合、迎角とかから時速90km程度の下降気流発生
と推測され、翼幅60mと仮定すると、下降気流の厚さは地上換算
で6m程度と計算された。(あくまでもエスティメートだが)
速度が仮に1/3になった場合、迎角を増さなければ、下降気流も
ほぼ1/3になるだろう。そうすると、9倍の45mの厚さを持たなけ
ればならず、数字がちょっと非現実的である...というのが流れ。

迎えを増して、下降気流の速度はそれほど下がらないから、厚さは
それほど厚くならない....という反論があった。
そもそも、この下降気流が粘性によるものならば、速度上昇に
伴って厚さが増えるというなら分かりやすいが、どうも逆の結論
を取らざるを得ないという点が変だよ
というのが836の主張じゃないの?>>836
844796:01/08/29 20:48 ID:rksreekw
>>842
実際の流れでは循環の減衰のおかげで翼の後縁部に余分な微小低圧部が
できるわけで、これは翼の形状から考えて抗力成分を含むことになるから
翼端渦の場合と同じで翼に対して余分なドラッグが増えることになる
と思います。よって翼は流れに対して余分な仕事を強いられると。
ただ、循環の減衰はそんなたいしたもんじゃないですよね?
簡単化のためにもこの際無視した方が良いのではと思います。
確か境界層の外はポテンシャル流として扱っても良いんじゃなかったっけ?
845ご冗談でしょう?名無しさん:01/08/29 22:15 ID:OITcsQ62
えー、こういう質問は、工学板のほうがいいと思います。
846ご冗談でしょう?名無しさん:01/08/30 00:32 ID:/2N6X8U.
>>843
あのエスティメーションの空気の量(高さ)は、全然「厳密」じゃない。
「そんなものでイケそーでしょ」ってカンジだな。ちゃんとケーサンする
なら、空気の流れが影響を受けている全領域で積分する必要があるね。
循環を求めるときと似てるかな。

あと、分子間力は、ほぼ分子間距離の関数。速度とは無関係。
よく使われるレナード=ジョーンズの(6,12)型のポテンシャルだと、
斥力は距離の12乗、引力は距離の6乗になってるけど、距離の大きな
ところでは、ポテンシャルの傾きの変化が小さいので、分子間力の差も
小さいか。ちょっと計算すると、1気圧の空気の平均分子間距離は、
ポテンシャルの極小点の10倍くらいだな。
847ご冗談でしょう?名無しさん:01/08/30 01:00 ID:cNTq3.5M
>>846
そんな柔なことじゃいけないな。
さっき飛行機飛んで、ガラスがビリビリ振動したけど、
空気の流れが影響を受けている範囲ってどこ?
もしかして、オレの背中や部屋の中の空気や、押し入れの中の振動まで
計算しろってこと?
やっぱり、下降気流が存在する以上、翼付近に運動量変化が集中するんじゃない
のかな?(それが揚力にどの位寄与するかは、工学板の問題とも絡んでくるかもね)
で、貴方は前スレの27さん?
分子間距離についてお詳しいようですが、もし良かったら、粘性と分子間力や圧力
速度(マクロな速度)等についての講義をお願いしたいのですが。
お忙しいところすみませんが。
848ご冗談でしょう?名無しさん:01/08/30 02:39 ID:qCxrK.vM
おばさんみたくネチネチと煽りながらも、放置プレーで24時間余りレスしない煽らーの
おかげで、形勢がヘンになっちゃったようだね。

ボクが前スレ27派を弁護してあげよう。前スレ27派の理論は別に>>836の疑問提起
で完全に論破されたわけじゃないということ。まだ、可能性を十分に残しているというこ
とだ。

そもそも粘性は速度場の勾配により、圧力とは異なる、非等方的な力が発生するという
性質。空気の性質であり、翼とは無関係。>>836の疑問は、ほぼ同じ下降気流の発生にも
関わらず、何故粘性の効果が発現する高さが異なるのか?ということのようだが、
下降気流が出来るプロセスについて考えると、の疑問は必ずしも不思議では無い。

下降気流が相対的に短い時間で発生すれば、粘性の効果は大きく現れるだろう。

例えば

速度が300km/h程度で迎え角が大きい時、例えば翼のある前後方向5cm程度の領域
で、真上の空気がそこを通過する時に引っ張られて一気に下降し、ある適当な高さの範囲に粘性が及びそれらの空気も下降する。

速度が900km/hで迎え角が小さいとき、今度は前後45cm程度の領域で、同じ速度の
下降気流が作られるとしたら、通過に要する時間は、3倍になる。(翼の速度が3倍であ
るのにも関わらず)
3倍の時間をかけて作られた下降気流、ほぼ同じだが、粘性の影響は今度は小さく
なる可能性も暗示している。

粘性は、空気をある方向に出来るだけ速く一気に動かすと、より強く発現する性質がある
から。

この可能性がまだ残されており、まだ前スレ27派の主張が否定されたとは言えない。
まぁ、下降気流による運動量が揚力の説明としてわかり易いという主張は、細かく見ていくと必ずしもそうとも言い切れないって感じはしてきたけど。

しかし、ここまで遡って考えると、もうこれは理論とかで検証することより、応力の実測
データ(迎え角・高度・気流速度を色々と変えたもの)で証明するしか無いんじゃないのか
な?ただ、軍事機密などの厚い壁が立ちはだかっている以上、その術は未だ皆無であろう
が。
849ご冗談でしょう?名無しさん:01/08/30 02:44 ID:/2N6X8U.
>>847
>さっき飛行機飛んで、ガラスがビリビリ振動したけど、
>空気の流れが影響を受けている範囲ってどこ?

振動ねぇ。「音」って、空気が流れていって伝わるのかい(藁
850ご冗談でしょう?名無しさん:01/08/30 03:14 ID:/2N6X8U.
>>848
別に27派の形勢が悪くなったわけじゃないでしょ。揚力のミクロなメカニズムを
理解できないデムパが、煽りで教えてもらおうとしてるだけ。前スレ27に「蟲」
されてたハズ。

>まぁ、下降気流による運動量が揚力の説明としてわかり易いという主張は、
>細かく見ていくと必ずしもそうとも言い切れないって感じはしてきたけど。

運動量で考えるのは直感的に「わかり易い」よ。で、理解し難いかもしれないけど、
その詳細についてもちゃんと説明できる。
ベル派は翼上面の流速が速くなるのをちゃんと説明できてないんだけどな。
「できた」とカン違いしてるケースはあるみたいだけど、メカニズムの説明が
いつの間にか「循環は不滅」とかになっちゃってるし、しばらくアタマを冷ます
べきだな。

で、理論で検証できないと思うのは何故?
まぁ、定量的な議論をしようと思えばできそーだが、面倒なのは確かだな...
851ご冗談でしょう?名無しさん:01/08/30 05:17 ID:XObzcg8U
>「わかり易い」よ。で、理解し難いかもしれないけど...

この表現は確かに理解しにくいですね。自己矛盾文ですかな?
それとも哲学板表現ですかね?
852ご冗談でしょう?名無しさん:01/08/30 10:45 ID:EV498xKc
>>851
こんなこと書いちゃって。自らの文章読解力の低さを宣伝することも無いだろ。
 一般力学的に「運動量を変化させた力積の反作用が揚力」なのはわかり易い
 分子運動論+統計力学的な扱いは、なかなか理解してもらえないから難しい
ってことでしょ。

コテハン796以外で「ベル派の説明」が出ないなら、このスレも終了かな。
結局「ベル派には揚力の説明は不可能」ってのがスレ的な結論。
計算はバッチリできるんだから、工学的には問題無いんだろーけどね。
揚力を間違って説明しているウェブサイトは無くなって欲しいと思うぞ。

27派に対する質問があれば答えても良いけど、ちょっと勉強すればわかる
ことは自分で考えようぜ。
あと、デムパはいやづら。
853796:01/08/30 12:39 ID:qO3JJrF.
>>852
循環が不変じゃなんかまずいかな?そんなにすぐに減衰する訳じゃないから
>>841の説明で問題無いんでは。それともこっち↓の疑問?

過去ログ見直したんだけどもしかして問題視されてるのはなぜ循環が翼回りに
とどまるかって事?
もちろん翼が渦を垂れ流しにしてたら翼回りの循環がガンガン減っていくことに
なるし、これを補うために無視できないほどの低圧部が存在することになりそう。
これも説明する必要があるみたいだね。
854ご冗談でしょう?名無しさん:01/08/30 14:50 ID:.LeSUPOg
>>853
そりゃ循環がゼロでなくなる理由を流体力学的に説明できれば、何の問題も無いよ。

キミが間違っているのは
 翼上面で一時的に生じた低圧部が消滅しても、流速は速くなったまま
って思い込んでるところ。流速を常に速く保つためには別の「力」が必要。
低圧部が消滅する過程で翼上面の流速は元に戻るって>>837に書いてあるけど、
理解できないのかい?

ベルヌーイの定理と合わせて考えると更に変だぜ。(藁

 気流は低圧部により加速されて流速が増す。
 速くなるとベルヌーイの定理により圧力は減少する。
 低圧部の圧力はさらに低くなり、流れをどんどん加速する。

さーどーする?
これの変なところがわかれば、間違いに気づくでしょう。
855796:01/08/30 19:15 ID:Fv5zw6.w
>>854
>そりゃ循環がゼロでなくなる理由を流体力学的に説明できれば、何の問題も無いよ。
ってそんだけでいいの?
一時的に上面で流速が増すってのは問題ないんだよね。
このときに単純に翼周りで循環を計算したら0では無いよね。

問題はこれがこの後翼周りに留まるかどうかじゃないかな?
>流速を常に速く保つためには別の「力」が必要。
ってのは翼の周りに循環がとどまらないという前提での事ですよね。

んでベルヌーイは
>速くなるとベルヌーイの定理により圧力は減少する。
>低圧部の圧力はさらに低くなり、流れをどんどん加速する。
ってこれは減少してるのは静圧だよね?
856ご冗談でしょう?名無しさん:01/08/30 21:33 ID:Wu.21xp.
>>855
なにをウダウダと誤魔化してる? ちょっとウザイ。

>>854に書いてある
>キミが間違っているのは
> 翼上面で一時的に生じた低圧部が消滅しても、流速は速くなったまま
>って思い込んでるところ。

これにちゃんと答えてみればいいだろ > 796
857796:01/08/30 23:17 ID:7z69sroA
>>856
>キミが間違っているのは
> 翼上面で一時的に生じた低圧部が消滅しても、流速は速くなったまま
>って思い込んでるところ。
>これにちゃんと答えてみればいいだろ
答えてるつもりですが…
これは翼回りの場合の一様流+循環では通用しないでしょ?
たとえば管内流れだったら分からんでも無いが。
この場合加速(循環発生)したらしっぱなしだと思うんですが。
>>853で書いた通り出発時にできた循環は翼が渦を放出するなり粘性で減衰するのを
気長に待たない限りはなくならないから翼上面の流速は速くなったまま。
仮に減衰しても>>835の(2)によって流速は維持される。

あなたが不満を感じるのは一体どれに対して?
1.この説明ではそもそも循環は生まれない
2.循環が生まれても翼のそばに留まることは無い
3.低圧部がなくなれば循環も無くなる
858796:01/08/30 23:19 ID:7z69sroA
下げちゃった
859ご冗談でしょう?名無しさん:01/08/30 23:20 ID:DO/n6M2M
860ご冗談でしょう?名無しさん:01/08/31 01:08 ID:TGiYtU8M
>>857
>この場合加速(循環発生)したらしっぱなしだと思うんですが。

で、上面の流速は常に維持されるってことだから、飛行機の速度が低下しても
揚力が小さくなることはない。
つまり、一度離陸した飛行機は着陸できないって結論だぜ (藁

>あなたが不満を感じるのは一体どれに対して?

平気でウソを書けるところかな。ワザとか?
あと、循環と循環流は区別できてる?
861ご冗談でしょう?名無しさん:01/08/31 01:12 ID:h7M.Vv0.
傍観者として高みの見物をしてずいぶん楽しませてもらったが、このスレ的な結論?
は次のように分類されると思われ。私なりに長所と短所に分類してみた。
(改行が余り出来なくて読みにくくてスマソ)
・標準派>>ベル派
音速より低い速度を前提とした時の翼上下気流同時到着理論=翼上気流
水平速度不変説 とベルヌーイの等式を組み合わせて、翼上下の気圧差を
演繹する。揚力はこの上下の圧力差。空気は渦なしの完全流体と見なしてい
る。
長所:伝統的に長期に渡って信頼性の高い説明とされてきた。圧力と気流速度
の間に一方が増えれば他方が減るという関係は、受け入れやすくそれでいて
厳密性は低くない。
短所:気流同時到着説がどうも実験事実と合わないという人が居るらしい。
音速に近くなると、完全流体近似は正確であるとはとても言えないので、
亜音速以上の飛行機の揚力の説明には向かない。因果関係でなく、圧力と
速度の論理関係で説明されており、そこらへんが誤魔化されたという気がする
人が多い。
・標準派>>クッタージューコフスキー派
無限領域(地面に相当する境界条件無し)・2次元での特殊理論であるが、
厳密に数式で表せる流れの場を用いて、揚力を説明する。揚力は、循環という
物理量で表現される。翼型障害物の場合も、厳密な式が得られ、揚力が発生
することが計算される。(抗力は発生しない。)粘性がある場合、圧縮性がある
場合についても、拡張されているという。この場合、翼の周りの局所的な単位
時間の気流運動量変化と翼にかかる揚力の関係は必ずしも明確では無い。
長所:厳密な式で表され、結果も覚えやすい。正確さという点ではピカ1.
短所:流れに速度∞の点があるなど、物理的実現性(数学的実現性も含め)
を疑う人が多い。2次元無限領域という特殊条件が、地面という重要な
境界を持つ現実の条件とどれくらいマッチしているか、今ひとつ不明で
その点は信頼性が低い。複素関数論を使っているので、とても初心者向けの
説明とはいえない。重力の影響が循環の中に隠蔽されてしまう点も初心者
向けでは無い。
・粘着粘性下降気流派=(旧スレ27派?)
翼と空気の界面に強い引力が働き、翼表面に空気分子が付着する。
その分子と、翼上方の空気分子の間の分子間力が、マクロには、粘性として
発現する。翼が、短時間に上表面の空気を下に曲げて流すと、上の空気も分
子間力で下に引っ張られ、反作用で翼表面の分子に上向きの力をかけるので
翼は表面空気分子をより強い力で引っ張り、最終的にその反作用が揚力で
ある。
長所:気流獲得下向き運動量と揚力の間の関係がすっきりとしている。
短所:翼の上表面の形状に非常に敏感に揚力が影響を受けそうである。
特に低速では安定した揚力が得られるか疑わしい。
・衝動派(=曲げ派?)
翼下面で、前方から侵入してくる空気を下に押し出した反動が揚力である。
長所:最もわかりやすい。
短所:上面について何も考えていない。都合よく、下にのみ流れる理由がはっきり
しない。(サイドに散逸しない理由が、はっきりしない。)低速な場合について
粘着粘性下降気流派よりも疑わしい。
・空洞派(=???)
意味不明な理論であるが、空気を完全流体と見なし、慣性の影響でそれが翼上 表面から離れて運動することも認め、空洞(Cavity)の存在を認める。
現実の空気は、膨張するので空洞は、周囲から流入の膨張低圧空気で満たされ
(その空気は空洞表面に接する空気集団から均等に貰う)ることになる。翼上下 の圧力差はこれで説明される。
・長所:上下の圧力差発生を直接説明している。
・短所:定量的説明が全くない。超音速領域等、圧縮も局所的に起こる場合や、
粘性の影響等が考慮されていないので、信頼性に欠けている。
どれが一番解り易いか、説明としてどれが最も適切であるかの評価は別問題として、
どの「派」にもそれぞれ長所と短所があるようで....まだ決定打に欠けていると思われ。
862796:01/08/31 02:23 ID:HSyb8V46
>>860
循環と循環流の違いは…
循環Γは閉曲線上で流速の接線方向成分を積分
循環流はF=-iΓ/2πlogzな流れ
だったっけ?正直言って自信ないです。
んなもんでウソついてたら言って下さい。平気でカマしてる恐れあり。

それと着陸の話(というかいかに循環を減ずるか)ですが、
>で、上面の流速は常に維持されるってことだから、飛行機の速度が低下しても
>揚力が小さくなることはない。
飛行中はその対気速度・迎角におけるクッタ条件を満たす様な循環がありますよね。
ここでたとえば速度を減らしたとすると、クッタ条件を満たす循環も小さくなります。
したがって、余剰循環があるわけで、流れが(つまりよどみ点も)翼の下面側へ回りこみ、
剥離を生じて例のパターンの逆が起こって循環は減少します。
イメージとしては発進渦と逆の渦を放出してる感じです。

ところで実際の飛行の場合には定常下降するときは揚力を減らしてるわけではなくて
推力を減らすことで降りてたんじゃなかったかと思います。
機体に働く揚力・抗力・推力・重力の内推力を減らせば釣り合い飛行で下降していけますよね。
速度を減らしたければ迎角を増すことで揚力を保つと。
さらに接地した後にはスポイラーを使って積極的に循環を減らしてたはず。
863ご冗談でしょう?名無しさん:01/08/31 02:40 ID:NTEhPVK6
>>861
いやぁ、よくまとまってるねぇ...と言いたい所だが、ベル派の短所は
アマクなっているので、テイの良いベル派の擁護になってるな。

ベル派の短所:
 気流同時到着説は根拠なし。
 翼周りの流れを説明できない。計算しないと速度場がわからない。
 「誤魔化されたという気がする」レベルじゃなくて、全然ダメってこと。

クッタ‐ジューコフスキー派(?)の短所:
 粘性を考えれば、遠方では流れは減衰して消滅しているので「無限領域」の
 問題は無い。速度が無限大にならないようするのがクッタの条件。
 この辺は短所じゃないね。
 与えられた流れに対して計算はできるが、逆に流れを決められないのが短所。

27派の短所:
 翼の形状が揚力に影響するのは、粘着の条件を使うベル派などと同程度。
 あと低速で揚力が不安定になるわけではない。どちらも短所じゃないね。
 空気を連続体として扱う立場からは「理解し難い」ってのが短所かな。

曲げ派と空洞派は、そんなもんでしょう。その他の説明や長所に関しては
だいたいオッケーだと思います。

とゆーことで、決定打は27派かな (藁
864ご冗談でしょう?名無しさん:01/08/31 02:55 ID:NTEhPVK6
>>862
>ここでたとえば速度を減らしたとすると、クッタ条件を満たす循環も小さくなります。

つまり、翼上面の気流が高速のままで維持されるってのは、ウソだね。

>>860

>で、上面の流速は常に維持されるってことだから、飛行機の速度が低下しても
>揚力が小さくなることはない。

ここは「循環が小さくなることはない」の間違いか。
揚力は翼の速さと循環の積に比例だな。
865ご冗談でしょう?名無しさん:01/08/31 07:15 ID:q4BZIqHQ
age
866796:01/08/31 23:34 ID:ORdvYlkw
>>861
同時到着説はよく信じられてるけど実際には上面のほうが早く到着してます。
これはよく教科書なんかにスモークワイヤ(だっけ?)とか使って示してあるんで
完全な迷信と考えてもいいのでは?

>>864
説明不足でしたね。スマソ。

正しくは
「翼上面の流速が飛行速度・迎角に応じたクッタ条件を満たすような
速度で維持される。飛行速度・迎角を変えた場合には前述のメカニズム
によって翼回りの循環を変ずることによって翼上面の流速も変わる。」
ですね。このときに出るのってshed vortexだったっけ?
867ご冗談でしょう?名無しさん:01/08/31 23:34 ID:corbjh9I
このスレどころか、2ch全体が無くなるとかで、そろそろお開きにしたら?
868861:01/09/01 00:43 ID:VzwQVjG.
別にベル派が、説明不足であるという結論に皆納得したわけじゃないと思うね。

確かに、「何故、翼上通過空気自身がベルヌーイの法則を知っているが如く、勝手に圧力を下げるのか(=速度を上げるのか?)」という疑問は決して哲学の議論では
無く、純粋に物理的な議論の余地が残るところ。このスレではこの点が強く叩かれた。

空気が圧力を上げ下げする理由は内部応力にある。それは密度分布とその慣性が関連
していて密度が流れによって運ばれることから内部応力(圧力・粘性)が発生する。
この関係で、圧力や速度の非一様性が発生する.....という「説明」を入れておけば
問題は無いんだが、何故これに言及しないのか?

それはダランベールのパラドックスとの絡みで、明確な因果関係を言い切れない
から、わざとその点をぼかしてるんだと思う。(上向の揚力が発生する流れになる
ということが原理的に断定できない)
だから、ベル派の説明としては、揚力が発生しているのを認めている人に対して、近
似的に同時到着が成立していることを言って、ベルヌーイ・オイラーの公式を援用し
て、圧力が低下していることを認めさせるという方法で説明する。あくまでもエステ
ィメート。

圧力分布で都合よく上向き揚力が発生するものにならない可能性も大いにあるの
だから。(それは幸運にも上向き揚力になることを否定するものでは勿論無いが)
(その辺の事情は27派以外全て同じで、27派の人の他派に対する「いちゃもん」
は、正にそれに言及したもの)
失速する場合も、ベル派の説明は皮肉にも当てはまっている。

といったところが、ベル派の最大公約数的回答ってところじゃないの?

現象を肯定する理由が、実は否定する理由でもあったという点が面白いよね。

因みにベルヌーイは17世紀のフランス数学者でオイラーは19世紀のロシア数学者。
現在知られている式はベルヌーイよりも、オイラーによって明確化されたもの。
-------------------------------------------------
このスレのスーパースターである、粘性下降気流反動派こと前スレ27さんの説明は
斬新である。何よりも、揚力が分子間力に起因し、それゆえ確実かつ永続的に起こる
ということを主張するから。(他の「理論」では、途中まで揚力が発生してもそれが突然
失われる可能性を否定出来ない。)失速も、起こりにくいだろう。というより起こる理由を
見つけにくい。
飛行機に依存する部分が多い、現代人にとってこの説は実に都合が良い。

他にも運動量理論や分子運動論との相性も良く、わかりやすい。ビジュアル化しやすい
などの数多くの利点を持つ。
出来ればそうであって欲しいと思うのだが、まだかなり重大な困難がある。
それは、800番台中間当たりのレスで誰かが指摘していたことだが、「低速で巡航飛行
すればするほど翼の相対的に小面積な部分に応力が集中し、翼に負担をかけざるを得
ない」という点。

これは残念ながら、常識にも事実にも反する。27派の理論を発展させていく為にはこの
困難をどうしても乗り越える必要があるだろうね。
869ご冗談でしょう?名無しさん:01/09/01 01:17 ID:rbfqwpoA
>失速する場合も、ベル派の説明は皮肉にも当てはまっている。
ってどういう意味?うまく読み取れん。
それと
>失速も、起こりにくいだろう。というより起こる理由を見つけにくい。
ってのはむしろヤバくないかな?失速なんて起こそうと思えばすぐ起こせるけど…
迎角にして10度ちょっとで起こっちゃうよ。
870861:01/09/01 03:15 ID:W2SirIM.
>>869
>失速も、起こりにくいだろう。というより起こる理由を見つけにくい

勿論、現状での(このスレで説かれた)粘性下降気流説(=前スレ27説)に
揚力の殆どを依存している場合についてです。

恐ろしいことを言わないで下さい。現実には失速はパイロットの操作ミスや
マイクロバーストなどの気象条件により、簡単に起こってしまいますから。
871前スレ27ではないが:01/09/01 06:01 ID:n7fiLCRk
比喩が多いからか、私の読解力が足らないのか判らないが、
>>868(=861)の27説への反論が理解できない。失速は
また別の現象だし、低速時の翼の負担に至ってはそんな
記述はあったっけ?

そもそも、27支持派で標準派説明(近似としてのベルヌーイ
の定理での説明も含む)を否定している人は居ないと思うぞ。
標準説明で気に入らないのは、前スレ27氏も言っている
ように、なぜ上面の流れが速くなるかの説明がない事だよ。

それはすなわち、標準派(ベル派)が、翼が空気の流れを
下向きに曲げるという「揚力発生の本質」を考えていない
という事じゃない?

---- 長いので続く -------
872前スレ27ではないが:01/09/01 06:02 ID:n7fiLCRk
---- 上の続き ----------

空気にしろ水にしろ、粘性がある流体は、曲がった面があ
ればそれに沿って流れる性質があるはずだよね。その時に
曲がりに対して凸の側の流速が速くなるんだよね。(これ
って感覚的に理解できるんだけど、どなたか物理・工学的
に説明できます? これこそまさに前スレ27氏が標準派
(ベル派)に証明しろと言っている事だと思うけど)

翼で言えば、(迎え角を持った)翼の上面に沿って空気が
流れるから、(+下面に沿っても流れるから)空気は下向
きに曲げられる訳だけど、上の理屈で上面の速度が速くな
って、上下面で圧力差がついて、揚力が発生する訳だよね。
(もちろん、下向きに空気を曲げた反作用で揚力発生とい
ってもOK)

標準派の説明はわかるんだけど、「翼は空気を下向きに曲
げるんだけど、その結果として翼周りを見ると....」とい
う本質を語らずに、説明を始めるから変なんじゃないの?


ちなみに私は27支持派=マクロな意味でのコアンダ効果
支持派=粘性派。前スレ27氏の分子間力での説明はちょ
っと理解できないので興味深く見ている。ガイシュツ記事だが、
下記のアンダーソン氏の考え方に私は近いと思う。
http://journal.msn.co.jp/worldreport.asp?id=010529ns_airplane&vf=1
873前スレ27ではないが:01/09/01 06:18 ID:HKWOJ8rc
何度もスマソ、ちょっと補足。

>>872
>曲がりに対して凸の側の流速が速くなるんだよね。

水が流れているとして、流れの方向に対して30度で
衝立板が立っていると考える。水は衝立板で分けら
れて、その分けられた流れはどちらも、板の傾いて
いる方向へ曲げられるけど、その時の曲がりの外側
の流れの方が、内側より相対的に速くなること。
874ご冗談でしょう?名無しさん:01/09/01 08:51 ID:OgvrijQc
>>873
>粘性がある流体は、曲がった面があ
ればそれに沿って流れる性質があるはずだよね。

正しいような正しく無いような...すみません。私の理解では
そういう感じがします。貴方の思い込みだって気がします。
流体に粘性があるからといって、それだけでは、面に沿って流れ
る必要は無いような気がするのですが。(流れることは否定しま
せんが、必ずしもそうなるとは限らないと思います。)
粘性は、粘着性とは違う概念と理解しているのですが、違うので
すか?
粘着性があれば、そういうことも言えるかも知れませんけどね。
粘性ってのは、圧力のように周囲に平等に力を与えるという
わけでなく、速度勾配のある方向に差別的に力を与える性質
って習った気がするんですが....

空気だけでも勝手に曲がるということは、渦運動を見てもある
ので、翼上面の空気が曲ったように見えても、それが翼が引っ
張ったものなのか、それとも空気が勝手に曲ったのか即断する
ことは出来ないと思います。勿論、翼が引っ張ってそういう運動
をするなら、確かに空気の運動軌跡が曲ったことの反作用が揚力
の一部になったということは言えると思います。
875前スレ27ではないが:01/09/01 09:26 ID:7gZeRMZI
このスレ見てて思うんだけど、粘性と粘着の混同、
あるいは空気の分子間力と空気分子/翼原子の相互作用
の混同をしている人が多い気がする。(>>874を含め)

以下、ガイシュツの話しだと思うが、
「粘性がある」

「せん断応力が発生する」

「流体が壁/地面/翼等に沿って流れるとき速度勾配
が生ずる(壁面/地面/翼面では0、離れるにつれ流速
が増え、ある程度離れると一定)」

って事じゃないかな。この速度勾配をイメージした
時に、壁面が凸に曲がっていれば壁面に沿って流れ
ていくのが理解できると思うけど。これが、空気に
粘性があるが故に翼上面に沿って曲がるってことだよ。

翼が空気を引っ張るとか、空気が翼に粘着性を持つ
とか、そういうデムパな事を言うのはやめてくれぃ。
876ご冗談でしょう?名無しさん:01/09/01 12:34 ID:hswjwgLY
>>875
「翼が空気を引っ張る」は、悪くないでしょ。
877前スレ27ではないが:01/09/01 12:59 ID:7gZeRMZI
>876
言葉のあやかもしれないが、そうすると流体と翼の間に
引力が働いているイメージがないか?事実、このスレで
はそう誤解している輩が大勢いる。よって使うべきで
ないと思う。あくまで流体が粘性という性質を持ってい
るために、翼という障がい物の上面に沿って流れるだけ。
878 :01/09/01 17:11 ID:BXRB2jlk
zennzenn susunndenaine.
879ご冗談でしょう?名無しさん:01/09/01 19:42 ID:UITaw5rk
>>877
空気に対する力積の反作用として、翼は上向きの力を受けるんでしょ。
少なくとも翼の上面では「空気を引っ張る」しかないよ。
880ご冗談でしょう?名無しさん:01/09/01 19:55 ID:UITaw5rk
>>866=796
>翼上面の流速が飛行速度・迎角に応じたクッタ条件を満たすような
>速度で維持される。

ならば、

>出発時に翼上面の後縁付近に「一時的」な低圧部発生

どーして「一時的」な低圧部が発生するの。ゆっくり速度を上げれば常に
「クッタ条件」は満たされる、つまり剥離点は常に翼の後端にあるんでしょ。

キミの考えを「ウソぴょん」にしてるのは、
 「一時的」な低圧部により生じた流速差が「恒常的」に維持される
ってとこ。種々の条件が変化しても「剥離点」が翼の上や下に回り込んで
なんとかするってのが、都合よすぎ。
例えば

 翼上面の流速が遅くなれば、剥離点が上面に回り込んで気流を加速する。
 翼の速度が遅くなれば剥離点が下面に回り込んで上面の流速を下げる。

こんなことを言われても信じられないぜ。
下面に回りこんだ剥離点は何で下面の気流を加速しないの?

説明に一貫性が無いぜ。言い訳だらけのご都合主義ってカンジなんだが、
結構ちゃんとわかってるみたいなので、これまでのは無かったことにして
もう一度やり直して欲しいと思うぞ。
881 :01/09/01 21:09 ID:hWd4CF5M
>あくまで流体が粘性という性質を持ってい
>るために、翼という障がい物の上面に沿って流れるだけ。

どうしてこういうことが断定出来るのか?
まぁ、流体の方程式を使って議論する場合、上面に沿って流れることを
仮定するけど、それは数学的な簡単さを少しでも得る為で、それは決し
て自明なことじゃないと思うがね。
少なくとも、上面から離れて流れたってなんら不思議ではないと思うが。

トイレの便器をちょっと観察して御覧なさい。洋和式問わず、水が溜ま
っていて、そこに大小便が流れ込む。水溜まり付近はスロープになって
いて、水が流れる時、そのスロープを伝って水が水溜まりに落ちて行く。
その際、一瞬ではあるが、水はスロープを離れて流れ、水溜まりに落ちるよ。
粘性があっても、上面に沿って流れない場合の典型例だと思うが。
882ご冗談でしょう?名無しさん:01/09/01 23:41 ID:9jPCY496
>>881
>少なくとも、上面から離れて流れたってなんら不思議ではないと思うが。

粘着の条件って知ってるかい?

>粘性があっても、上面に沿って流れない場合の典型例だと思うが。

だから何? これって翼の場合だと迎角が大きすぎて失速した状態でしょ。
どーでもいーけど、バッチィ例えなので詳細をソーゾーしたくないぞ。
883前スレ27ではないが:01/09/02 00:07 ID:Nr2gMvJM
>879
湖の上で小さい手漕ぎボートに乗っているとする。乗って
いる人が東に向かって飛びこむと、ボートは西向きに力を
受けるので、西向きに進んでいくよね。「ボートが人間を
押し出した」から西向きの推進力を得たって普通言うかい?

人間がボートを飛び出すときに蹴った力の反力を意識して、
「ボートが人間を押し出した」からボートが西向き推進力
を得る。というのは間違いではないかもしれないが誤解を
招く表現ではないかい?

今の場合、空気と翼の間には引力は働いてないよね。翼が
意思を持っているわけでないよね。それを理解した上で、
自分自身でそのちょっと変わった見方の言い方をしても、
物理的に間違いではないかもしれないが、唐突に「翼が
空気を引っ張る」なんて言うとデムパとしか思えない。特に
このスレでは。
884ご冗談でしょう?名無しさん:01/09/02 00:40 ID:Yez9vK.2
>粘着の条件って知ってるかい?
粘性のある流体が、境界に空洞を作らないように流れる条件です。
特別な場合においては、流体速度@境界=0とします。
流体の方程式を扱う場合、この条件を仮定するのは、流体の方程式が
難しすぎて、この程度の仮定をつけなければ到底扱えるシロモノでは
ないからです。
この境界条件は、決して不当なものではなく、定常状態では、この
境界条件を満たしているのでは無いかと推測されますが、過渡的状態
ではどうかというと、トイレの便器の例でわかるように、必ずしも
条件を満たすとは限らないと考えるのが妥当なようです。また、過渡的
状態から定常状態に近づくか?という問いさえ現代科学でははっきり
していません。
以上
885796:01/09/02 00:46 ID:e8isXvfU
>>880
>下面に回りこんだ剥離点は何で下面の気流を加速しないの?
これは、もちろん下面で加速するような働きをします(つまり循環が減る作用)。
実際の流れは循環+一様流なので結果としては下面の流速は小さくなりますが。

そして、
>翼の速度が遅くなれば剥離点が下面に回り込んで上面の流速を下げる。
>こんなことを言われても信じられないぜ。
この仕組みについての説明がやはり必要ですよねえ。
上下面での加速減速がペアで起こってないと、実際の現象的にもおかしいわけですから。
今まで以上にご都合主義的な気もしますが(その辺は認識しております)、
翼の前縁付近の流れを考えてみると、後縁と違って上下面の明確な境界はありませんから
下面で加速すると上面側の流れも下面側に引っ張り込まれ、その結果上面で減速といった
イメージではどうでしょうか(逆の場合も同様)。ただ、平板翼の場合は厄介ですね。
886ご冗談でしょう?名無しさん:01/09/02 01:03 ID:pQHS8y3w
流体は粘性によって自分で曲がる」というこれまたデムパな主張の
間違いを示す好例であることは間違いないと思われ。
今、考察されているのは、流体が粘性で翼に沿って流れるから、上下
で圧力差(速度差)が生じ、揚力が発生するという主張(>>872)
粘性と翼に沿うことは直接関係無いから、粘性があるから翼の上の空気が
早くなるという考えも正しいとは言えないと思われ。
887ご冗談でしょう?名無しさん:01/09/02 01:04 ID:hu8//5pI
>>883
>今の場合、空気と翼の間には引力は働いてないよね。

ええと、翼にどーやって揚力が発生するのか、もう一度書いてちょ。

>>884
トイレの話は水と空気の2流体があるので、例としては不適切じゃないかな。
水没させて水を流さないとダメでしょ。

>>885
「一時的」な低圧部はどーしたの? 別に急がないから、ゆっくり考え直してから
まとめてね。

ところで「2ちゃんねる終了」ってネタなの?
888ご冗談でしょう?名無しさん:01/09/02 01:05 ID:pQHS8y3w
トイレの例は確かにバッティー話で、想像したくは無いが
流体は粘性によって自分で曲がる」というこれまたデムパな主張の
間違いを示す好例であることは間違いないと思われ。
今、考察されているのは、流体が粘性で翼に沿って流れるから、上下
で圧力差(速度差)が生じ、揚力が発生するという主張(>>872)
粘性と翼に沿うことは直接関係無いから、粘性があるから翼の上の空気が
早くなるという考えも正しいとは言えないと思われ。
889前スレ27ではないが:01/09/02 01:56 ID:Xu0CfWyw
>887
>ええと、翼にどーやって揚力が発生するのか、もう一度書いてちょ。

27支持派としては、下記のようになると思う。
すなわち、下記参考リンクにあるように、空気に下向きの
運動量を与える反作用として揚力が発生する。
http://www.allstar.fiu.edu/aero/airflylvl3.htm
http://journal.msn.co.jp/worldreport.asp?id=010529ns_airplane&vf=1

27派は、ただし、それを別の観点から見ると、翼上下面
の圧力差(流体の速度差)が揚力となっていると言っている。
ちなみに俺は、>>872で言っている様に、翼上面で流れが
速くなる原因はわからないし、逆にそれを教えて欲しいと
言っている。

ちなみに空気分子/空気分子間の相互作用や、空気分子/翼
の構成原子間の相互作用については、上記モデルでは考えて
ない。これらを使った説明は「前スレ27氏」が試みている
が、興味深く見ている。

補足だが、27支持派とは、「前スレ27氏」の発言(特に
分子間力や空気分子/翼構成原子の相互作用)の支持派という
意味でなく、上記2リンクの内容の支持派という意味。

>886=888
何か、反論の「根拠」が提示されてないよね。
コアンダ効果っていう物理現象知ってるよね。なぜ起こる
か知ってるよね。
890前スレ27ではないが:01/09/02 02:00 ID:Xu0CfWyw
ちなみに、俺がここに書きこんだのは、
>>868(=861=870)の27説への反論が良く判らなかったので、
何を言っているのか訊きたかったからなんだけど、彼(女)
はどっか逝っちゃた?
891ご冗談でしょう?名無しさん:01/09/02 02:11 ID:558wAGfI
粘性は、小さいうちは、流体の内部歪を適切に分散させ、流れを調和させます
が、大きくなるとそうなるとは限りません。歪を解消するのに必要な自由度を低下させる
ことになるからだと予測されています。

翼の上を通る流体は、例えその上を滑らかに流れることができた場合でも、内部歪を発
生させざるを得ません。基本的に流体を構成する要素は直線運動をしようとするから
です。

その内部歪が応力として、圧力や粘性力を生み出します。前述の通り、圧力だけでは
流体の流れは調和するとは限らず、粘性が調和を促進するのに大きく寄与すると考え
られています。

通常の速度ではこの調和作用が効果を発揮し、コアンダ効果が出現します。
(空気が滑らかに流れることだが、翼の上に沿って流れることを直接主張しているわけ
ではありません。)

しかし、高速になると、粘性の効果は逆に乱流を生み出す原因とされています。
乱流は飛行機にとっては一般的に言って敵ですが、適度な乱流は、かえって失速を予防
するとされています。現に、翼後部にわざと乱流を発生させるVoltex Generatorという
装置を取り付けた翼も存在しします。

翼の上を空気が滑らかに流れる場合、形状からいって、圧力勾配が、内部応力の反映
として、発生することは不思議ではありません。これは、曲がった軌道を取る為に発生
した内部の歪を解消しようと、空気自身が発現させた力で、翼には直接関係はありま
せん。ただ、前縁で圧縮される可能性がある以上、圧力勾配があるからといって上面
が下面より低圧であることは主張できません。状況によっては上面のほうが高圧に
なる可能性があります。この場合は負の揚力になってしまいます。ですから、現時点では
正の揚力が発生することを前提として、上面の圧力低下を認めることになります。
892ご冗談でしょう?名無しさん:01/09/02 03:39 ID:hu8//5pI
>>889
なんかベル派みたいだぜ。

>空気に下向きの
>運動量を与える反作用として揚力が発生する。

ならば、翼と空気の間には引力か斥力が作用しないとダメだよ。

シアーの応力だけで流れを曲げようとすると、直感的には流速は
遅くなるハズだが、実際には速くなっている。ベル派の泣き所だな。

>>891
う〜ん、何が言いたいのかな。ベル派の説明がダメってこと?
893前スレ27ではないが:01/09/02 09:49 ID:tjtXnHkY
今まで自分は27支持派(もちろん圧力差が揚力)だと思ってたん
だけど、>>889 に書いたように、前スレ27氏が試みている分子
間力による説明は否定も肯定もせず、興味深く見ている立場だ。
すなわち、>>889 のリンクにあるような考え方の支持者ということ。
「27支持派」という言葉は分子間力の説明も支持するという事に
なるのかな?とすると、名前を変えねばならない。「反作用派」
あるいは「正統派」とでも。(もちろん圧力差の積分で揚力計算可)

---- 続く ---------------
894前スレ27ではないが:01/09/02 09:51 ID:tjtXnHkY
---- 上の続き -----------

>>892
>なんかベル派みたいだぜ。
「反作用派」はベルヌーイの定理を厳密に適用する事はさておき、
標準派の説明で揚力は計算できるという立場だから、似てるのは
当然かもね。あくまで空気の粘性で説明できるマクロ的な見方。

>翼と空気の間には引力か斥力が作用しないとダメだよ。
言葉のあやの問題かもしれない。マクロな物体である翼とマクロ
的にみた空気には引力も斥力も働かないのは良いよね。>>883
出したボートの作用反作用の例を考えて、ボート上にたくさんの
人間が居て、次々に飛び込むとしようか。人間はボートを蹴る事
によりボートに力を加えて、その反作用で人間はボートから押さ
れるよね。これって、ボートと人間に引力or斥力が働くってこと?
言いかえれば、翼と空気分子に引力or斥力が働くってこと?
もし、翼と空気(分子といいたいのかな?)に引力or斥力が働け
ば、地上に駐機中に翼周りの空気密度が増えていくか減っていくよ。

ただし、分子論的に見ると(見て良いとすると)、空気分子に
平均的に下向きの運動量を増やした反作用で揚力が生じるのは
理解できるよ。

言い方の問題でしょ。言いたい事は理解できるから、言葉変えれば?
895前スレ27ではないが:01/09/02 10:24 ID:fm.YYekk
いろいろ考えている内に、「反作用派」の立場から見た「標準派」
が説明できない翼上面での流れ速度の増加が定性的に説明できる
ような気がしてきた。同じような考えはあったかもしれないけど、
以下に簡単なモデル書くんで、間違い等あったらどなたか指摘して。

翼上面では、粘性のために空気流れは上面に沿って流れようと
する。しかし、元々前方から流れてきたのだから、慣性で真っ直
ぐ後方にも流れようとする。この2つの力の兼ね合いで、空気に
膨張作用が働く(←つまり上面で低圧部生成)。圧力と流れの
速度の間には一定の関係(特殊な場合がベルヌーイの関係)が
あるから、低圧になった分、上面の流れが速くなる。

翼は迎え角を持っており、また粘性のため空気は上面、下面に
沿って流れ、更に翼の後縁が鋭利なため、翼周りで空気は後縁
方向に曲げられる(下向きの運動量を与えられる。)

この曲げられた空気の反作用で揚力が発生するといっても良い
し、翼上下面の圧力差を積分しても揚力(正確には+抗力)に
なる。

翼上面の低圧部が定常状態で保持されるのは、常に定常前進し
ているからで答えにならない?ちなみに、翼端をぐるっと回る
翼下面から上面への流れ(翼端渦)もあるけど、これの効果も
含めた上での話しね。

標準派は「翼が迎え角を持っており、空気の流れを下方向に
曲げるんだけど、その時の翼上下面の流れを見ると.....」と
いう**揚力発生の本質**から説明を始めれば、問題無いので
はないかと思うけど。
896ご冗談でしょう?名無しさん:01/09/02 10:46 ID:hu8//5pI
>>894
運動量保存則しか頭にないのかな。
物体の運動量を変化させるのは力積だろ。
で、面の法線方向に作用する「力」を考えれば、向きは+か−しかないぜ。

>人間はボートを蹴る事
>によりボートに力を加えて、その反作用で人間はボートから押さ
>れるよね。これって、ボートと人間に引力or斥力が働くってこと?

押してるんだから「斥力」だろ。
正確には靴の裏の「電子」とボートの縁の「電子」の相互作用だがな (藁

>もし、翼と空気(分子といいたいのかな?)に引力or斥力が働け
>ば、地上に駐機中に翼周りの空気密度が増えていくか減っていくよ。

これはちょっとマヌケすぎないかい。
力の釣り合いとか平衡状態とか知らないわけじゃないだろ。

空気をカタマリ(連続体)で考える傾向にあるので
キミは「ベル派みたい」なんだな。
897ご冗談でしょう?名無しさん:01/09/02 10:54 ID:hu8//5pI
>>895
つまり「空洞派」ってことか(藁

空気の慣性ねぇ。
翼の前縁で翼に「衝突」するところでは圧力が高くなりそうだが、
実際はそーなっていないからダメってことだったぜ。
898前スレ27ではないが:01/09/02 11:51 ID:Nr2gMvJM
>>896
なんか反論になってるのか、ゴネてるだけなのか
わかんないよ。何人もの人が言ってるけど、統計力学
と流体力学(マクロなね)の接続は理論的に完全じゃ
ないよね。揚力の話しはマクロな範疇で話しが付く
んじゃないか。もちろん未開の理論を立てようとして
いるのは賞賛に値するし、現状は興味深く見てる。

しかし、十分な説明ができてるように見えない+的外れ
な反論をするのを見ていると、首をひねらざるを得ないね。

言いたい事は分かるよ。粘性の起源と、流体が壁面で速度
0になる理由を分子論・統計力学から説明したいんだろう。
でも、マクロ的観点から揚力の本質が説明できるんだから、
この統計力学理論って必要か?

>>897
翼は迎え角持ってるから、翼下面(特に前縁側)は
正圧だろ。下面も慣性+粘性で説明できると思うが。
899ご冗談でしょう?名無しさん:01/09/02 13:20 ID:hu8//5pI
>>898
的外れねぇ。キミは
 揚力は翼が空気に与えた下向き運動量の反作用
としながらも
 マクロにみると翼と空気の間には力は働かない
と書いているんだぜ。
力は働かないけど反作用は受けるって、どーゆー状態なの?

>>897の話は翼前縁の上面、「山」の上流側で低圧になってる
とゆーことだぜ。
900前スレ27ではないが:01/09/02 22:07 ID:x48WAIyM
>>899
また、(わざとだと思うが)的外れなことを逝って
ごまかそうといる。

>マクロにみると翼と空気の間には力は働かない
とは俺は言っていない。マクロな物体である翼とマクロ
的にみた空気には引力も斥力も働かないといっているのだ。
ボールをバットで打つと飛んでいくけど、もちろんボール
とバットにには撃力とその反力が働くよ。しかし、ボール
とバットに引力は働いてるか?斥力は働いているか?

>翼前縁の上面、「山」の上流側で低圧になってる
>とゆーことだぜ。
何がおかしい?
その辺りではアップウォッシュになっているから、その
流れのベクトルと、翼表面の接平面のなす角を考えれば、
流れは逸れる方向なので負圧でもおかしくないと思うが。

ひとつ言わせてもらうが、自分の謝りを認めたり、他人の
話しを聞いてそれを訂正することは恥ずかしいことだとは
思わないぞ。君は、揚力の本質が空気に下向き運動量を与
えた反作用だという(以前からあるがよく知られていなかった)
考えを前スレ以来展開してきた。馬鹿な厨房がしきりに叩いて
きたが、その対応態度はともかく、君のおかげで多くの人が
正しい考え方を知ることができた。また、今のところうまく
逝ってないが、統計力学と流体力学(マクロなね)の接続を
試みている。この2点で充分、君は賞賛に値するぞ。これ以上
自分を貶めるような真似はよせ。
901ご冗談でしょう?名無しさん:01/09/02 23:39 ID:7tjGZBjE
>>900
少しハラがたってきたぜ。

「撃力」ってのは接触力だろ。非常に短時間に作用する「斥力」だろ。
違うのかよ。

翼は空気から「圧力」を受けてるんだろ。圧力は法線応力。
この時の応力は翼と空気を互いに引き離そうとする力だろ。
すなわち「斥力」だよ。

「空洞理論」な考えは、もう一度翼周りの圧力分布図を見直してから
出てきて欲しいと思うぞ。でも厨房は要らない。

>ひとつ言わせてもらうが、自分の謝りを認めたり、他人の
>話しを聞いてそれを訂正することは恥ずかしいことだとは
>思わないぞ。

ちょっとワラタヨ。これはオレのセリフだぜ。

このところマケズギライなコドモが多すぎだな。
902ご冗談でしょう?名無しさん:01/09/03 02:23 ID:XY9vKZYY
27派(アンダーソン論文派)-粘性下降気流派=ベル派?

上面だけを考えるとどうしても、そう考えざるを得ないだろうね。

粘性だけじゃ、少なくとも上面の気流は十分に下降しない。
上面の気流は前縁で圧縮され、そして膨張しつつ、進行速度を保ちつつ
翼上或いはその後方に自由落下するんじゃなかろうか。
その際に発生する、速度の全体の平均からのズレが、粘性力として、空気
全体に広く分散的に作用する。
そう。粘性は散逸性の効果であって、一般にそれ自身がエネルギーを作り出
すことは無い。
侵入してくる空気が粘性により、僅かに下降して入射してくるという
ことはあっても、侵入して来る空気が下向きに粘性で加速される
という見方は、ちょっと苦しいのでは?もしそう考えると時間と共に下降気
流のエネルギーが消散してしまう。
上面で下降気流が発生する真の理由は、翼前縁で、圧縮されたエネルギー
の一部が膨張の際に下向きの運動エネルギーに転換された為でしょう。
ベル=オイラー派の立場からすれば。
粘性の影響で、入射前の空気に下向きの成分が付けられていくので、
このエネルギー転換プロセスでのロスが大きくなり、上面での下降気流の
エネルギーが小さくなる流れに近づいていくことも予想出来る。
(ダランベールのパラドックスとの絡み)

下面では、27純正派(下降気流反動派)の活躍する余地が十分残されて
いる。超音速領域では、上面の圧力はさほど低下しない。それは前縁で圧縮
された空気が、殆ど膨張出来ず流れる為。ここに粘性も関わっている可能性
は高い。圧力による等方的な膨張が出来ないとしたら、密度低下がそれほど
起こらないということがありえるからね。だから、超音速領域では多分下面
のほうが上面よりも重要になってくる可能性がある。
下面ではそれこそ、曲げ派の主張するように、気流が水平速度を殆ど
変えず、下に流れていく可能性がある。これも粘性によって、サイド方向
に散逸するのが難しくなっているという説明も出来るかも知れない。

粘性は、それ自身だけでは少なくともエネルギーを作り出す存在では無いと
いうのが、普通の見方であることを踏まえて、今後のアンダーソン氏の論文の
行方を見守るのが、賢明なんじゃないかな?
903前スレ27ではないが:01/09/03 07:30 ID:.0moT.vA
勝手なんだけど「正統反作用派」で良い?
考え方は、下記参考リンクにあるように、空気に下向き
の運動量を与える反作用として揚力が発生する。空気は
あくまで連続体で取り扱い、分子論的扱いはしない。
http://www.allstar.fiu.edu/aero/airflylvl3.htm
http://journal.msn.co.jp/worldreport.asp?id=010529ns_airplane&vf=1

(反作用の立場を取りつつも、分子論的扱いで説明しよう
としているのが前スレ27氏であり、以降、これを27派
と呼ぶことでどうでしょう。)
904前スレ27ではないが:01/09/03 07:49 ID:.0moT.vA
--- 上の続き ----
でも「正統反作用派」は、翼の上下面の圧力差が揚力を与える事
を認めている(=標準派)けど、標準派と同じく、何故翼上面の
流れが速くなるかは説明してないよね。

>902
以後は「正統反作用派」から逸脱する内容かもしれないけど、
>>895で説明した事が誤解されてるみたいなので、もう一度
簡単に書きます。(飽くまで簡単なモデルであり、おかしい
ところ等は教えて。)

粘性ってのは飽くまで翼表面に沿って空気が流れていること
を示します。翼表面に沿って流れなきゃいけないけど、空気
が流れてきた慣性を考えて、更に翼は迎角を取っているを考
えると、翼上面、特に前縁部で空気が膨張する必要がありま
す。すなわち低圧部発生です。

流体の圧力と速度には(例え閉空間でなくても、あるいは擬
似的な閉空間と考えて)一定の拘束関係があると考えられま
す(その特殊な場合がベルヌーイの定理)。つまり低圧部で
は翼に沿う流れの速さが速くなるということ。

後は書くまでもないけど、翼は迎え角を持っており、また
粘性のため空気は上面、下面に沿って流れ、更に翼の後縁
が鋭利なため、翼周りで空気は後縁方向に曲げられる
(下向きの運動量を与えられる。)って事だけど、どう?
905前スレ27ではないが:01/09/03 07:55 ID:.0moT.vA
>902
何度もスマソ。結論として、粘性はエネルギーを生み出してる
訳じゃないって事を言いたかった。
906 :01/09/03 20:30 ID:zk8w1Uvo
どうして粘性にだけこだわるのか?
確かに連続体の内力は、圧力のような、専ら構成している質点間の
距離に依存する性質以外にも、相対速度に依存する成分を持つかも
知れない。(一般にはそう仮定する。但しそうでない場合も含める)
相対速度に依存する成分を持つ場合、その内部応力は粘性を持つと
言う。粘性が無い場合でも、下降気流は、空洞派的な考え方を採用する
ならば、発生しうる。だから、下降気流の発生理由は、粘性というよりも
内部応力であるというのが正確だろう。
907ご冗談でしょう?名無しさん :01/09/03 22:29 ID:2ouKPFQM
>>906
粘性の有る場合と無い場合の流れの違いは下図6だと思うが、
http://journal.msn.co.jp/worldreport.asp?id=010710s_suzuki&vf=1

粘性と呼ばない「内部応力」って具体的に何?
圧力のこと?
粘性が無く圧力だけでも流れが曲げられる(下降気流発生)ってこと?
908 906:01/09/04 09:40 ID:ufIJzfpY
やっと繋がった。
内部応力=圧力勾配+粘性+それ以外の内部応力(藁
要するに、粘性は内部応力の一種
密度が変化しない場合、圧力は常に一定であると仮定され、圧力勾配は0
となる。
しかし、この場合翼に接して流れることを仮定する必要は必ずしも無い
だろう。
粘性があれば、過渡的状態も終局状態も、無い場合と変わって来る可能性
がある。同じ終局状態になることも、渦無しとかの条件があれば起こる
可能性があって、粘性は終局状態への漸近を促進するとも、遅らせる
とも言われていますが、真義の程はまだ分かって無い筈です。
一般の物質では、粘性が流れを曲げるという性質もあってもおかしく
は無いが、通常は空気のように、内部に空洞を持つ物質の場合、圧力も
含めた内部応力を想定する。
空気を分子運動的に取り扱うということは、少なくとも内部に空洞を認めて
密度場を離散個の6次元自由度に分解するということでしょう。
時間統計/空間統計を取った物理量を扱う統計力学では、分子運動論は
一部の分野に過ぎません。
離散的な性質がはっきりと出てこない結論ならば、分子運動論を使った
ことにはなりません。
909前スレ27ではないが :01/09/04 11:17 ID:OkAB6zmM
>908(=906)
まず、正統反作用派は元より、標準派(ベル派)も分子運動論
(簡単な球モデル)なんて考えてないから、後半の数行の部分
>>903-905に対する意見だとすると的外れだよね。

あと、粘性=せん断応力の発生だよね。圧力,せん断応力と
圧力勾配をごっちゃにしているようなので判りにくいんだけ
ど、結論的に、連続体で考えた場合、流体に粘性が無く
(=せん断応力が働かず)圧力のみが作用する場合でも下図6
の右のように流れが曲げられると言っているの?
http://journal.msn.co.jp/worldreport.asp?id=010710s_suzuki&vf=1

俺は流体力学に詳しくないが、それは違うんじゃない?
910ご冗談でしょう?名無しさん:01/09/04 20:40 ID:TKYaptk6
>>902
>上面で下降気流が発生する真の理由は、翼前縁で、圧縮されたエネルギー
>の一部が膨張の際に下向きの運動エネルギーに転換された為でしょう。

都合よく下向きだけに膨張すると考えるのは苦しいのでは?

>>909
せん断応力のみが作用しても流れが曲げられると言っているの?
911前スレ27ではないが:01/09/04 21:23 ID:7hO4Axps
>910
圧力(法線応力)が働かず、せん断応力のみが作用する系
というのが今一つイメージできず、お答えできない。スマソ。

言いたいのは、せん断応力の発生(粘性)が必須ということ。
912Ledの教え子:01/09/04 22:10 ID:yKKpjZzU
段々とお子様会話になって来ましたね > ALL ^^/
事象の解明を探求することは良い事です。
ですが、「どちらが重要か?」などと下らぬ議論に終始していては厨房化してしまうよ。

貴方達が大人であるのならば、節度ある態度で議論に参加して下され!

# 節度を持ってあたれておればとっくに結論が出ていたよね♪
913ご冗談でしょう?名無しさん:01/09/04 22:52 ID:TKYaptk6
>>911
せん断応力は流れとは逆向きに作用するのに翼上面では流速が増加すると言っているの?
914ご冗談でしょう?名無しさん:01/09/05 04:29 ID:v0pK1BE2
>>912
流体にしろ、分子集合体にせよ、圧倒的な数の自由度を相手にしているから
仕方が無いンじゃない?(自由度の少ない剛体ですら似たような話になる)

かなり条件を絞っても、数学的には現象の否定を証明できないことで、物理的
経験的には起こらないことは、結構あるようだが、そういう話に近づいてきた
みたいだな。
こういうものって、大抵物理的経験的には明らかだが、その否定を数学的証明
では出来ない、つまり否定が発生しないことを**保証**出来ないという
ジレンマを持つ。
その「危うさ」が、現象をある程度面白くさせている面があるんだとは思うが。
915前スレ27ではないが:01/09/05 08:14 ID:4ixmFCm6
>113
そうなるかな。でも誤解しないでくれ。
粘性(せん断応力)と翼上面の速度の増加が
直接関係あると言っている訳ではない。

>>904をもう一度参照して欲しいのだが、まず、一番
のポイントは翼が迎角を持っていること。(これに
所謂、標準派(ベル派)が触れてないから彼らの説明
が合理的でない。)

粘性が(=せん断応力が働くこと)により、前方から
流れてきた空気が翼上面に沿って流れる。(= 迎角
をとっているために下に曲げられる)。しかし、慣性の
ため、空気は曲げられずにまっすぐ後方に流れようと
もする。すなわち、空気は膨張しなければならない
(=低圧にならなければならない)。

単純に言えばこの低圧部と下面との圧力差が揚力。
翼上面の流れに関しては、低圧になるから増速するっ
て事。粘性により曲げられた空気が最終的には翼後縁
から流れ出るけど、もちろん、この空気を曲げた
反作用として揚力が生じているという見方もできる。
916前スレ27ではないが:01/09/05 08:15 ID:4ixmFCm6
↑ ゴメソ、>>915>>913へのレスね。
917ご冗談でしょう?名無しさん:01/09/05 10:27 ID:apH3CFDQ
>>915
>翼上面の流れに関しては、低圧になるから増速するって事。

外力が作用していても常にベルヌーイの定理が成立すると言っているの?

>この空気を曲げた反作用として揚力が生じているという見方もできる。

翼が反作用を受けるためには空気に対する作用が必要だけど
せん断応力がその作用だと言っているの?
918前スレ27ではないが:01/09/05 11:31 ID:0cVkBCTM
>917
>外力が作用していても常にベルヌーイの
>定理が成立すると言っているの?

いや、そうは言っていない。実際の空気には、外力
を受けようが受けまいが、圧力と速度には何かしら
の拘束関係があるんだと思うっていう事。つまり、
低圧→速度大ってこと。(定式化されてないのかな?)

これの特殊な場合、すなわち外力が作用せず(=
エネルギーが保存され)且つ特殊な流体(粘性の無い
非圧縮性流体)の場合に成り立つのがお馴染みのベル
ヌーイの定理だよね。定式化されてて、もちろん、
低圧→速度大

>翼が反作用を受けるためには空気に対する作用
>が必要だけど、せん断応力がその作用だと言って
>いるの?

俺の理解力が今一つなのではっきり判らないが、
違うんじゃないかと思う。(いや、正直判らない)

作用・反作用の考え方は大雑把な見方で、単に
入ってきた空気と翼後縁から出て行く空気の流れ
速度・方向が変化してるから翼には力が働いてい
るという見方だよね。
    
  mv1 - mv2 = Ft
v1,v2は入出時の空気の速度ベクトル
Fは揚力ベクトル(正確には+抗力ベクトル)
     mは空気の質量(密度)、tは時間

これはこれで正しいんだけど...。
もうちょっと考えてみるけど、今のところ正直判らない。
あなたはどう思う?
(翼と空気に引力が働く、は無しだよね。)
919ご冗談でしょう?名無しさん:01/09/05 12:53 ID:GNvsuCS.
>>918
結局、説明の根拠とするところは「思うって事」や「正直判らない」ことだけど
それでもまだ自分の考えが正しいと言っているの?

空気に対する力積の反作用が翼の揚力であることを認めるならば、
それは「翼と空気に引力が働く」ことを認めているのと同じ。
引力を否定するから「正直判らない」んだよ。

で、
>ひとつ言わせてもらうが、自分の謝りを認めたり、他人の
>話しを聞いてそれを訂正することは恥ずかしいことだとは
>思わないぞ。

物理はわかってるみたいだから、もう一度ゆっくり考え直してから
出直して欲しいと思うぞ。
920ご冗談でしょう?名無しさん:01/11/04 18:04 ID:2aQiwR6O
「酸」と関係ないのになぜ「酸素」?
http://nara.cool.ne.jp/mituto
921 :01/11/05 00:02 ID:yra1rhkn
922超初心者:01/12/27 04:19 ID:CpiG2ssv
既出かもだけど、参照までにNASAのFoilSimでも
http://www.grc.nasa.gov/WWW/K-12/airplane/foil2.html
923ご冗談でしょう?名無しさん:01/12/27 08:30 ID:???
>>919
 空気に対する力積の反作用?
924ご冗談でしょう?名無しさん
>>923
冬厨か?
このスレは物理を解さない航空ヲタとデムパなオヤジが
来ないと盛り上がらないんだよ。